VTP animal nursing

Pataasin ang iyong marka sa homework at exams ngayon gamit ang Quizwiz!

What is the normal respiratory rate of dogs?

10-30 breaths per minute Dogs have a resting respiratory rate of approximately 10-30 breaths per minute. Horses have a respiratory rate of approximately 8-20 breaths per minute. Cats breathe 24-42 times per minute at rest.

What is colic in a horse?

Abdominal pain that can be secondary to multiple etiologies The best answer choice is abdominal pain that can be secondary to multiple etiologies. It is a very broad term, and colic can be secondary to any of the other answer choices. In effect, knowing that the patient has colic does not necessarily mean the clinician knows exactly what treatment to provide until the underlying cause of colic is identified.

What percentage of hemoglobin should be saturated with oxygen in a healthy animal?

>95% In a healthy animal, 95 to 100% of the hemoglobin should be saturated with oxygen. Below 90%, suggests fairly significant hypoxemia.

If a dog has atrophy of its muscles, what appearance will the muscles take on?

Appear sunken with loss of tone Atrophy means to decrease in size. Hypertrophy is to enlarge in size.

What is the average gestation length of a cow?

Approximately 285 days Cows have an average gestation length of 285 days (about 9 months). The average gestation length of dogs and cats is 63 days. Horses have a gestation length of approximately 340 days. Although the average for horses is approximately 11 months, it can vary by as much as 30 days.

What is the average gestation length of a horse?

Approximately 340 days Horses have a gestation length of approximately 340 days. Although the average is approximately 11 months, it can vary by as much as 30 days. Cows have an average gestation length of 285 days (about 9 months). The average gestation length of dogs and cats is 63 days.

How often should an IV peripheral vein catheter be replaced as a standard?

Every 3 days It is recommended to replace a peripheral catheter every 72 hours (3 days). The risk of catheter-induced sepsis goes up after this time period. Sometimes they have been left in for up to 4-5 days. Obviously, the site should be watched closely for any discharge, pain, and swelling and if these occur the catheter should be removed and another placed at a different site.

Which of the following is NOT a use for an indwelling arterial catheter?

Constant rate infusions Injections should never be given into an arterial catheter (a-line). Arterial catheters are a great way to get real-time blood pressure measurements. These measurements are also helpful when trying to evaluate general cardiac output. They are often used to collect small blood samples, most notably for blood gas measurements.

On average, cows have their estrous cycle how often?

Every 21 days Cows are non-seasonal polyestrous (have estrous cycles year-round). The average estrous cycle in the cow is 21 days (every 18-24 days).

Canine Distemper Virus can be transmitted to which species?

Ferret Raccoons, ferrets, skunks and some other species can contract Canine Distemper Virus.

Which of the following mostly produces endotoxins?

Gram-negative bacteria Gram-negative bacteria release endotoxins (which are found in their cell wall) when they die.

What species offspring is labeled "precocious"?

Guinea Pig Due to the long gestation period for guinea pigs, the development of the fetus occurs in utero. After 58 days, the young are born with their eyes open, fully furred and able to walk. This is know as "precocious". The opposite is "altricial". This means to be born with eyes closed, hairless and helpless.

Colostrum provides passive immunity to the neonate because it is rich in which of the following?

Immunoglobulins Immunoglobulins are antibodies that are essential to the neonate to protect against infections.

The clinical condition of "founder" in the horse refers to what?

Inflammation of the laminae of the equine hoof and subsequent rotation of the third phalanx Founder, also known as laminitis, is a severe and debilitating disease of the equine hoof that results from inflammation of the interdigitations (laminae) between the hoof and the 3rd phalanx, causing rotation of the bone.

Which organism causes pinkeye in cattle?

Moraxella bovis Moraxella bovis is the bacterium that causes bovine keratoconjunctivitis (pinkeye) in cattle. It is transmitted from one bovid to another via flies, fomites, or direct contact. There are many different types of bovine herpesvirus that can result in different types of presentations. BHV-1 also known as infectious bovine rhinotracheitis virus can occur concurrently with moraxella bovis, complicating the disease. Arcanobacterium pyogenes can cause wound infections and mastitis. Dermatophilus congolensis contributes to "rain scald"

What is normal tear production for a dog when performing a Schirmer Tear Test?

More than 15 mm/min. Normal tear production is more than 15 mm/min. on a tear test strip. Low tear production is termed "dry eye" or keratoconjunctivitis sicca (KCS). Excessive tearing clinically is termed "epiphora".

Which species has ovaries that look like grape clusters?

Pigs In the pig the follicles and corpora lutea sit almost on the surface of the ovary giving the appearance of a cluster of grapes. Pigs are also litter-bearing and thus usually have more follicles.

Which species other than cattle is considered cloven hoofed?

Pigs A cloven hoof is a hoof split into two toes. This includes cattle, sheep, goats, deer, and pigs. Camels have two digits in their feet (III and IV) which are almost flat on the ground as part of a wide pad. Instead of hooves, the distal phalanges have nails on the upper surface.

An owner presents her diabetic cat and new glucometer with lancets. Where is the best place for her to collect a sample?

Pinna Lancets are best to use on superficial smaller veins such as veins on the ear. Pads are very thick and therefore difficult to use for sample collection.

You are assisting with a bone marrow aspirate on a dog with suspected neoplasia. You are asked to make slides to send to the laboratory with the samples the veterinarian is collecting. Which describes proper technique for this?

Place a drop on one end of the slide, tilt slightly and make a pull smear Pull smears should be made of bone marrow samples. The pull smear should be in a light fashion as to spread the fluid but not to crush the samples. The sample should be dried immediately to preserve integrity of the cells. A blow dryer can assist by quickening drying time. A stain of one of the slides should be performed to make sure the sample appears adequate before submission.

How is tularemia transmitted (what is the vector)?

Ticks The correct answer is ticks. Ticks are the usual vector, although flies and fleas may be able to transmit Francisella tularensis as well.

When giving an intramuscular injection, why do you pull back suction on the syringe before injecting?

To make sure you are not inadvertently in a blood vessel When giving an intramuscular injection, you should pull back suction on the syringe to make sure that you are not accidentally in a blood vessel. Certain medications, if inadvertently given IV instead of IM, could cause serious concerns.

If a disease is zoonotic, what does this imply?

Transmissible from animals to people A zoonotic disease is an infectious disease in animals that can be transmitted to people.

The clinician has been unsuccessfully treating a chronic cough. A culture of the lower airways is needed. What is the best method for this?

Transtracheal wash Transtracheal wash is used when a culture of the lower respiratory tract is needed. It is typically done under general anesthesia for small dogs or cats or under a local in larger dogs. It is a method of collecting bronchial exudate for culture and cytology. The procedure entails inserting a catheter into the trachea in the ventral neck region between two tracheal rings, and infusing and aspirating sterile saline in order to get a sample.

When collecting an arterial blood sample, the best location to obtain arterial blood in the standing adult horse is where?

Transverse facial artery The transverse facial artery is the easiest and most commonly used artery to collect arterial blood in a standing adult horse. It is typically located caudal and ventral to the eye and is easily detected. The dorsal metatarsal and carotid arteries are possible sites to collect arterial blood, but an adult horse typically will not stand still to collect blood from the extremity (hind limb). The carotid is also possible to collect arterial blood but is a deeper structure and is obviously a very large vessel; damage to this vessel during collection can be problematic.

Where is the thyroid gland located?

Ventral neck The thyroid gland is located in the ventral neck region.

Pyuria indicates which of the following?

White blood cells in the urine Pyuria indicates white blood cells in the urine (Py=Pus, uria=urine). Pus is an accumulation of white blood cells. Hematuria is red blood cells in the urine. Proteinuria is protein in the urine. Bilirubinuria is bilirubin in the urine. Bacteriuria is bacteria in the urine.

You are asked to run an electrocardiogram (ECG) strip on a pet. There are three leads on your machine. One is white, one is black, and one is red. Where should these be placed?

White: right forelimb, black: left forelimb, red: left hindlimb White= right front Black= left front Red= left hind Green (if present)= right hind The easy ways to remember these: You read your newspaper in the morning (black and white towards front of pet), Christmas comes at the end of the year (red and green towards the rear of pet). In right lateral recumbency: snow (white) and grass (green) are on the ground so are on the right side down towards the table. White also rhymes with right.

Which is responsible for the transmission of Yersinia pestis?

Xenopsylla cheops Xenopsylla is known as the plague or rat flea. Yersinia pestis are gram-negative rod shaped bacteria that are the cause of plague. Plague transmission from infected animals generally occurs via bites from infected rodent fleas (Xenopsylla), direct contact with infected tissues or fluids, or breathing infected droplets. Dipylidium and Vampirolepis are both tapeworms. Dermacentor is the American dog tick.

What is the "top" shell of a tortoise called?

Carapace The carapace is the top shell and the plastron is the bottom shell. Patagium is the skin or membrane that extends between the body and a limb or wing (such as seen in a flying squirrel or wing of a bat). A scute is a bony external plate or scale. A choana is either of the pair of posterior openings between the nasal cavity and nasopharynx. The cloaca is the posterior opening that serves to pass feces, urine, and reproductive excretions (connects the rectum, vagina, and urethra into a single channel), such as in reptiles and birds.

What is the term given to the end of the spinal cord?

Cauda equina The cauda equina is at the lower end of the spine and is made up of nerve roots. The base of this region includes around 3-5 lumbar, 5 sacral, and a coccygeal nerve. The reason it is called the cauda "equina" is that the appearance of this area grossly resembles a horse's tail.

The rear end of an animal is referred to as which of the following regions?

Caudal region Cranial is toward the head. Caudal is toward the tail. Ventral is toward the abdomen; dorsal is toward the backbone. Lateral is away from midline and medial is toward midline. Proximal is close to the spine or body while distal is away; these are typically used when describing limbs (e.g. fracture of the proximal femur vs. fracture of the distal femur). Palmar is the bottom of the paw of the forelimb; plantar is the bottom of the paw of the hind limb. Anterior is toward the head and posterior is away from the head. In veterinary medicine, anterior and posterior are usually used only to describe distal extremities or areas of the head.

Where is the standard tuberculosis test administered in the bovine species?

Caudal tail fold The TB test in cows is given intradermally in the caudal tail fold. This area is used because it is easy to give the injection at this location and a reaction can be easily seen. The eye lid is used in primates.

Where should you palpate a standing animal for the urinary bladder?

Caudoventral Caudo is towards the tail, ventral is towards the ground. The bladder is located caudoventral.

A 285-gram guinea pig weighs:

0.285 kg 1 kilogram equals 1,000 g 285 g/ 1,000 g/kg = 0.285 kg If you want to convert into pounds, multiply kg X 2.2 pounds/kg. 0.285 kg x 2.2 pounds/kg = 0.627 pounds

You draw up a canine distemper-parvo combination vaccine. How long is the vaccine considered viable at room temperature?

1 hour Vaccines should be discarded if they have been left out at room temperature for longer than one hour. Most references state that if they are refrigerated they can be used within 24 hours, but this is controversial. The best option is to use the vaccine shortly after it is drawn up to avoid any concerns.

A 620-kg horse weighs how many pounds?

1,364 pounds There are 2.2 pounds per kg. So, to convert the kg into pounds, multiply the kg by 2.2. 620 kg X 2.2 pounds/kg = 1364 pounds

What is the normal amount of gastric reflux obtained from a healthy horse via nasogastric tube placement?

1-3 liters Normally, a healthy horse will have a small amount of gastric reflux, 1-3 liters. If you get back 8-12 or more liters of reflux, the horse likely has an obstructive intestinal disease or ileus of some sort.

What is the normal temperature for a cat or dog?

100-102.6F Normal temperature for dogs and cats is around 100 to 102.5F. When they are stressed or excited, the temperature may increase somewhat or may remain normal for some patients. Therefore, a temperature slightly higher than the reference range should be interpreted based on clinical status of the patient.

A dog refuses a rectal temperature. You take his temperature under the axillary region and get a reading of 99.5F. What is likely his core body temperature?

101.5F Typically a temperature taken in the ear or under a peripheral limb reads 1 to 2 degrees lower than rectal temperature, or core body temperature.

What is the average cloacal temperature of a chicken?

107F The deep body temperature of a chicken is around 107 degrees Fahrenheit.

How many pairs of cranial nerves are there?

12 There are twelve pairs of cranial nerves. They are designated by Roman numerals as follows: I - Olfactory nerve II - Optic nerve III - Oculomotor nerve IV - Trochlear nerve V - Trigeminal nerve VI - Abducens nerve VII - Facial nerve VIII - Vestibulocochlear nerve IX - Glossopharyngeal nerve X - Vagus nerve XI - Accessory nerve XII - Hypoglossal nerve

Pregnancy check in the mare can be performed via ultrasound as early as which of the following?

14 days Ultrasound is the method of choice for detecting pregnancy in the mare and can be done as early as 11 to 14 days (around 2 weeks).

The gestation period for hamsters makes them a valuable research model. What is their gestation period?

15 days Hamsters have the shortest gestation period of all the rodents. Their rapid reproduction makes them valuable in the biotech field, saving on cost. In addition, it allows quick results for any reproductive study.

What is the maximum volume that can be injected intramuscularly in the larger muscle groups of an average adult horse?

15 ml Volumes approaching 15mls or over, should be divided into 2 different sites.

What is the average gestation for goats and sheep?

150 days This list of gestations should be committed to memory: Llama 1 year (350 days), Horse 11 months (330 days), Cow 9 months (280 days), Sheep/Goat 5 months (150 days), Pig 4 months (114 days), Dog/Cat 2 months (63 days), Ferret 1.5 months (42 days)

The heart rate of an adult cat should range between which of the following?

150-220 bpm An adult cat typically has a resting heart rate between 150-220 bpm. Keep in mind that if the cat is very excited the heart rate might be faster. A dog's heart rate ranges between 70-160 bpm. As with the cat, there may be variation. Remember that puppies and kittens will have faster heart rates.

How tall would a horse that measures 66 inches be, in hands?

16 - 2 hands A hand is equivelent to 4 inches. Thus 66 divided by 4 = 16 hands + 2 inches or 16 - 2 hands.

How many thoracic vertebrae do horses have?

18 Horses have 18 thoracic vertebrae. Dogs, cats , cows, and sheep have 13 thoracic vertebrae. Pigs have 14-15 thoracic vertebrae. Humans have 12 thoracic vertebrae.

How long after birth can foals absorb protective maternal antibodies?

18-24 hours Foals and calves can absorb maternal antibodies from colostrum directly into the bloodstream for 18-24 hours after birth. After this time the gut closes for this direct transfer. Calves should get at least 2 liters and foals 1 liter of colostrum before this occurs. Adequate colostrum helps prevent against failure of passive transfer which can lead to increased risk of infections.

Determination of adequate passive transfer of maternal antibodies (IgG) to a neonatal foal is typically performed at what age?

18-24 hours of age Newborn foals are immunocompetent but have few antibodies in their body when they are born; thus, they rely on acquiring antibodies from their dam's colostrum. In the foal, the IgG should be checked around 18-24 hours of age to allow for the foal to absorb the maternal antibodies that are consumed. The foal's gut "closes" or loses its ability to absorb colostral antibodies around 18 hours of age.

How many mammary glands does a horse have?

2 Horses have 2 mammary glands. Cows have 4 mammary glands. Extra mammary glands can occur, mostly in cows, but are usually non-functional and termed "supernumerary".

How many mammary glands does the goat have?

2 The goat has 2 mammary glands. The cow has 4 mammary glands; the horse has 2 mammary glands.

Approximately what percentage of body weight should a horse consume in forage (i.e. hay) per day?

2% The average horse should consume approximately 2% of its body weight in forage. In a 1000 lb horse, this would equal 20 lbs of hay/day.

A neonatal foal should have his immunoglobulin G (IgG) measured at what time after birth?

24 hours Neonatal foals have a functioning immune system but have minimal antibodies present in the blood when born; therefore, they rely on passive transfer of maternal antibodies by consumption of colostrum from the mare. The IgG should be evaluated in foals between 16-24 hours of age. This is testing for failure of passive transfer.

What is the normal respiratory rate of a cat?

24-42 breaths per minute Cats have a resting respiratory rate of approximately 24-42 breaths per minute. Dogs have a resting respiratory rate of approximately 10-30 breaths per minute. Horses have a respiratory rate of approximately 8-16 breaths per minute.

The heart rate for an adult horse should generally be in which of the following ranges?

25-50 beats per minute Horses on average have a heart rate of around 35 beats per minute (with a range of 25 to 50 beats per minute).

A patient is slowly being weaned off fluid therapy. Her body weight is 10 kg, and she is currently at a rate of 30 ml/hr. The doctor wishes to decrease her fluid rate by 15%. What is her new rate?

25.5 ml First, we need to determine how much 15% of 30 ml is. 30 ml x 0.15 = 4.5 ml Now we subtract the answer above from the original rate. 30 ml - 4.5 ml = 25.5 ml is the new fluid rate. Notice that the body weight is not necessary to calculate this rate change.

How many chambers does the stomach of a llama have?

3 The llama has a stomach that functions similar to that of a true ruminant, but it only has 3 compartments (rumen, omasum, and abomasum). True ruminants such as cows, goats, sheep, and deer have 4 compartments including the rumen, omasum, abomasum, and the reticulum.

How many feline blood types exist?

3 There are three feline blood types: A, B, and AB. Group A is the most common, as 98% of cats are type A. Type B is more rare and seen most often in some exotic breeds. Only approximately 1.7% of cats have type B blood. AB is rare in all breeds. Always crossmatch cats unless you know their blood types. Dogs have 13 blood types.

When conducting a food trial for possible food allergy manifesting as allergic skin disease, how long as a minimum should the trial last?

3 months For food-related skin allergies, a food trial should last at least 3 months. For possible food allergy manifesting as inflammatory bowel disease, the trial should last at least 3 weeks. A food trial should consist of a novel protein and a novel carbohydrate source such as Rabbit, Duck, Venison, etc. Another option is a diet with hydrolyzed proteins such that the body cannot form an immune response (Z/D is an example of this).

What is the ideal age for tail docking in puppies?

3 to 5 days Tail docking in puppies should occur within 3 to 5 days after birth.

A puppy presents for his first distemper-parvo puppy shot. He is currently 8 weeks of age. When should he receive his next set of vaccines?

3 weeks from now The puppy series starts at 6 to 8 weeks of age, and the distemper-parvo combo vaccine is given 3 weeks apart for a total of 3 to 4 vaccines, depending on the breed. Black-and-tan breeds such as Rottweilers are thought to be more susceptible to parvo-virus, so this breed may receive a series of 4 to 5 vaccines, depending on the age and status of the pet.

You are helping admit a patient to the hospital. Rudy is a 5-year old female spayed Yorkie with pancreatitis. The hospital is very busy and all of the fluid pumps are currently being used by other patients. You are asked to start fluids at 24mL/hr for Rudy. Calculate the drip rate for this patient. Her body weight is 8 pounds. The drop size for your administration set is 10 drops/mL.

4 drops/minute The body weight given in this question is irrelevant. You already know what rate you need. You would only need the body weight if you were calculating her rate. Formulas for drop sizes per mL are as follows (you may wish to memorize these): 10 drops/mL: (mL/hr) / 6 = drops/minute 15 drops/mL: (mL/hr) / 4 = drops/minute 20 drops/mL: (mL/hr) / 3 = drops/minute 60 drops/mL: (mL/hr)/ 1 = drop/minute So to calculate using these simple formulas: 24/6 = 4 drops per minute Or, If you did not know this formula, or if you forget, just convert it using what you know. First convert hours into minutes: 24 mL per hour divided by 60 minutes in an hour: 0.4 mL per minute You administration set will give 10 drops per 1 mL. So, multiply 0.4mL/min X 10 drops/1mL (the mLs cancel out) = 4 drops/minute Now check your math: 4 drops per minute X 60 minutes in a hour (minutes cancel out)= 240 drops per hour Your set gives 10 drops in 1 mL. 240 drops/10 drops/mL (drops cancel out) = 24 mL per hour (correct)

You remove an IV catheter from a peripheral vein and place a pressure bandage. What is the maximum amount of time this bandage should stay on?

4 hours Venous return can become compromised if a compression bandage or pressure wrap is left on for more than 4 hours. It is best to remove the pressure bandage once the bleeding has stopped so that the bandage is not forgotten and venous return is not compromised.

How long does estrus in the dog typically last?

4-13 days Estrus typically lasts 4 to 13 days, with an average of 9 days. The entire estrous cycle in the dog is approximately 6 months.

At what age should pigs be castrated?

5 - 14 days Piglets are castrated very young due to ease of handling.

Regarding fluid types, which cannot be given subcutaneously?

5% Dextrose Only use isotonic fluids for subcutaneous administration. This means that the osmotic properties are equal to that of the extracellular fluid. Dextrose given subcutaneously can cause sloughing of the skin and abscess formation.

A cat is starting weekly subcutaneous fluid administration for renal failure at the clinic where you work. The client asks how long it will take for him to absorb the fluids after they are given. You tell him most cats will absorb the fluid within:

5-8 hours Subcutaneous fluids are most commonly absorbed within 5 to 8 hours of administration. However, fluids are absorbed when they are needed, and absorption is dependent on having adequate SQ blood flow, which can be decreased in severely hypothermic or dehydrated patients.

You are planning to get some goats out on your ranch property and would like to keep them confined to a certain area to keep the weeds down. How tall of a fence (minimum) would likely be required to keep the goats confined?

6 feet Fencing is a very important aspect of raising sheep and goats. Most goats can jump or climb over a fence that is shorter than 6 feet, so this would be the minimum recommended height.

Without a heartworm test, what is the latest age that heartworm preventative could safely be started in a dog?

6 months Heartworm prevention should be started at 6 to 8 weeks of age. Most heartworm preventatives can be started once an animal is over 2 pounds body weight. Since heartworm prevention kills microfilaria, heartworm preventative must be started before 6 months to insure that the microfilaria have not matured to the adult stage.

What is the maximum amount of oral fluids that a horse should receive at a time?

6-8 Liters The average size horse (800 to 1200 pounds) has a stomach capacity of only about 4 gallons. It is best to give smaller quantities of fluids more frequently, and no more than 6 to 8 liters should be given at one time.

When collecting blood for a blood transfusion, the maximum recommended amount that can be collected from a donor horse (500 kg) is what volume?

6-8 liters A safe amount of blood to collect from a horse is 15-16 ml/kg, equaling 6-8 liters in a 500 kg horse. Typically, blood is only collected from a donor every 30 days or longer.

How many cervical vertebrae do cows have?

7 Cows as well as dogs, sheep, pigs, cats, goats, horses and humans have 7 cervical vertebrae.

How many lumbar vertebrae do dogs and cats have?

7 Dogs and cats have 7 lumbar vertebrae. Horses and cows have 6 lumbar vertebrae. Humans only have 5 lumbar vertebrae. Pigs and sheep can vary in the number of vertebrae they have (6-7).

A breeder calls and wants to know how long sperm may be viable to fertilize an egg after dogs have mated. What will you tell her?

7 days Canine sperm may live or be able to fertilize an ovum for about 7 days.

The heart rate of an adult dog should range between which of the following?

70-160 bpm A dog heart rate ranges between 70-160 bpm. As with the cat, there may be variation. An adult cat typically has a resting heart rate between 150-220 bpm. Keep in mind that if the cat is very excited, the heart rate might be faster. Remember that puppies and kittens will have faster heart rates.

Socialization plays an important role in companion animal behavior. At what age is it vital that painful and/or traumatic experiences be avoided?

8-10 weeks Between the ages of 8 and 10 weeks puppies experience a fear period. Because of this, painful and traumatic situations should be avoided. Between the ages of 3 and 8 weeks puppies socialize best with other dogs. At 16-20 weeks puppies go through a period when they are curious and exploring new environments.

The respiratory rate for an adult horse should generally be:

8-20 breaths per minute Horses breathe at a rate of 8 to 20 times per minute. Dogs breathe 10-30 times per minute at rest; cats breathe 24-42 times per minute at rest.

Some orphaned kittens are left on the doorstep of the clinic where you work. You volunteer to take care of these kittens. What volume can the stomach of a kitten or puppy hold on average?

9 mL/kg The stomach volume of a kitten is around 9 mL/kg. Giving more than this at a time can lead to regurgitation and causes increased risk of aspiration.

What percentage of mammary gland tumors in the cat are malignant?

90% Mammary tumors in the cat have about a 90% chance of being cancerous. In the dog, the chance is closer to 45-50%.

While caring for a week old orphan puppy, it is important to monitor body temperature. Rectal temperature should be between:

94-99F If the temperature falls below 94F, the heart rate will drop and intestinal motion ceases. Death will occur if not corrected.

What is the normal rectal temperature for a horse?

99-101.5F Average rectal temperature for a horse is around 100 degrees Fahrenheit. Averages for other species are: Cattle 101F, Cat/Dog 101-102F, Goats/Sheep 102.3F, Pig 102.5F, and Rabbit 103.1F

A female dog has some vaginal bleeding, swollen vulva, and attracts males but will not allow them to mount. What stage of the estrus cycle is she in?

A female dog has some vaginal bleeding, swollen vulva, and attracts males but will not allow them to mount. What stage of the estrus cycle is she in? The correct answer is proestrus. During this phase, there is vulvar swelling, vaginal bleeding, and attraction of males but no mounting is allowed. When the female will stand to be mounted, she is in estrus.

What is an osteosarcoma?

A malignant bone tumor An osteosarcoma is a malignant bone tumor. Osteosarcoma is the most common bone tumor in dogs and has a 90% metastatic rate. An osteoma is a benign bone tumor. Tumors of the skull sutures are rare and do not have a specific name; a benign bone tumor of the skull would be a skull osteoma. There are various types of cancerous splenic tumors with hemangiosarcoma (a tumor of blood vessels) being the most common. A benign tumor of cartilage is a chondroma.

Which ICU patient on fluids needs to be watched closely for jugular distention?

A patient with a heart murmur A patient with underlying cardiac disease is at the highest risk of pulmonary edema (fluid in the lungs) from fluid overload. An indication of possible fluid overload is jugular distention. A good way to monitor hydration status of patients like this in the ICU is via central venous pressure readings.

A client calls and asks if you could do a necropsy for her pet. What is she asking for?

A post-mortem exam to be performed in an effort to find out the cause of death Necropsy is examination of an animal after death to try and discern possible causes of death by determining abnormal changes or disease processes which occurred during life. This procedure should only be performed with client consent. Necropsy should be performed immediately if possible. If not, the body should be refrigerated (not frozen) until the procedure can be performed.

Which organization establishes standards for label information and the description of ingredients on pet food sold in the United States?

AAFCO AAFCO is the Association of American Feed Control Officials and is the organization that sets these standards. They ensure that adequate information is communicated to the consumer about the food product.

What is the most common cause of flexural and angular deformities in horses?

Abnormal development Flexure and angular deformities occur as a result of musculotendinous and musculoskeletal growth deformities. These abnormalities may be present at birth or develop over the first few months to years of life. Treatment may be as simple as corrective shoeing or bandaging, but more advanced cases may require surgical intervention. The three main types of flexural deformities are carpal, distal interphalangeal (coffin joint), and metacarpophalangeal.

If a snake is undergoing "dysecdysis", what does this mean?

Abnormal shedding of the skin "Ecdysis" means the shedding or sloughing of the skin and occurs in reptiles including snakes. Dysecdysis is the term used when there is abnormal or incomplete sloughing. In pet reptiles, this is most commonly due to a habitat that is not sufficiently humidified.

Which of the 4 stomach compartments in the ruminant is considered the true glandular stomach?

Abomasum The abomasum is the true glandular stomach and is responsible for mixing food with digestive enzymes. The omasum grinds up the food material and absorbs water and bicarbonate. The rumen is the where all the fermentation is occurring thanks to the help of symbiotic organisms that help break down the food. The rumen occupies the majority of the LEFT side of the abdominal cavity. The reticulum acts as a conduit for food and helps pace the contractions of the rumen.

Trichomoniasis causes which of the following conditions in cattle?

Abortions Trichomonas fetus (trichomoniasis) is a venereal disease of cattle that causes infertility and abortions. It is a protozoan found only in the reproductive tract of the bull and cow.

What is a localized accumulation of pus in the body is known as?

Abscess An abscess is a collection of pus that accumulates and is surrounded by inflammatory tissue. A cyst is a closed-off sac that may contain fluid, air, semi-solid material, etc. A vesicle is a small blister on the skin or mucosa, similar to a cyst. Adenoma is a benign glandular tumor. Seborrhea is a skin condition characterized by either greasy skin from excessive sebum (overproductive glands in the skin), or dry skin characterized by flaking scales.

A young male cat with symptoms of feline lower urinary tract disease and struvite crystalluria should receive:

Acidifying diet Struvite crystals form when the pH of the urine is high or alkaline. Therefore, an acidifying diet would help to decrease the pH of the urine and prevent formation of the crystals. It can often be of benefit in cases of FLUTD in addition to increasing water intake (canned diets can help with this) and decreasing stress in the home.

Which disease can cause a severe bradycardia that could be life-threatening?

Addison's disease Hypoadrenocorticism (Addison's disease) can cause a high potassium level (hyperkalemia) and can be life-threatening. Addison's occurs when the adrenal glands do not produce enough cortisol, resulting in a crisis. A low heart rate is often associated with high potassium. Sodium chloride is the fluid treatment of choice to help reduce the potassium.

In cattle, a balling gun is sometimes used for which of the following.

Administering medications A balling gun is used to give medication boluses to cattle. Fluids and medications can also be given via orogastric tube.

The licensure we receive before being able to practice as a veterinary technician is an example of which branch of ethics?

Administrative ethics Administrative ethics are defined as: Rules set by governmental bodies that regulate veterinary medicine.

Which organ is responsible for producing cortisol?

Adrenal gland The adrenal gland is responsible for producing specific hormones in the body, including cortisol, epinephrine, and aldosterone.

The veterinarian diagnoses a tension pneumothorax in a dog that was hit by a car. What does this mean?

Air is building up in the pleural space and can't escape Tension pneumothorax is when air accumulates under pressure in the pleural space (around the lungs, not in the lungs). This condition develops when a tear or hold in the chest forms a 1-way valve, allowing air to enter the pleural space but preventing the air from getting out. In this situation, the pressure inside the chest exceeds the pressure outside the chest.

Which food is a good source of calcium for large animals?

Alfalfa Legumes (such as clover and alfalfa) and beet pulp are all high in calcium; corn, oats, and barley are low in calcium.

You are assisting with chemotherapy treatments. Which of the following is true regarding blood collection from chemotherapy patients?

All blood samples should be drawn from the jugular if possible Blood should be drawn from the jugular veins when possible. Peripheral veins are spared for intravenous chemotherapy administration or for catheter placement during hospitalization stays. The other answer choices are false.

The doctor is trying to retrieve a foreign body out of a dog's ear and is using this tool (see image). What instrument is this?

Alligator forceps The alligator forceps is a two-bladed instrument with a handle for compressing or grasping tissues. It works really well for grasping foreign bodies from the ear canal.

A chihuahua is born with no hair. What is a term for no hair?

Alopecia Alopecia is lack or loss of hair/baldness. This chihuahua has hypotrichosis (lack of hair). It can be familial in this breed and is also more common in poodles. Hypertrichosis is excessive hair. Mange is a skin condition caused by mites that can lead to alopecia and skin pathology. Pruritus is the term for itchy.

What causes cutaneous larval migration in humans?

Ancylostoma The correct answer is Ancylostoma. These are hookworms; when they come in contact with unprotected skin, the infective larvae penetrate the epidermis but generally cannot penetrate the basement membrane. They therefore migrate aimlessly, and the disease is usually self-limiting in humans.

Which of the following can potentially infect and cause dermatitis in a child walking barefoot through a field?

Ancylostoma spp. The correct answer is Ancylostoma spp. Ancylostoma caninum, the dog hookworm, can infect a human by penetrating skin. It can cause dermatitis via cutaneous migration of the parasite. Toxocara is a roundworm that causes visceral larval migrans.

A 4-year-old domestic short hair cat presents with her pupils being noticeably of two different sizes (see image). What is the proper term for this condition?

Anisocoria Anisocoria is the proper term for two differently sized pupils. It can have a variety of causes, including Horner's syndrome, trauma, middle ear disease, inflammatory conditions, or neoplasia. It is also important to know the meaning of the other terms listed. Buphthalmos is enlargement of the globe or eye itself. Mydriasis is the dilation of both pupils; miosis is the constriction of both pupils.

What is the average gestation length of a cat?

Approximately 60 days The average gestation length of dogs and cats is 63 days. Horses have a gestation length of approximately 340 days. Although the average for horses is approximately 11 months, it can vary by as much as 30 days. Cows have an average gestation length of 285 days (about 9 months).

Assuming the daily maintenance fluid requirement is 60 ml/kg/day, how many milliliters per hour should a 4400 lb animal receive per minute to achieve this requirement?

Approximately 80 ml/min First, convert the body weight to kilograms. 4400 lbs / (2.2 lb/kg) = 2000 kg Next, calculate daily fluid requirement. 2000 kg x (60 ml/kg/day) = 120,000 ml/day Now, divide by 24 to figure out the hourly requirement. 120,000 ml/day / (24 hr/day) = 5000 ml/hr Lastly, divide by 60 to calculate the amount of fluid necessary per minute. 5000 ml/hr / (60 min/hr) = 83 ml/min

The natural ovulatory season for mares in the northern hemisphere occurs during approximately which months?

April - August The natural ovulatory season for mares occurs during April through August and September in the northern hemisphere. Some horse breeders will induce early ovulatory cycles in mares by supplying artificial lighting during the winter months.

What is the fluid in the anterior chamber of the eye called?

Aqueous humor Aqueous humor is the fluid in the anterior chamber of the eye. It is produced by the ciliary body. The vitreous humor is the gel filling the area between the lens and retina.

What is the proper location for a hog snare?

Around the maxilla, over the snout The hog snare is used to control the movement of the pig. By controlling the maxilla and snout you maintain good control of the whole pig. It is similiar to the twitch in the horse in terms of behavioral control.

Which of the following should be a sterile procedure?

Arthrocentesis Arthrocentesis, or joint tap, should be performed with sterile technique. A joint tap is the act of inserting a needle into the joint space to withdraw synovial fluid for culture or analysis. The oral cavity is not sterile, and tooth extraction is not sterile. Lymph node and mass aspirates should be clean procedures but are not sterile. The same applies for culturing an abscess or site of infection.

What is commonly done to get a mare to cycle out of season?

Artificial lighting Most breeders prefer for a foal to be born early in the year. This is because all horses are considered 1 year old on January 1 no matter when they are born. Many mares are not cycling at the right time for this to occur, so artificial lighting is used to trick the mare that it is the breeding season. While keeping a stallion in the barn may be of benefit, horses are seasonally polyestrous, so they cycle when there are longer days, typically March-September. 16 hours of artificial light mimics this time period. Oxytocin is the hormone responsible for milk letdown. Palpation of the ovaries does not cause cycling.

In which species would you find the bursa of Fabricius?

Birds Birds have a bursa of Fabricius (BF), which is an outpocketing of the cloaca. This bursa is the site of hematopoiesis in young birds and is needed for B cell development. It involutes during sexual maturity.

What is the most common swine gastrointestinal parasite?

Ascaris suum Ascaris suum is the large roundworm of pigs. This parasite undergoes hepato-tracheal migration (the egg with L2 is ingested; the larvae undergo hepatic migration and molt to L3, then migrate to lungs where they are coughed up and swallowed. They molt two more times and become adult roundworms in the intestine). Stephanurus dentatus is the swine kidney worm. Erysipelothrix is a bacterium which causes diamond skin disease in pigs. Trichuris suis is the swine whipworm which lives in the cecum; it is also fairly common but not as common as the roundworm.

What is the biggest complication in patients with megaesophagus?

Aspiration pneumonia Megaesophagus patients are at a high risk of aspirating due their high incidence of regurgitation.

You are helping treat a dog with severe papillomas of the muzzle and oral cavity. This new therapy involves weekly intraperitoneal injections. Where is the safest place to do an intraperitoneal injection?

At the location of the umbilicus, 2 inches right or left of midline At the area of the umbilicus, 2 to 3 inches on either side is the safest place to give an injection into the abdominal cavity to prevent organ damage.

Which joint allows rotational movement of the head from side to side (from right to left or left to right)?

Atlantoaxial The atlantoaxial joint is a pivot joint which allows rotation of the head from side to side. It is located between the atlas and axis. The atlantooccipital joint provides the up and down motion of the head and is located between the occipital condyles of the skull and C1 (atlas). The coxofemoral joint is the hip joint. The scapulohumeral joint is where the scapula joins with the proximal humerus.

Which of the following disorders is caused by a Type I hypersensitivity where chemical mediators are released from mast cells after encountering an antigen?

Atopy There are 4 types of hypersensitivity: Type I is known as immediate hypersensitivity and results from inflammatory mediators being released immediately after exposure to an antigen. Common examples are atopy and anaphylaxis. Type II hypersensitivity is known as antibody-mediated hypersensitivity and results from antibodies directed against one's own cells. An example is autoimmune hemolytic anemia. Type III hypersensitivity is known as immune complex disease and occurs when antibodies and antigens form complexes that collect and cause problems. Glomerulonephritis can be caused by type III hypersensitivity. Type IV hypersensitivity is a delayed hypersensitivity that results from T-lymphocytes reacting against antigens. Examples include certain contact allergies (such as poison ivy) or from plastic food dishes (in some dogs).

Pemphigus is a disease affecting the skin and mucous membranes. Pemphigus is which type of disease?

Autoimmune Pemphigus is an autoimmune condition that affects the skin and mucous membranes. Antibodies made by an animal's immune system attack the bridges that hold skin cells together. Antibodies are deposited in the intercellular spaces causing the cells to detach from each other (also called acanthosis). Treatment for this condition involves the use of immunosuppressants such as corticosteroids. In some cases it may be caused by a drug allergy or reaction, but is still an autoimmune condition.

A 5-year-old domestic short hair cat presents with some lesions on his chin. The doctor diagnoses feline acne and prescribes some antibiotics and ointment. Which of the following is a good suggestion a technician could make to the owner?

Avoid plastic food and water dishes In some cats, a hypersensitivity to plastic can cause feline acne. This is a good suggestion to make to the owner of any cat with this condition. You cannot make recommendations for medications other than what the doctor prescribed during the visit. Popping the pimples can further irritate the skin and should not be performed by the owner. Taking the cat outside will not likely help this condition.

What is the term that describes an animal with elevated blood urea nitrogen and creatinine levels measured in the blood?

Azotemic Azotemia is the term meaning an animal has elevated blood urea nitrogen and creatinine. Uremia indicates a clinical syndrome seen in animals with severe kidney disease where they develop nausea, inappetence and potentially other signs such as oral ulcers. Hyperemia refers to red-colored mucous membranes. Nephrotic syndrome refers to a series of changes found in animals with ongoing or severe proteinuria.

Uric acid is the major end product of nitrogen metabolism in which of the following animals?

Birds Birds metabolize most of their nitrogen to uric acid, whereas mammals metabolize most of their nitrogen to urea.

Two black mice present for progressive, partial alopecia that started at the face and head and progressed to the trunk. The underlying skin is normal. The other mouse that the two are housed with appears normal. What is the most likely cause of the alopecia?

Barbering or chewing of the hair by their cage mate The correct answer is barbering or chewing of the hair by their cage mate. The key to this question is that the underlying skin is normal and the cage mate is normal. Barbering is usually a behavior exhibited by more dominant mice. Dermatophytosis or a mite infestation would cause crusting and scaling of the underlying skin, pruritis, and the cage mate would most likely be affected. Pattern alopecia would not typically start on the face and head and would also not likely affect two of the three mice. The hair would not appear chewed with pattern alopecia, but would show a more uniform or symmetrical loss.

What is the most common lead system used in a horse for electrocardiography (ECG) tracings?

Base-apex lead The most common is the 3-lead base-apex system in which the electrodes are placed at the left axillary region, right jugular region and a ground on the neck or withers region.

Approximately how much urine should a 40-pound dog produce in a 24-hour period if the dog is drinking and urinating normal amounts and is not dehydrated?

Between 450 and 850 mL of urine Normal urine output for a dog is 1-2 mL per kg per hour. This dog is 40 pounds, about 18 kg (40 pounds/2.2 = 18.1 kg) 18 kg X 1 = 18 mL/hr X 24 hours = 432 mL 18 kg X 2 = 36 mL/hr X 24 hours = 864 mL Normal urine output for this dog is between 432 mL and 864 mL in a 24-hour period. 1.5 Liters is equal to 1500 mL (there are 1,000 mL per Liter) 2.2 Liters is equal to 2200 mL

Which of the following is NOT true regarding platelets?

Birds and reptiles have nucleated platelets Birds and reptiles have nucleated red blood cells, not nucleated platelets. A purple top is used for platelets counts. Clumping of platelets can cause a low reading, so the periphery of the slide should be checked for clumping. In cases where clumping is present, a platelet estimate may be more valuable. The megakaryocyte is a bone marrow cell responsible for the production of blood thrombocytes (platelets).

What is the primary defense method for a pig?

Biting Pigs bite as a method of aggression. They will scream when frightened, but bite if challenged.

What is a comedone?

Blackhead A comedone is another term for blackhead. Comedones are fairly common in dogs with seborrhea (excess production of sebum), endocrinopathies, and demodex.

Snakes have the same organs as mammals except they lack which of the following?

Bladder Snakes have a cloaca which the ureters empty into, so they do not have a urinary bladder.

Which of the following is the best method for killing Parvovirus?

Bleach A 1:30 (1 parts bleach to 30 parts water) will destroy parvovirus.

The menace test will elicit which response? Which two nerves is this testing?

Blink, Cranial nerves II and VII The menace is when you wave your hand to the eye to try and elicit a blinking response. This is testing vision via the Optic nerve (Cranial nerve II). I-Olfactory nerve (smell) II-Optic nerve III-Oculomotor nerve IV-Trochlear nerve V-Trigeminal nerve (sensory to the face) VI-Abducens nerve VII-Facial nerve VIII-Vestibulocochlear nerve IX-Glossopharyngeal nerve (sensory for gag reflex) X-Vagus nerve (motor for gag reflex) XI-Accessory nerve XII-Hypoglossal nerve

An owner of a large breed dog calls the clinic and said she just came home from work to find the pantry open and a large portion of the dog food gone. The dog is salivating, dry heaving, and has a distended abdomen. What is a common condition in large breed dogs that can cause these symptoms?

Bloat A dog with a history of the listed symptoms should be seen immediately by a veterinarian. Large breed dogs are at a higher risk for "bloat" which occurs when the stomach becomes distended with gas, fluid, and/or food. The stomach can actually rotate upon itself in some cases and cause a life-threatening emergency called a Gastric Dilatation Volvulus (GDV). Straining to urinate, dribbling, or difficulty urinating could be symptoms of a urinary obstruction. Pyometra occurs in unspayed females and is when the uterus fills with pus. Common symptoms of clinical heartworm disease would include exercise intolerance and coughing. Allergic reactions most typically cause a swollen face or muzzle, hives, and sometimes vomiting.

What is a good way to assess daily changes in hydration status of a patient that is in the hospital for several days or longer?

Body weight Body weight is a good way to assess hydration of a patient. It is especially helpful when monitoring hydration status of hospitalized patients that are weighed daily. Tacky or dry mucus membranes and increased skin tenting are clinical signs of dehydration. Heart rate is often increased when patients are dehydrated, but increased heart rate can also be from other causes, such as stress.

Which vaccination is most commonly given the intranasal route?

Bordetella Bordetella bronchiseptica is a common cause of kennel cough. An intranasal vaccine against Bordetella is widely used. There is also an injectable vaccine available for Bordetella.

For which of the following diseases should an animal be isolated?

Bordetella bronchiseptica Bordetella is one cause of kennel cough and is contagious to other dogs. The other diseases listed are not contagious. Diabetes is not a contagious condition as is a result of an endocrinopathy. Demodectic manage is secondary to a non-contagious mite. Ehrlichia canis is a tick-borne disease and is not directly transmissible.

Ticks can serve as vectors to transmit several diseases in animals including Lyme disease. What is the agent that causes Lyme disease?

Borrelia burgdorferi Borrelia burgdorferi is the name of the rickettsial organism that causes Lyme disease. Ixodes is a genus of tick that may transmit the disease. Rickettsia rickettsii is the causative agent of Rocky Mountain spotted fever. Neorickettsia helminthoeca is the causative agent of salmon poisoning.

When dogs are mating, the male dog "ties" with the female so that the penis cannot be removed from the vagina for a period of time. This is due to the swelling of which structure?

Bulbis glandis The bulbis glandis is erectile tissue on the penis that swells during mating and locks or ties the penis in the vagina.

Which cranial nerve is known for three major branches?

CN V Cranial nerve V (five) is the Trigeminal nerve and has three major branches: First branch: Ophthalmic nerve- sensory to the eye socket and its contents, the anterior nasal cavity, and the skin of the nose and forehead. Second branch: Maxillary nerve- sensory to maxilla, nasal cavity/sinuses, palate, and part of the face. Third branch: Mandibular nerve (the largest branch)- sensory fibers to the lower jaw, the floor of the mouth, the anterior two-thirds of the tongue, the lower teeth, and supplies motor fibers to the muscles of mastication (note: that this is the only branch that supplies motor function).

For which very contagious and rapidly fatal disease are ferrets vaccinated against?

Canine distemper Ferrets are very susceptible to the canine distemper virus. It is very contagious between ferrets and thus has a very high morbidity and mortality rate.

Which of the following terms best describes the region at the corner of the eye where the upper and lower eyelids meet.

Canthus The "canthus" is the palpebral commissure (where the eyelids come together at the corners). There is a medial and a lateral canthus to an eye. Medial is towards the nose and lateral is towards the ear.

What is the best construction form for a tortoise outdoor enclosure?

Cement blocks Cement blocks mortared together will prevent falling as tortoise digs, and is sturdy enough for even the biggest tortoise. Chain link fence is fine if the bottom of the fence is covered. If a tortoise can see through the fence, it will try to escape. Pressure treated wood may have chemicals in the wood that is bad for the tortoise. Chicken wire is not strong enough to hold a large tortoise. The rocks could cause injury if they fell on the tortoise while it was digging, or if it tried to climb.

Which part of the brain is associated with regulation and coordination of movement, posture, and balance?

Cerebellum The cerebellum is involved in the coordination of voluntary motor, balance and equilibrium. It is located just above the brain stem.

What is the vertebral formula for dogs and cats?

Cervical 7, Thoracic 13, Lumbar 7, Sacral 3 The dog and cat have 7 cervical, 13 thoracic, 7 lumbar, and 3 sacral vertebrae as a general rule (in rare cases the dog may have 6 lumbar vertebrae).

Which of the following is another term for the type of parasite known as a tapeworm?

Cestode Tapeworms or cestodes are long, segmented, flat worms. Examples include the canine tapeworms, Taenia pisiformis and Dipylidium caninum. Trematodes are flukes, nematodes are roundworms, ascarids are a type of nematode (roundworm), and protozoans are single-celled organisms that may be parasitic.

A rabbit is brought in for skin problems. Upon exam, you notice large flakes of dead skin over the dorsum. Some of these flakes appear to move. What might you be seeing?

Cheyletiella Cheyletiella is also known as "walking dandruff". It is a 8 legged mite that can live on the skin of rabbits, dogs, cats and humans. They do not bury in the skin but live in the keratin layer. They have a 21 day life cycle and cannot live more than 10 days off the host.

Which of the following small mammals has the longest life expectancy?

Chinchilla The life expectancy of a chinchilla is 10 to 15 years. This exceeds the life expectancy of most other small mammals including rabbits (5 to 8 years depending on species), rats (3 years), guinea pigs (4 to 7 years) and hamsters (1.5 to 2 years). Other small mammals with relatively long life expectancies are ferrets (8 to 10 years) and sugar gliders (8 to 10 years).

Many factors can contribute to insulin resistance in pets. Which of the following can lead to insulin resistance and type II diabetes?

Chronic use of corticosteroids Type II diabetes occurs mostly due to insulin resistance and can occur from things such as chronic use of corticosteroids, obesity, or pancreatitis. Acinar cells secrete digestive enzymes. Beta cells of the pancreas actually make insulin, and degeneration of these cells can lead to type I diabetes. Increased drinking and urinating is a clinical symptom of diabetes but doesn't cause insulin resistance. A high-protein, low-carbohydrate diet actually helps with glycemic control.

Endoscopy can be a beneficial diagnostic tool for which of the following conditions?

Chronic vomiting Endoscopy is a scope that is used in the upper gastrointestinal tract. It is used to take biopsies of the stomach and small intestine. It is very useful to examine for underlying disease grossly (via the endoscope's camera) and microscopically via the biopsy specimens. Colonoscopy is used to examine the large intestine. Bronchoscopy is used in the airways and would be useful in a case of chronic coughing.

A rupture or tear in the thoracic duct would result in which of the following?

Chylothorax Chyle (a fluid high in chylomicrons and lymph) effuses from the thoracic duct into the pleural space if the thoracic duct tears or ruptures (usually trauma-related). The effusion typically appears as a milky white fluid and is odorless. This is only one cause of chylothorax. Other causes include lymphangiectasia, lung lobe torsion, heartworm disease, cardiomyopathy, neoplasia, idiopathic, and others. If the triglyceride level of the effusion is greater than that of peripheral blood, this can help to determine the effusion is chylous. The necropsy on this animal reveals a chylous effusion. Note the milky appearance of the fluid.

Which of the following is an appropriate anticoagulant for collection of blood from a donor horse for the purposes of blood transfusion to a recipient horse?

Citrate-phosphate-dextrose adenine (CPDA) CPDA is an appropriate anticoagulant to use for blood transfusions; the solution must prevent clotting of blood while providing a "fuel" source, such as dextrose, to the cells.

What is the single most important thing to do when removing a stomach tube from a patient?

Clamp or pinch off stomach tube prior to removing Clamping or pinching off a stomach tube just prior to removing the tube is essential to prevent spillage of the contents within the tube as it is being removed. Any inadvertent spillage could result in contents entering the airway, subsequently causing aspiration pneumonia. None of the other answer choices effectively prevent this potential complication.

A proliferative honeycomb beak (as seen in the image below) along with scaly crusty lesions on the legs and feet of a bird can be a sign of which of the following?

Cnemidocoptes pilae The correct answer is Cnemidocoptes pilae. Cnemidocoptes pilae, is also known as "scaly face" or "scaly leg." This mite can be treated with topical or oral ivermectin. It is more common in birds that are immunocompromised, and beak deformity may be permanent even after the mite is cleared. Avian pox can cause skin lesions, diptheric membranes, or septicemia. Mycobacterium avium causes granulomatous disease throughout the liver and GI tract. Chlamydophila psittaci is the causative agent of psittacosis, and causes lethargy, respiratory, and GI signs.

Which intestinal parasite is NOT considered zoonotic to humans?

Coccidia Coccidia are host-specific. Cystoisospora (the coccida found in dogs and cats) is not infective to humans. The other parasites listed are zoonotic.

Which of the following could cause a hamster to go into "pseudo-hibernation"?

Cold temperature Hamsters have the ability to "hibernate" in a sense. If the temperature drops too low (usually below 41 degrees F) they can go into pseudo-hibernation and owners can mistake them for dying. Low Vitamin C can lead to scurvy in hamsters. Scurvy symptoms include loss of fur, lethargy, squealing, weight loss, and weakness. Good air quality is important for overall health which is why hamsters should not be kept in aquarium type housing. Low calcium may lead to brittle bones and broken teeth.

What is the term given to the initial milk produced by a female after giving birth?

Colostrum The initial milk produced is known as colostrum because it contains a large amount of antibodies which provide the newborn with immediate immunity. Colostrum plays a very important role in providing immediate immunity to disease in certain species, depending on the type of placenta. For example, it is crucial for calves to receive colostrum, but this is not as important in dogs and cats. The amnion, allantois, and chorion are all fetal membranes around the embryo. The amnion is the closest layer to the fetus and forms a fluid filled sac in which waste material (amniotic fluid) is deposited. The middle layer is the allantois, and the outermost layer is the chorion. The chorion is attached to the endometrium.

Which of the following disorders is caused by a Type IV hypersensitivity, also known as delayed type hypersensitivity, as a result of a T-cell response to an antigen?

Contact allergy to plastic There are 4 types of hypersensitivity. Type I is known as immediate hypersensitivity and results from inflammatory mediators being released immediately after exposure to an antigen. Common examples are atopy and anaphylaxis. Type II hypersensitivity is known as antibody-mediated hypersensitivity and results from antibodies directed against one's own cells. An example is autoimmune hemolytic anemia. Type III hypersensitivity is known as immune complex disease and occurs when antibodies and antigens form complexes that collect and cause problems. Glomerulonephritis can be caused by type III hypersensitivity. Type IV hypersensitivity is a delayed hypersensitivity that results from T-lymphocytes reacting against antigens. Examples include certain contact allergies (such as poison ivy) or from plastic food dishes (in some dogs).

What is the most rostral structure of the eye?

Cornea The cornea is the most rostral structure of the eye and is where light first enters. The retina is at the back of the eye and is the location of the photoreceptors. The lens is responsible for focusing light onto the retina and is located caudal to the pupil and iris.

Through which structures does light pass as it moves from the outside to the inside of the eye?

Cornea, pupil, lens, retina The cornea is the clear coating that surrounds the eyeball itself, and light passes through the cornea first. The iris is the colored part of the eye which encompasses the pupil. The iris controls the amount of light coming in to the eye by dilating or constricting the pupil. After passing through the pupil, light reaches the lens, which focuses light onto the retina. The vitreous body is located behind the lens and in front of the retina.

When discussing the equine hoof, which structure produces cells that become the hoof wall?

Coronary band The hoof wall grows from the coronary band. The coronary corium produces cells that become the horn of the hoof wall. It is soft skin tissue such as the cuticles on our fingers. If the coronary band is ever damaged, the hoof may grow improperly and could cause lameness. The ergot is a callus-type structure on the underside of the fetlock. The sole is the bottom of the hoof. The frog is the structure that is shaped like a heart and extends forward across two-thirds of the sole. It grows from front to back where it then merges with the heel periople.

What is the term used to describe the junction between the skin and hoof of the horse's distal limb?

Coronet The transition is called the coronet or coronary band; the hoof is formed by epithelial keratinization over a greatly modified dermis.

Which of the following pairs of measurements are equivalent to each other?

Cubic centimeter and milliliter A cubic centimeter is the same volume as a milliliter. A liter of water weighs one kilogram, but these terms are not interchangeable because a liter is a volume and a kilogram is a weight. A liter of a heavier fluid would weigh more than a kilogram. A cubic foot is a volume, and a meter is a distance.

Which of the following hormones is produced by the adrenal glands?

Cortisol Cortisol is a steroid hormone, or glucocorticoid, produced by the adrenal glands. The other hormones listed here are all produced in the pituitary gland.

What type of placenta does a ruminant have?

Cotyledonary Ruminants (not including camels) have a cotyledonary placenta. This means they have numerous smaller placentae instead of a single large contact area between the mother and fetus. Ruminant placental cotyledons attach to a caruncle to form a placentome. The ruminant placenta is epitheliochorial (they have all three layers: endometrium, connective tissue, and uterine endothelium). The cotyledon is the fetus side of the placenta. The caruncle is the maternal side of the placenta. The placentome is the cotyledon and caruncle together. Dogs and cats have a zonary placenta. The placenta forms a band of tissue surrounding the fetus. They have an endotheliocorial placenta (only has the uterine endothelium). Primates and rodents have a discoid placenta (a single discoid shaped placenta). They have a hemochorial placenta (no maternal layers, but instead direct contact of maternal blood with the chorion). Horses and Pigs have a diffuse placenta. The placenta is formed by the whole allantochorion surface. They also have an epitheliochorial placenta like ruminants.

What is most often seen in a cat with heartworm disease?

Coughing Most cats with heartworm are not clinical for the disease. If they do show symptoms, they are typically respiratory in nature (coughing, wheezing). Chronic vomiting can also be a sign. All coughing cats should be tested for heartworm.

What is the best way to restrain a foal?

Cradle one hand under the neck and grasp the base of the tail with the other hand It is best to keep the foal with the mother. Foals are unpredictable and not trained to use a halter. Best control is achieved when cradling one hand under the neck and grasping the tail base with the other hand.

You are examining a blind dog and want to know if the dog has a sense of smell. You wave a treat in front of the dog's nose, and he smells it right away. Which nerve were you testing?

Cranial nerve I (one) The Olfactory nerve deals with the sense of smell and is Cranial nerve I. Cranial nerves: I-Olfactory nerve II-Optic nerve III-Oculomotor nerve IV-Trochlear nerve V-Trigeminal nerve VI-Abducens nerve VII-Facial nerve VIII-Vestibulocochlear nerve IX-Glossopharyngeal nerve X-Vagus nerve XI-Accessory nerve XII-Hypoglossal nerve

You have been working in the barn with baby calves that have been having severe diarrhea. You now are having a severe watery diarrhea. Which parasite would be most likely for you to have contracted from these calves?

Cryptosporidium Cryptosporidium parvum is a protozoan parasite that causes gastroenteritis in animals and humans. People with an adequate immune system have self-limiting disease, but immunocompromised people can develop life-threatening complications from this parasite. Some studies show that greater than 44% of calves with diarrhea can have this parasite isolated. Thus, this is a very important cause of calf scours.

What zoonotic pathogen might a technician contract from treating a 10 day old dairy calf with diarrhea?

Cryptosporidium parvum Cryptosporidium is a major cause of calf diarrhea, and just a few oocysts can be infective to humans and cause diarrhea lasting several days.

Which of the following is an ectoparasite of animals?

Ctenocephalides felis By definition, an ectoparasite is a eukaryotic organism that lives on the outside of the host. Examples include lice, flies, fleas, and mites. Ctenocephalides is a flea. Ancylostoma, Filaroides, and Paragonimus are all endoparasites. Feline infectious peritonitis is not a parasite, it has a viral cause.

Which of the following is the intermediate host of Dipylidium caninum and is important in its transmission?

Ctenocephalides felis Dipylidium caninum is the common canine tapeworm. The flea (Ctenocephalides felis) is the intermediate host. The parasite is usually acquired by dogs when they groom and ingest fleas carrying the tapeworm cysticercoid. This is important because the diagnosis of tapeworms in a dog may also be a sign that flea control is needed.

Ancylostoma causes which zoonotic disease in humans?

Cutaneous larva migrans Ancylostoma is a hookworm and is the most common cause of Cutaneous larva migrans. This is an intestinal parasite of dogs and cats. In this zoonotic disease, the parasite burrows under the skin in humans. Image shows hookworm eggs. Ocular and Visceral larva migrans are caused by Toxocara canis (roundworm). Crohn's disease is a type of severe inflammatory bowel disease in people with no correlation to our pet patients.

A 3-year old female spayed DSH cat presents for evaluation of a fistulous swelling of the ventrum. Physical exam is unremarkable, except for an area of matted hair over an erythematous 1 cm swelling on the ventrum that exudes a small amount of purulent material which you shave and examine. The cat has no history of fighting; what parasite can cause a lesion like this?

Cuterebra The lesion described is what is seen with Cuterebra infestations. Cuterebra is a botfly. The adults deposit eggs in the environment that animals may acquire as they pass through contaminated areas. The heat from the host causes eggs to hatch, and the larvae typically enter the body through the mouth during grooming or through open wounds. The larvae migrate to subcutaneous locations on the body and develop. They require a breathing pore in the skin and eventually will exit the skin. Lesions in dogs and cats are most commonly seen around the head, neck, and trunk. Suspect lesions should be explored, enlarged, and probed with forceps. The lesion should not be squeezed because rupturing the larva can cause anaphylaxis. It is preferable to remove the larva in one piece as residual pieces may cause recurrent infection or abscesses. Ctenocephalides felis is the cat flea, which can cause irritation but not the fistulated swelling described here. Demodex cati and Notoedres cati are mites and microsporum canis is a dermatophyte. These can all cause pruritus, dermatitis, and crusting but should not cause the fistulated swelling described here.

Which of the following is a term for a method of urine collection from a patient in a sterile manner?

Cystocentesis Cystocentesis involves the sterile introduction of a needle into the bladder for collection of urine. A free catch is the opportunistic collection of urine from a voiding patient. This urine is not sterile because it becomes contaminated as it exits the urethra. A cystotomy is an incision into the bladder. A cystogram is a radiographic contrast study of the bladder.

Which of the following is not typically associated with pruritus?

Demodex Demodex mites are typically not itchy. It is important to note however that with severe infections of demodex, secondary bacterial infections are common, and those can be itchy. Sarcoptes (scabies mites), Cheyletiella mites, Otodectes (ear mites), Malassezia (yeast), and Microsporum canis (ringworm) all cause pruritus.

Which breed is over-represented and prone to intervertebral disk disease?

Dachshunds Due to their anatomy, Dachshunds' long backs make them more prone to having a disk protrusion that can compress the spinal cord. This is why it is necessary to recommend to owners of Dachshunds (and other breeds with this type of conformation - corgis, bassets, etc.) that the dogs should not jump on and off furniture or beds; this is one measure that may help to prevent such an injury.

A diet high in concentrates or grains may lead to which of the following in cattle?

Decreased rumen pH Rumenal acidosis (decreased pH) may occur when feeding a high concentrate diet. This is due to microbes fermenting the starches in the grains too quickly such that the pH drops dramatically (less than a pH of 5.5). This increased acidity in the rumen can cause severe damage to the rumenal papillae.

Which of these is the term for the animal that harbors the mature, sexual stage of a parasite?

Definitive host The definitive host harbors the adult, sexual stage of a parasite. The intermediate host harbors larval, asexual, or immature forms of a parasite. A paratenic host (also known as a transport host) indicates a type of intermediate host on which a parasite does not undergo development into the next stage. A reservoir host is another vertebrate host for a parasite that serves as a source of infection for people or domestic animals.

What is the most common cause of urate bladder stones in a Sulcata tortoise?

Dehydration Dehydration leads to crystallization of urates and causes bladder stone formation. Dehydration and improper diet are the two most common causes of bladder stones in tortoises, but poor husbandry in general may contribute

A cigar-shaped mite that is seen on a skin scraping from a dog with severe generalized skin lesions is probably what type of mite?

Demodex Demodex is the cigar-shaped mite. The other mites are much rounder. The generalized skin lesions are not specific for Demodex as they can also be seen with Sarcoptes.

Which of the following mites that can be found on dogs is not contagious?

Demodex The correct answer is Demodex. Demodex is a normal inhabitant of the skin and causes disease only when there is either a genetic predisposition or systemic disease that allows Demodex to develop into disease. Therefore it is not contagious in dogs. Demodex gatoi in cats is thought to be contagious amongst cats.

These parasites are potential zoonotic pathogens EXCEPT for which of the following?

Demodex canis Toxoplasma can cause birth defects in humans. Cryptosporidium and Giardia can cause severe diarrhea. Toxocara canis can cause visceral larva migrans. Demodex is a mite that commonly lives in the hair follicles. It is not contagious to humans.

Cattle, or other livestock, exposed to prolonged rainy conditions are most likely to develop:

Dermatophilosis Dermatophilosis (also known as Rain Rot or Rain Scald) is caused by the bacterium Dermatophilus congolensis. It is a common skin infection seen in livestock during rainy seasons or in damp warm weather. Habronema is a gastrointestinal parasite seen in horses. Pediculosis is a lice infestation. Photosensitization would be more common with prolonged sun exposure.

Ringworm is the common name for which of the following types of organisms?

Dermatophytes Ringworm is the common name for the cutaneous fungi called dermatophytes. This group of fungi includes Microsporum canis, Microsporum gypseum, and Trichophyton mentagrophytes.

Which disease occurs when the body is unable to make or utilize insulin?

Diabetes Diabetes mellitus occurs when the pancreas does not make insulin or when there is insulin resistance such that the body is unable to utilize insulin.

Which of the following diseases would be most likely to cause the development of cataracts in dogs?

Diabetes mellitus Diabetes mellitus very commonly causes the development of cataracts in dogs. Cats do not typically get cataracts from diabetes. Toxocara canis (roundworm intestinal parasite) may cause ocular larval migrans in humans and thus is a zoonotic disease. It does not cause cataracts. Hypoadrenocorticism (Addison's disease) does not cause cataracts. Hypothyroidism could lead to elevated blood lipids and an increased chance of lipid corneal deposition but would not lead to cataract formation.

Giardiasis may be the cause of which clinical sign?

Diarrhea Giardia is a protozoan parasite that often causes a watery diarrhea. It may sometimes also cause vomiting. Seizures, urticaria (hives), coughing, and panting are not typical symptoms of giardiasis.

Where is the hypothalamus is located?

Diencephalon The diencephalon is composed of the thalamus, hypothalamus, and pituitary gland. The brain stem is the posterior part of the brain adjoining the brain and spinal cord. Most of the cranial nerves originate in the brain stem. The brain stem includes the medulla oblongata, pons, and the midbrain. The thymus is located in the mediastinum. The mesencephalon is the mid-brain (part of the brain stem).

Where is the hypothalamus is located?

Diencephalon The diencephalon is composed of the thalamus, hypothalamus, and pituitary gland. The brain stem is the posterior part of the brain adjoining the brain and spinal cord. Most of the cranial nerves originate in the brain stem. The brain stem includes the medulla oblongata, pons, and the midbrain. The thymus is located in the mediastinum. The mesencephalon is the mid-brain (part of the brain stem).

Occasionally, animals will present with dystocia. What does the term dystocia mean?

Difficult birth Dystocia means difficult birth or labor.

Chyme is a part of which bodily function?

Digestion Chyme is the term for partially digested food of the stomach that passes into the intestine.

Which of the following parasites can a dog acquire by ingesting an infected flea during grooming?

Dipylidium caninum The intermediate host of Dipylidium caninum is the flea; dogs are usually infected when they ingest a flea carrying the parasite while grooming.

Colic is a common condition in horses. Which of the following is not a common cause of colic?

Displaced abomasum Horses are not ruminants and therefore do not have an abomasum. The abomasum is the fourth stomach in ruminants and functions in a similar fashion to a monogastric stomach. Enteroliths are stones that can form in the gastrointestinal tract that result in an obstruction. Intestinal incarceration is the trapping of intestines through a small opening. Finally, an intestinal volvulus occurs when a section of intestine spins about its axis and occludes its own blood supply.

To obtain an accurate blood pressure in this patient, the doppler crystal should be placed:

Distal to the blood pressure cuff but proximal to the metatarsal pad To obtain an accurate blood pressure measurement, the crystal must always be placed distal to the cuff so that blood flow is properly occluded.

Which method is used to determine the sex of young mice?

Distance between the anus and genitalia Anogenital distance is the best way to determine whether juvenile mice are male or female.

A patient in the hospital has the orders "NPO" on its cage. What does this order mean?

Do not give the patient any food or water. The order NPO means that the patient should not be given anything to eat or drink. The abbreviation NPO stands for "nil per os", meaning nothing by mouth. The remaining answers are incorrect because the patient should not have any amount of food or water.

Which breed most commonly has von Willebrand's disease?

Doberman Pinscher Von Willebrand's disease is an inherited bleeding disorder most often seen in the Doberman. Other breeds can also have the disease, just not as commonly.

You are entering an exam room to obtain a history and perform a physical exam on a 2 year old German Shepherd. Which of the following body language signals would indicate that the dog is nervous or anxious?

Dog is standing but crouched down, ears pinned back, tail down, and the head lowered. A dog that is crouched down, with the ears pinned back, head lowered, and the tail down is displaying nervous body language. A dog with erect ears, tail down, and looking around the room is calm. A dog holding his ears erect and forward with the tail down is alert. A dog laying on his back with his tail tucked and ears flat is submissive. A dog standing at attention with hackles raised and growling is showing aggression and may be protective of the owner.

Which of the following correctly lists the species in order from shortest to longest gestation length?

Dog, Pig, Sheep, Horse, Llama This list of gestations should be committed to memory: Llama 1 year (350 days), Horse 11 months (330 days), Cow 9 months (280 days), Sheep/Goat 5 months (150 days), Pig 4 months (114 days), Dog/Cat 2 months (63 days), Ferret 1.5 months (42 days)

The Greyhound is an example of which of the following breeds?

Dolichocephalic The Greyhound has a long and narrow head. The nose and cranium are parallel and are divided at eye level, where the cranium descends to nose level. The Greyhound is a classic example of a dolichocephalic breed. Examples of a brachycephalic breed include a Pug, English bulldog, and Pekingese. In the brachycephalics, the face is very short and the cranium wide. The eyes are more widely spaced. Mesaticephalic breeds have a skull with the cranium and nasal cavity about equal lengths. The Labrador is a good example of this type. Brachygnathic is having an abnormally short lower jaw (mandibular) or an abnormally short upper jaw (maxillary).

To perform a bile acids test, what is required?

Draw a fasted blood sample, feed the animal, draw another sample 2 hours after feeding A typical bile acids test includes "pre-prandial" (before eating) and "post-prandial" (after eating) samples. A fasted sample is drawn, the pet is fed, and another sample is drawn 2 hours after feeding. A 12-hour fast is recommended for this test.

You accidentally write the wrong name for medication in the chart. What should you do?

Draw a single line through it and initial When a mistake is made in a medical record, a single line should be drawn through the mistake and initialed by the person making the mark.

A pet that is exhibiting the symptom of ptyalism has which clinical sign?

Drooling Ptyalism is drooling or hypersalivation. The term for a penis that is stuck protruding from the prepuce is paraphimosis.

Delivering puppies and kittens can be a rewarding experience and a procedure that the technician should always be ready to perform. What are the basic steps that need to be taken once a puppy or kitten is delivered?

Dry the newborn, massage gently, dry and suction any secretions from mouth and nose The correct handling of a newborn kitten or puppy involves drying the infant, massaging the body gently to help stimulate breathing, and removing any secretions from the mouth or nose. One of the most important concerns is maintaining a warm body temperature for the newborn. Bathing would result in hypothermia and is not recommended. An injection of vitamin B complex is not necessary, and the neonates are also too young to receive a rabies vaccination. Rabies vaccinations are typically not administered until the kittens or puppies are 4 months of age.

In a fetus, blood is shunted from the pulmonary artery to the aorta via what structure?

Ductus arteriosus The ductus arteriosus is the shunt connecting the pulmonary artery to the aortic arch. In this way, the blood bypasses the lungs, or pulmonary circulation in utero. Patent ductus arteriosus (PDA) is when this opening persists after birth and arterial blood is re-circulated through the lungs, which is abnormal. The foramen ovale allows blood to enter the left atrium from the right atrium. The ductus venosus shunts some blood flow from the umbilical vein to the vena cava. The foramen magnum is the large opening in the occipital bone at the base of the skull through which the spinal cord passes.

What are the three layers of the meninges?

Dura mater, arachnoid mater, pia mater The meninges are layers that cover the brain and spinal cord. The meninges consist of the dura mater, the arachnoid mater, and the pia mater. The meninges, along with the cerebrospinal fluid, function to protect the central nervous system (CNS).

Which of the following is required for chinchilla health?

Dust baths Chinchillas require dust baths every 1-2 days in order to maintain coat health. The dust bath should not be left in the enclosure at all times since frequent bathing may cause conjunctivitis. Chinchillas are heat-intolerant, and the enclosure should not be kept at temperatures above 80 degrees Fahrenheit. Cedar shavings should be avoided as bedding, since they can cause respiratory irritation. Chinchillas do not require supplemental Vitamin C. Healthy chinchillas do not require dental adjustments, although dental disease is a common problem.

A patient presents for difficulty swallowing and is found to have a bone lodged at the back of its throat during physical exam. Which of the following is the term for this presenting complaint?

Dysphagia The term for difficulty swallowing is dysphagia. It may be caused by an oral or esophageal foreign body, esophagitis, neurologic disease (myasthenia gravis), etc. Anuria is the term for absence of urination. Ptyalism is drooling and is sometimes observed in patients with dysphagia. Stomatitis is inflammation of the soft structures in the oral cavity (cheeks, gums, tongue, etc.).

What is the most sensitive and preferred test for detecting Giardia?

ELISA The most sensitive test for detecting Giardia is the ELISA. This will detect Giardia in greater than 95% of cases. The Fecal flotation with zinc centrifugation will only detect Giardia less than 70% of the time. A direct smear can be used to look for trophozoites but is not highly sensitive, and multiple smears may be needed to find the organism.

A client calls to schedule an appointment with the doctor because her dog has been shaking his head and now his ear flap feels swollen, warm to the touch, and is squishy. What should you book the appointment as?

Ear hematoma An ear hematoma occurs when the vessels inside the pinna rupture (from shaking the head, trauma, or vasculitis). The pinna (ear flap) fills with blood like a balloon. The term for this condition is an ear hematoma. The most common treatment includes placing a drain in the pinna that stays in place for a couple of weeks. Underlying ear disease should be treated as well.

The most common method of identification of pigs is what?

Ear notching Ear notching is used because it can be identified from a further distance than the other methods.

What is cerumen?

Ear wax Cerumen is the proper term for ear wax. It is the yellowish waxy hydrophobic protective substance that is secreted in the ear canal. Sebum is an oily substance that is produced by the sebaceous glands.

This instrument is most often used to examine which part of the body?

Ears The instrument pictured is an otoscope and is used to examine the ear canal. It is important to confirm that the tympanum (ear drum) is intact prior to using certain medications in the ears.

Coprophagy is the term used to describe an animal that is doing what?

Eating feces An animal that is eating its own feces or feces of other animals is said to be coprophagic. Coporophagy is a normal behavior of rabbits and many rodents.

From youngest to oldest, which is the correct order of the tick's life cycle?

Egg, larva, nymph, adult The exact time course and types of hosts can vary, but female ticks commonly lay thousands of eggs, usually on the ground. These hatch into larval ticks, which will feed on a host and then molt to the nymph stage. The nymphal tick then feeds off a second host and falls off when fully fed. It then becomes an adult that will feed, and the cycle repeats.

The doctor is going to spay a 6 pound Chihuahua and asks you to make sure the patient stays warm during and after the procedure. Which of the following methods is NOT appropriate to use as a heating source for anesthetized patients?

Electric heating pad Severe burns may result from electric heating pads even when using a barrier between the pad and patient. In addition, the electric heating pads are not recommended for conscious animals because of the danger of electrocution from the patient bithing the cord.

Where is the best place to give an intramuscular injection in a dog?

Epaxial muscle This is a large group of muscles between the spine and the wings of the ileum. They are easy to find and use. Semitendinosus muscles on the rear legs can be used, but there is a possibility of sciatic nerve damage if not done carefully. The sternohyoid muscle is a muscle of the neck and not appropriate to use for intramuscular injection. Muscles used for intramuscular injection need to be large, vascular, and easy to find and use. Not all muscles are appropriate.

The best locations to administer intramuscular injections, such as penicillin, to a horse include all of the following EXCEPT?

Epaxial muscles The epaxial muscles, which are at the dorsum of the horse surrounding the vertebrae, are not an acceptable location for an intramuscular injection due to their relatively small size and location near the spinal cord. All the other injection sites are acceptable.

What does the abbreviation "q" stand for on a prescription?

Every "q" means every. It may say something such as: Give 1 tablet q 12 hours for 14 days. This would mean give 1 tablet every 12 hours for 14 days. The abbreviation for orally is "P.O." (per os).

Where do sperm mature?

Epididymis The epididymis is the tube attached to the backside of the testes. It is found within the scrotum and is a coiled segment of the spermatic ducts in which spermatozoa mature. It connects the testicle with the vas deferens. Sperm matures within this tube. Oviducts, also called fallopian tubes, are found in the female reproductive tract. The egg passes from the ovary into the uterus via the oviduct. The prostate stores and secretes a portion of the seminal fluid.

Which of the following is the correct medical term for a bloody nose?

Epistaxis Epistaxis is Greek for "to bleed from the nose." Ptyalism is excessive drooling. Melena is the term for digested blood in the stool (black or tarry stools). Hemoptysis is coughing up blood.

Which of the following vaccines is not available for horses?

Equine Protozoal Myeloencephalitis Vaccine While equine protozoal myeloencephalitis is a common neurologic disease in horses caused by Sarcocystis neurona, there currently is no effective vaccine to prevent this disease. The other vaccines listed are all available.

Ruminants frequently belch or burp to release gas from the forestomach. What is the term for this process known as?

Eructation Eructation is the process of belching or burping.

Where would you find Anaplasma marginale?

Erythrocytes of cattle Anaplasma marginale is the most prevalent tick-borne pathogen in livestock. This is a rickettsial disease, and the organisms are found in the red blood cells of cattle.

When a calf is born, milk goes immediately to the omasum and bypasses the rumen and reticulum. Which anatomic structure makes this possible?

Esophageal groove The esophageal groove is a structure at the distal esophagus that forms a tube to allow milk to go directly into the abomasum. The purpose of this is to prevent the milk from being fermented by microorganisms found in the rumen. This structure is also sometimes called the reticular groove.

Which hormone causes the vaginal cells of the female dog to proliferate and form cornified epithelium?

Estrogen The correct answer is estrogen. As estrogen rises during proestrus, the vaginal epithelium proliferates, cornifies, and keratinizes, presumably in preparation for mating.

What is the most likely period of time during the estrous cycle that a female will be receptive to mating?

Estrus Estrus is the main period of sexual receptivity. At this point, the uterus and uterine horns are primed for receiving an embryo. Proestrus is the period just prior to estrus and is under the influence of a hormone call follicular stimulating hormone (FSH). During this period FSH stimulates release of estrogen. During diestrus the corpus luteum (ruptured follicle) secretes hormones. If pregnancy does not occur, the corpus luteum will degenerate and stop secreting hormones. Anestrus is the period in which there is sexual inactivity.

Ovulation occurs during which part of the estrous cycle?

Estrus Ovulation occurs when a mature egg is released from the ovary and is available to be fertilized. It occurs in estrus or just shortly thereafter.

Your neighbor just bought some pet mice and asks you about breeding them. He wants to know how long their estrus and gestation are. Which of the following is most accurate?

Estrus 4-6 days, gestation 19-21 days Mice come into estrus every 4-6 days and gestation average is around 20 days.

A dog has been brought into your clinic showing stage two signs of rabies. What are the animal's symptoms?

Excitability, viciousness, biting, and snapping Stage two of rabies is the furious stage. Animals will bite, snap, and attack for no reason, spreading the disease in their saliva though bite wounds. Stage one is the prodromal stage which is characterized by changes in attitude. The animal may become shy, nervous, and apprehensive. Stage three of the disease is the paralytic stage. Muscles will become uncoordinated and paralysis sets in. The animal will lose the ability to swallow and excessively salivate.

Aside from diabetes, what is another cause for glucosuria in a cat?

Excitement Excitement, stress, and fear increase blood glucose levels. If those levels exceed the renal threshold, glucosuria can occur. The other answer choices do not result in glucosuria.

Which of the following is a common cause of epistaxis in horses?

Exercise-induced pulmonary hemorrhage Exercise-induced pulmonary hemorrhage is a common cause of epistaxis (bloody nose) in the horse. The other choices do not commonly cause epistaxis.

Where is a meibomian gland located?

Eyelid Meibomian glands line the eyelids. They secrete sebum and help to prevent evaporation of the tear film. Meibomian growths are common in dogs. The dog in this photo has a meibomian gland tumor on his upper eyelid.

Harderian glands are associated with which part of the anatomy?

Eyes The Harderian gland is an accessory lacrimal gland on the inner side of the orbit in reptiles and birds; it is also present but is usually degenerate in mammals. Ears have ceruminous glands. There are perianal glands and apocrine glands associated with the anus of dogs and cats. The bladder is made up of transitional epithelium and is not glandular.

Syncope is a condition that one should be ready to report if observed. What is this?

Fainting A syncopal episode is an episode of fainting. A common cause of this in dogs is pulmonary hypertension or underlying cardiovascular disease.

Sheep mostly breed during which season?

Fall Sheep are polyestrous in the fall so they will give birth in the spring.

Oviducts are also known as which of the following?

Fallopian tubes Fallopian tubes and oviducts are the same structure. It is the structure that transports the egg from the ovary to the uterus.

The term pseudocyesis means which of the following?

False pregnancy Pseudocyesis is a false pregnancy. Dogs can become pseudopregnant after estrus when the female dog is not bred, or if it is bred by an infertile male. Pseudopregnant dogs can develop mammary glands, lactate, and have nesting behaviors. Spaying is the best way to prevent future episodes.

A patient who has been diagnosed with pancreatitis should avoid foods that are high in which of the following?

Fat Pancreatitis means inflammation of the pancreas, the glandular organ that secretes enzymes needed to digest food. When something causes these enzymes to be activated prematurely, they can actually begin to digest the pancreas itself, resulting in pain and inflammation. There is evidence that feeding a low protein, high fat diet can induce pancreatitis.

Feline Leukemia Virus

FeLV is shed in saliva and other bodily secretions (from the nose, urine, feces, and milk). Cats can transfer the virus via a bite, through grooming each other, and via sharing food and water dishes/litter boxes, etc. It is also transmitted from an infected mother cat to her kittens either in utero or via nursing. FIV is more commonly transferred via a bite wound and not from casual contact. FIP is caused by a coronavirus which is typically passed in feces, saliva or in utero. The coronavirus mutates in some cats and causes clinical FIP. The Toxoplasma parasite is typically acquired through eating raw meat/prey in cats that hunt.

Where is the easiest place to take a dog's pulse?

Femoral artery The easiest place to take the pulse is the femoral artery which is located in the medial thigh area just caudal to the inguinal area.

Which of the following species is an induced ovulator?

Ferret Induced ovulator means that the animal does not ovulate until the act of breeding occurs. Cats and Ferrets are the most common induced ovulators. Camels and Llamas are also induced ovulators.

Which procedure could be performed without using a surgical scrub, and an alcohol wipe would be sufficient?

Fine needle aspiration Surgical scrubs are needed for all other procedures listed here.

Flea allergy dermatitis is a very common cause of pruritus in dogs in endemic areas. This is usually an allergy to what?

Flea saliva Many pets are allergic to flea saliva and it causes a dermatologic hypersensitivity. It may manifest with symptoms of pruritus, erythematous skin, secondary pyodermas, and hot spots.

Lufenuron, Fipronil, and Imidacloprid are all used in treatment of which of the following?

Fleas Lufenuron (Program) controls flea populations by stopping the life cycle at the egg stage. It is an oral formulation. Fipronil acts to inhibit GABA and kills adult fleas. It is a topical and also works against some ticks. Imidacloprid is a topical flea adulticide and is a neurotransmitter blocker (Ach inhibitor). Selamectin (not listed) would act against fleas and sarcoptic mites.

You are combing through a pet's hair with your fingers and notice a large amount of dark debris which looks like dirt. This may implicate which parasite?

Fleas "Flea dirt" is organic debris which includes digested blood that is passed in the feces of adult fleas. Finding this debris suggests that the pet likely is infested with fleas.

The term myiasis indicates a problem with which of the following?

Flies Myiasis refers to infestation with larvae of flies (diptera). Pediculosis is the term for infestation with lice.

What is the name of the dye commonly applied to the eye to identify a corneal ulcer?

Fluorescein dye Fluorescein dye is the most commonly used dye to identify corneal ulcers; the dye adheres to exposed corneal stroma and stains it bright green. Rose bengel dye is occasionally used but much less frequently than fluorescein.

What term describes an enlarged thyroid gland?

Goiter Goiter is the abnormal gross enlargement of the thyroid gland. Thyroiditis is inflammation of the thyroid tissue. Thyroidosis is not a medical term. Hypothyroidism is inadequate production of thyroid hormone.

A client who feeds his dog "ad libitum" follows which schedule?

Free choice Ad libitum feeding means free choice feeding, or leaving the food out at all times for the animal to eat when he wants. SID = Once daily BID = Twice daily

Which of the following is not an acceptable means of euthanasia in reptiles?

Freezing The AVMA Panel on euthanasia has published several guidelines for reptilian euthanasia. Many methods remain controversial. It is the consensus that freezing is considered inhumane (see link below). "Immobilization of reptiles by cooling is considered inappropriate and inhumane even if combined with other physical or chemical methods of euthanasia."

When looking at the bottom of a horse's hoof, what is the name of the triangular shaped structure behind the sole of the foot?

Frog The name of the triangular structure is the frog (cuneus ungulae); this structure fills in the gap between the bulbs of the heels. The frog is a soft structure and is a normal part of the hoof anatomy.

A dog presents with lethargy and coughing. The doctor asks you to take a blood sample to test for Coccidioides immitis, also known as Valley Fever. This disease, most commonly seen in the southwestern United States, is caused by which of the following?

Fungus Coccidioides is the fungus that causes Valley Fever which can have many different symptoms. Symptoms most commonly include lameness, coughing, lethargy, and loss of appetite. The treatment of choice is fluconazole. Rickettsia is a genus of Gram-negative bacteria which are transmitted most commonly by a tick bite. Examples of rickettsial diseases include Ehrlichia canis (Tick Fever), Rocky Mountain Spotted Fever (Rickettsia rickettsii), and Salmon Disease (Neorickettsia helminthoeca). These are treated with doxycycline.

You are working with an aggressive chihuahua, and the standard muzzles available in the practice will not fit this dog. Which alternative could you try?

Gauze muzzle If it is difficult to place a regular muzzle on a dog or if the correct size is not available, a gauze muzzle can be used by tying the gauze around the muzzle and behind the ears. You should never grasp around the neck as described. Some breeds, such as brachycephalics, may have underlying airway disease and this would put them at risk for becoming cyanotic from excessive restraint. The same applies for the towel. It is too dangerous to try and attempt to hold an aggressive dog's muzzle with your hand given the risk of being bitten.

In the name Clostridium perfringens, Clostridium is the:

Genus Clostridium is the genus and perfringens is the species. The genus name is always capitalized.

If restrained in a stressful or frightening manner, which species might be prone to seizure?

Gerbil Gerbils have a high incidence of spontaneous seizures, which may be precipitated by being scared or stressed. To restrain a gerbil, it should be grasped by the tail base (not the tip, which could cause a degloving injury). The other hand then grasps the skin at the scruff of the neck. If possible, support the rear legs, as this can help to keep the gerbil calm.

Which breed is most at risk for developing Degenerative Myelopathy?

German Shepherd This is a progressive spinal cord disease that causes that myelin sheath around the nerves to degenerate. German Shepherds are over-represented for this condition. Dachshunds most commonly get intervertebral disk disease.

Which of the following areas should be avoided when giving intramuscular injections in cattle?

Gluteal muscles The gluteal region should be avoided because of increased chance of injection-induced abscesses at this site.

What equine gastrointestinal parasite, when deposited at an abnormal site on the skin or mucous membranes, is responsible for causing the eosinophilic granulomas known as summer sores?

Habronema spp. Habronema spp. and Draschia spp. are stomach worms of the horse. The larvae in feces are ingested by the maggots of flies, and the flies can deposit them at susceptible moist skin areas, damaged skin areas, or mucous membranes where the larvae cause an eosinophilic granuloma as a reaction to their migration.

Where would you find an arrector pili muscle?

Hair follicle Arrector pili muscles are tiny bands of muscle tissue that connect the hair follicles to the dermis. This muscle is what makes the hair stand up or the hackles go up on a dog.

What is most commonly used to restrain a horse for basic procedures?

Halter The halter is a "head collar" or headgear that fits behind the ears and around the muzzle. A lead rope is attached to the halter. It is a basic piece of equipment that is almost always used when handling an adult horse. Twitches are also used when greater restraint is needed.

What is the number one way to prevent transmission of disease in the large-animal hospital setting?

Hand hygiene Hand hygiene is the number one way to help prevent transmission of disease. Washing hands before and after patient contact, after using the bathroom, before and after eating, and when entering and leaving the hospital are all important times to wash. Environmental decontamination and/or disinfection are the other most important ways to prevent disease transmission. The other things listed are additional ways to help prevent the spread of disease in the hospital setting.

What is the number one cause of spreading nosocomial infections in a hospital setting?

Hands of hospital staff Nosocomial bacterial infections are infections that are acquired in the hospital. Consistently, the most important vector for spreading these infections is the hands of hospital workers. This is why hand washing and the use of exam gloves are so extremely important. Another term to know is "fomite"-- an inanimate object that can be contaminated and become a transfer agent for germs.

Rats are known to produce a substance called "red tears" or chromodacyorrhea. Where are red tears produced?

Harderian gland The Harderian gland produces the substance known as red tears. The red color is from a porphyrin pigment in the tears. Rats under stress may show this red tear production. Due to the connection between the tear ducts and the nasal cavity, the red stain may show up as a nasal discharge. Many owners will think it is blood.

Traumatic reticuloperitonitis or pericarditis in cattle is most commonly known as which of the following conditions?

Hardware disease Hardware disease is caused by ingestion of wire, nails, etc; these foreign bodies migrate into the reticulum and may pierce through and cause damage to internal organs (including the heart and diaphragm). It is most easily prevented via a magnet being placed into the reticulum to prevent these ingested metallic foreign bodies from migrating.

What is the meaning of the term multiparous?

Has more than one offspring at birth The definition of multiparous is having two or more offspring at birth.

Goats exhibiting aggressive behavior may do so most commonly in which manner?

Head butting Head butting in goats can be play behavior, but is primarily a form of aggression with other goats and possibly humans. Always watch behind you when you are in with a herd of goats, as they will often come up behind you!

Which clinical symptom can be an indicator of vestibular disease?

Head tilt Vestibular disease usually causes a head tilt (one ear lower than the other) sometimes with leaning and falling to that side. Other signs of vestibular disease include ataxia (in which the body may sway and the pet may have a wide-based stance) and abnormal nystagmus (eyes moving back and forth). Hypermetria is an exaggerated gait seen with cerebellar disease. Head bobbing or intention tremors are more consistent with cerebellar lesions. Decreased conscious proprioception can be seen with central/brain disease or from spinal cord disease of any kind and is not a specific finding. A behavior change such as aggression would not be expected in animals with vestibular disease.

Purkinje fibers are found in which of the following organs?

Heart Purkinje fibers are conductive fibers within ventricular walls. They relay cardiac impulses to ventricular cells, which allow contraction.

The urine in this collection set is demonstrating:

Hematuria The red tinge to the urine demonstrates that there are a high number of erythrocytes in the sample. You would need to check the urine specific gravity to know if there is isosthenuria. You would need to do a dipstick to check for ketones.

Stopping a blood vessel from bleeding is known as what?

Hemostasis Hemostasis is the arrest of blood flow from a bleeding vessel.

What does Eimeria stiedai cause?

Hepatic coccidiosis in rabbits Eimeria stiedai is the species of Eimeria that causes hepatic coccidiosis in rabbits. The rabbits ingest sporulated oocysts, which contain four sporozoites that hatch and travel via the hepatic portal vein to the liver and penetrate the bile duct epithelium. The schizonts in the bile duct cause bile duct hyperplasia; they block the bile duct, causing hepatomegaly, icterus, and liver failure.

An obese cat that is not eating for multiple days is at an increased risk for developing which condition?

Hepatic lipidosis Hepatic lipidosis is also known as fatty liver syndrome. Excess fats are broken down from the cat's fat stores and transported to the liver. The liver should then process this fat for utilization. In cats with hepatic lipidosis, this is impaired, resulting in accumulation of liver fat. The liver cells fill with fat and swell. This condition must be treated aggressively or can lead to death. The most important treatment for this condition is feeding. Most cats require placement of a feeding tube.

Which of the following fluid types is a colloid?

Hetastarch A colloid contains larger molecules than a crystalloid. This size difference allows the fluid to stay in the intravascular space for longer and thus helps to increase oncotic pressure. Examples of colloids are Hetastarch, blood products (whole blood, plasma, etc.), and Dextrans.

You see that a Shar Pei is scheduled to come in for a second opinion about his entropion. What is wrong with the pet?

His eyelids roll inward Entropion is when the eyelid edges roll inward towards the globe of the eye so that the lashes are rubbing against the cornea. It is a painful condition that often causes tearing and secondary infections, ulcers, etc. It is treated surgically. The proper term for itching is pruritus. Paraphimosis is the term for an extruded penis that cannot be retracted. A hernia is a hole or tear in the body wall from which abdominal contents can protrude subcutaneously.

A male ferret is also referred to as which of the following?

Hob A male ferret is known as a hob, while a female is called a jill. If you want to get really technical, these terms hob and jill are used for intact ferrets. A gib would be a neutered male and a sprite would be a spayed female (but often they are just called neutered hobs and spayed jills).

Which of the following methods is not recommended when picking up a snake?

Hold it by the tail Some snakes can lose their tails as a defense so should not be picked up by the tail. This will prevent skin loss or degloving injury. The same applies to lizards.

After which procedure is it most important that you immediately wash your hands as soon as the procedure is complete?

Holding a dog that may have Microsporum canis \ Microsporum canis is the most common cause of ringworm and is a zoonotic fungal disease. Fluoroscopy is a term that describes the acquisition of real-time x-rays. Demodex is a non-contagious mite. FIV is transmitted by close contact between cats. Of course, it is safe practice to always wash your hands before and after handling patients.

Which parasite is commonly passed from a mother to her young via the transmammary route?

Hookworms A common mode of hookworm transmission (Ancylostoma) is transmammary (nursing) through infected milk.

Which species is not susceptible to "hoof-and-mouth" disease?

Horses Hoof-and-mouth disease (often called foot-and-mouth disease) is a viral disease of cloven-hoofed animals including cattle, pigs, sheep, and goats.

Fermentation is a microbial process that uses enzymes to decompose food so that it may be utilized by the animal. In which species is this process most utilized in the hindgut during digestion?

Horses Horses are hindgut (large-intestinal) fermenters. They are herbivores, and the cellulose in the plants they consume is difficult to digest. They have a fermentation system which contains specific microflora, that break sdown the plant material, allowing them to access useful nutrients. Ruminants (sheep and cows) are also herbivores but are foregut fermenters as their fermentation process occurs in the rumen. Dogs are carnivores and monogastrics.

Which species is unable to vomit?

Horses Horses are unable to empty their stomach by vomiting. Passing a nasogastric tube is important in cases of colic to prevent the stomach from filling or rupturing. Placing a nasogastric tube: The tube is placed in the ventral meatus of the nasal passage and gently advanced through the nasopharynx. The neck should be flexed to help the horse swallow the tube and prevent advancement of the tube into the trachea. Sucking back on the tube and getting negative pressure helps to confirm that the tube is in the esophagus.

You are asked to prepare a patient for a bone marrow aspirate. Which of the following sites are commonly used in dogs to obtain a bone marrow sample?

Humeral head and iliac crest The most common collection sites in the dog are the humeral head, iliac crest, and femoral canal. In cats, the femoral head can be used. In horses, bone marrow can be obtained from the sternum.

What is the most common and life-threatening metabolic abnormality in a cat with a urethral obstruction?

Hyperkalemia A high potassium level is life-threatening. The majority of potassium is excreted in the urine. When the cat is unable to urinate, potassium builds up in the bloodstream and can cause serious metabolic consequences. Although the BUN and creatinine are often high, they are usually elevated from pre-renal (dehydration) causes and post-renal (obstruction) causes and will normalize after fluid therapy.

What clinical signs are associated with infection with Clostridium tetani in the horse?

Hypersensitivity to sound and touch characterized by signs such as muscle spasms (stiff gait) and protrusion of the third eyelid Clostridium tetani causes muscle spasms as a result of the organism producing toxins that block normal inhibitory neurons. Classic clinical signs include hypersensitivity to sound and touch resulting in muscle rigidity (lockjaw) and protrusion of the third eyelid.

Hyperthyroidism in cats can be associated with which condition?

Hypertrophic cardiomyopathy (HCM) Hypertrophic cardiomyopathy is a thickening of the heart muscle (myocardium) and is often associated with hyperthyroidism or hypertension.

Which of the following is known as Addison's disease?

Hypoadrenocorticism Hypoadrenocorticism is a decrease in circulating cortisol. Hyperadrenocorticism is an increase in circulating cortisol and is known as Cushing's disease.

A bird that is laying eggs with thin soft shells likely has:

Hypocalcemia Calcium is necessary to construct a rigid egg shell. If the laying bird is hypocalcemic then not enough calcium will be deposited into the shell, resulting in a soft egg. Giardia is a parasitic disease that does not result in soft eggs and usually affects the gastrointestinal tract. Anemia is a decrease in hematocrit below normal levels which may result in pallor and weakness. There are many possible causes for anemia. Pstittacosis is caused by a type of chlamydia bacteria which causes a combination of weakness, ocular discharge, respiratory illness and gastrointestinal disease.

Which electrolyte abnormality is known to cause ventroflexion of the neck and extreme weakness?

Hypokalemia Hypokalemia (low potassium) can result from various metabolic and endocrine disorders. It causes muscle weakness and can cause cervical ventroflexion. Hypokalemic myopathy has been linked with chronic kidney disease and excessive potassium loss (especially in cats). Hypokalemia can also result from osmotic diuresis, correction of acidosis, or insulin-mediated cell uptake that is seen in diabetic ketoacidosis therapy. Some medications can also contribute to hypokalemia, such as furosemide, which causes excess potassium loss in the urine. Other causes include inadequate potassium intake in the diet, gastrointestinal loss, and hereditary causes; also it is often seen in cases of hepatic lipidosis.

Which of the following is not a common cause of dystocia?

Hypokalemia Hypokalemia is not considered a cause of dystocia. Hypocalcemia, on the other hand, can result in decreased ability to contract and will be a cause for dystocia. Oxytocin release will lead to contraction and depletion of calcium stores; inadequate release of oxytocin may result in uterine inertia. An oversized fetus, as is common in small breed dogs, may necessitate a planned cesarean delivery. Animals that have a decreased pelvic diameter will also be at risk of developing dystocia. A common cause of decreased pelvic diameter is previous trauma.

What is the causative agent of what is commonly known as "Ich" in fish?

Ichthyophthirius multifiliis The correct answer is Ichthyophthirius multifiliis, a protozoan ectoparasite. The other answers are simply tricky distractors.

What causes Metabolic Bone Disease (MBD) in tortoises?

Improper calcium/phosphorus ratio If the dietary calcium is too low, it will cause malformation and softening of bones with deformed shells (called pyramiding). Typical tortoise diets are low in calcium and high in phosphorus, hence the need for most tortoises to receive a calcium supplement. Too much calcium or over-supplementation may cause zinc, copper, and iodine deficiency, and could lead to bladder stones. If the temperature is too low, it will hinder the ability to digest food properly. UVB light or natural sunlight is needed to absorb and metabolize calcium. There are other factors that contribute to MBD, but improper care and diet are the main factors.

This New Caledonian Giant Gecko (Rhacodactylus leachianus) is suffering from dysecdysis. What is the major cause of this abnormality in captivity?

Improper humidity Especially in climates with very low ambient humidity, reptiles kept in an improper humidity range will often have difficulties with normal ecdysis. Owners may need to provide a "humidity box" to give these animals an area with increased humidity to help with shedding. Excessive dietary phosphorus supplementation may sometimes cause excessive bone formation or renal mineralization, but rarely is a direct cause of dysecdysis. Improper UVB lighting may affect normal calcium metabolism but does not directly relate to normal shedding. Signs of hypervitaminosis A include epidermal necrosis and sloughing. Generally, there are other skin lesions present in addition to dysecdysis (if present).

Where would you find pia mater?

In the brain Pia mater, arachnoid, and dura mater are the three layers of the meninges of the brain. The meninges cover the brain and spinal cord and thus are found in the central nervous system.

You are reading a report from an oncologist and read that the popliteal lymph nodes were enlarged. Where are these nodes located?

In the caudal portion of the hindlimbs at the level of the stifle and back of the thigh The popliteal nodes are located on the back of the thigh just caudal to the stifle. The pre-scapular nodes are located at the chest over the pectoral muscles. The mandibular nodes are located under the jaw rostral to the salivary glands. The inguinal nodes are located inside each thigh at the inguinal region.

Where is the thymus located?

In the cranial mediastinum in the chest The thymus is an immune organ located in the mediastinum. It extends from the thoracic inlet to the pericardium upon which it is molded. The thyroid gland is located in the ventral neck region. The adrenal glands are located just above the kidneys. The hypothalamus is located in the brain.

In the typical life cycle of the flea (i.e. Ctenocephalides felis), where does the larva pupate and form into the adult flea?

In the environment (off of the host) In the flea life cycle, the adult female feeds, mates, and lays eggs on the host, but the eggs are smooth and usually fall off into the environment, often where the host typically lies down. After several days to weeks, the eggs hatch into larvae that feed on skin debris or other organic matter in the environment and then create a cocoon and pupate. The pupa then develops over 10 days or can remain in the environment considerably longer before emerging as an adult flea. This fact is important because this is why treating the host for fleas is not enough to eradicate a flea problem, as the environment is frequently contaminated and can re-infest the host after treatment.

What does the word anhidrosis mean?

Inability to sweat The cause is unknown, but some horses lack the ability to sweat. This condition in known as anhidrosis. Keep horses with anhidrosis as cool as possible during hot or humid weather. Use of mist, sponging, fans, shade or stall rest can be helpful during exposure to heat.

Which of the following is a classic symptom of pyometra?

Increased drinking and urinating Pyometra is an infection in which the uterus fills with pus. It is most commonly caused by E. coli. It is an emergency in which ovariohysterectomy must be performed. Clinical symptoms include decreased appetite, lethargy, fever, and polyuria/polydipsia.

What is meant by a "left shift" when describing a complete blood count?

Increased numbers of immature neutrophils in the blood A left shift refers to an increase in the number of immature neutrophils, usually bands, in the blood. This occurs when the bone marrow is increasing neutrophil production in response to inflammatory signals.

If the kidney has a drop in blood flow and a part of the kidney dies, what would this area be termed?

Infarcted An infarct is an area of necrosis due to a local lack of oxygen from due to a disturbance in blood flow to the area. In the kidneys, this is termed renal infarct.

Which of the following is most appropriate when placing a urinary catheter in a dog or cat?

Insert the catheter until urine flows, then advance 1 cm further and secure A sterile urinary catheter should always be used and coated in sterile lube. Sterile gloves should also be worn. Contaminants around the prepuce should be cleaned away with a gentle scrub. The catheter should be advanced until urine flows then 1 additional centimeter before securing in place. Placing the catheter until resistance is met can cause damage or irritation to the bladder wall, as well as cause bleeding, and can cause the catheter to loop back on itself.

Where are the anal sacs located?

Inside the anus at the 4 and 8 o'clock positions Anal sacs (also referred to as anal glands) are located just inside the anus at the 4 and 8 o'clock positions. Many dogs have difficulty expressing these sacs, which should normally occur during a bowel movement. Thus, manual expression is needed and is a procedure frequently provided by technicians. A symptom of full or irritated anal sacs is scooting or licking the rear end.

What is the primary reason a clinician collects an arterial blood sample (as compared to venous blood sample) when performing a blood gas analysis?

Interested in knowing the oxygenation of the patient There will be mild-to-moderate differences in CO2, HCO3, and other variables between an arterial and venous blood gas sample. However, one of the primary reasons to collect an arterial sample is to evaluate how well the patient is oxygenating (patient's partial pressure of oxygen in blood). Thus, one of the main indications to perform an arterial blood gas is to evaluate the respiratory system of the patient.

How is total parenteral nutrition (TPN) administered?

Intravenous Total parenteral nutrition (TPN) is feeding exclusively intravenously, bypassing the GI tract. It is very useful in cases when animals are unable to eat or will not eat for an extended time period. The other choices listed are all feeding tubes that enter the GI tract. Esophagostomy tubes go into the esophagus, PEG tubes go into the stomach, and jejunostomy tubes go into the jejunum (intestine).

Two technicians are inducing anesthesia on a cat for an ovariohysterectomy. The veterinarian is not present in the room. The cat becomes apneic. In what order should steps be taken to intervene? The cat should be:

Intubated, manually ventilated with 100% oxygen, veterinarian should be called It will take both technicians to properly intubate the patient. It would be suboptimal to wait for a veterinarian to intubate the patient. A patient in imminent danger of respiratory or cardiac arrest should be given life-saving care without delay. 100% oxygen should be used to increase the amount of oxygen available to tissues that have recently been deprived of oxygen. The veterinarian should be called immediately once an airway has been established and manual ventilation has commenced.

What three parts make up the uveal tract in the eye?

Iris, choroid, ciliary body The uvea is made up of the iris, choroid, and ciliary body. It is the middle layer of the eye. The uvea provides most of the blood supply to the retina. Uveitis is inflammation of this part of the eye.

Which nutrient must piglets be supplemented with?

Iron Sow's milk does not contain iron. This must be supplemented to prevent anemia in piglets.

Your dog's histopathology results have arrived and show an adenoma. What is true about an adenoma?

Is not malignant Benign tumors are not cancerous. There are many large benign tumors that are difficult to remove or are inoperable that can cause the patient serous morbidity. Just because a lesion is not cancerous does not mean that it will not be large or problematic for the pet.

What does polyvalent mean when talking about a vaccine?

It contains multiple antigens A polyvalent vaccine contains multiple antigens. The DHPPC-L vaccine is one example of a polyvalent vaccine and, as such, it contains antigens for distemper, hepatitis, parainfluenza, parvo, corona, and leptospirosis.

The dog penis differs from that of other animals in what way?

It has a non-articulating bone known as the os penis Dogs have a non-articulating bone known as the os penis. Cats have spiny epithelial projections on their penis that give them a "barbed" look. All animals have some degree of erectile tissue in their penis. The dog and stallion penis mostly consist of erectile tissue and very little connective tissue. This is in contrast to the ram, boar, and bull, all of which have more connective tissue and very little erectile tissue.

What is the cause of hip dysplasia?

It is a multifactorial disease Hip dysplasia is multifactorial. Many things such as genetics, environment, nutrition, and breed conformation play a role in the disease. Therefore, the treatment is often multi-modal. This may include an exercise plan, special diet, supplements for cartilage protection, anti-inflammatories, surgery, and/or other treatments.

Which of the following is true of the liver?

It makes and stores glucose Glucose comes from liver production and from the GI tract when food is absorbed. The liver helps to break down fats, converts glucose to glycogen, produces urea (the main substance of urine), makes some amino acids, filters harmful substances from the blood, stores vitamins and minerals (vitamins A, D, K and B12) and maintains a proper level of glucose in the blood. The liver also produces cholesterol, albumin, bile, and coagulation factors. Amylase comes from the pancreas and also from saliva. The main sources of ALP are the liver and bone, but smaller amounts can be found in the intestine and kidney. Liver cells can regenerate.

What is the name of the tick that is the primary vector for transmitting the bacteria which causes Lyme Disease?

Ixodes scapularis Lyme Disease is caused by Borrelia burgdorferi (a spirochete bacteria) and is transmitted by Ixodes (Deer Tick). It may cause fatigue, lameness, decreased appetite, and depression. The pet may not show symptoms until months after infection. Rhipicephalus (Brown Dog Tick) transmits Ehrlichia canis and prefers warmer climates. Dermacentor variabilis (American Dog Tick) and Dermacentor andersoni (Rocky Mountain Wood Tick) are known for transmitting Rickettsia rickettsii, which causes Rocky Mountain Spotted Fever. Amblyomma (Lone Star Tick) transmits Ehrlichia chaffeensis, which causes Human Monocytic Ehrlichiosis. It has also been implicated in other diseases.

Of the choices below, which tick is correctly matched with the corresponding disease transmitted?

Ixodes, Lyme disease The correct tick to rickettsial disease pairs: Rhiphacephalus (brown dog tick), Ehrlichia Ixodes (deer tick), Lyme disease (Borrelia burgdorferi) Dermacentor, Rocky Mountain Spotted Fever (Rickettsia rickettsii)

An 8-year old cat suffering from intestinal lymphoma has been prescribed TPN. Which of the following is necessary for the administration of TPN?

Jugular catheter TPN stands for total parenteral nutrition. TPN solutions contain 10% (or greater) dextrose and must be given in a central catheter to avoid phlebitis. TPN solutions cannot be administered through peripheral catheters. Medications should never be injected into A-lines.

When a blood sample is needed, the best site for venipuncture in the horse is which of the following?

Jugular vein The jugular vein is the best site for venipuncture in horses, especially for large blood samples. The cephalic vein and transverse facial veins can be used in debilitated horses for smaller samples of blood.

What is the most common location of an intravenous catheter in the horse?

Jugular vein The jugular vein is very accessible and carries a low risk of injury to the person placing the catheter when compared to trying to place a catheter on a limb. Jugular veins are large and easy to find along the neck of the horse as well as easy to maintain.

Which of the following is an appropriate thing to do for a horse that is diagnosed with tetanus?

Kept in a dark stable with extra padding Horses with tetanus are light sensitive and are sensitive to loud noises and handling. It is best to keep affected horses in a dark, quiet, padded stall.

Erythropoetin (EPO) is a glycoprotein hormone produced mainly by the:

Kidney EPO is produced mainly by peritubular fibroblasts of the renal cortex. It is a cytokine for red blood cell precursors in the bone marrow. This is partly why animals with chronic renal failure become anemic. The kidneys are no longer able to make EPO efficiently.

Where would you find the Loop of Henle?

Kidney The Loop of Henle is located in the kidney and is part of the nephron. The nephron is the functional unit of the kidney.

Which of the following animals would have an increased energy requirement?

Lactating Chihuahua Energy requirements are lower in geriatric healthy patients because they are typically less active and sometimes have a slower metabolic rate. Any animal that is pregnant or lactating will have a higher energy requirement.

A horse with extended forelimbs and arched and tense back that is painful when turning most likely has which of the following conditions?

Laminitis Laminitis, also known as founder, is a very painful condition in which the lamina in the hoof becomes inflamed. The horse tries to compensate for this pain and postures with an arched back and extended forelimbs in an attempt to take weight off the toes. It is especially painful for the horse when walking or turning on hard surfaces.

Where is the most recommended area to administer a subcutaneous injection in a horse?

Lateral aspect of the neck The lateral aspect of the neck is most commonly used due to the ease of drainage if a problem occurs secondary to the injection. Horses are known to commonly have reactions to injections. If an abscess forms, the area on neck is easiest to drain.

A dog is fractious, and you need to draw blood with the dog laying on its side. You cannot access the front end of the dog. Which vein would be the best for collecting a sample?

Lateral saphenous vein The lateral saphenous vein runs along the lateral side of the hindlimb and is most accessible below the stifle just above the tarsus. The other vessels listed are all located at the cranial end of the dog. Auricular is an ear vein; jugular is under the neck; cephalic is located in the forelimb.

You are at a ranch property and are asked by the vet to tie a horse to a fence. How should the rope be tied?

Leave 3 feet of slack and tie the rope level with the top of the horse's back Tie the rope level with the horse's back or withers (shoulder blade region) and leave approximately 3 feet of slack. Leaving more slack in the rope or tying the rope too low could increase the chances of the horse getting tangled up or injured.

Which vessel should be avoided when performing venipuncture in pigs?

Left jugular vein The phrenic nerve is located near the left exterior jugular vein, so the left side of the neck in general should be avoided in pigs. The right anterior vena cava is a commonly used site to draw blood from pigs. The auricular vein can be used to draw up to 5 mLs of blood. The tail vein and the orbital sinus near the medial canthus of the eye are other sites that can be used to draw small quantities of blood in the pig.

A cataract is an opaque protein deposit on which structure?

Lens A cataract is an opacification or clouding of the lens.

What structure in the eye is responsible for focus?

Lens The lens helps to refract light to be focused on the retina. By changing its shape, it changes the focal distance. The lens is a transparent biconvex disc located in the anterior chamber of the eye. The iris regulates the amount of light entering the eye by controlling the pupil size. The iris is the colored part of the eye. The retina contains photoreceptors (rods and cones) and is the location of the optic nerve. The sclera is a fibrous tunic and is the white part of the eye. The cornea is transparent and covers the front part of the eye. It also helps to refract light.

Which disease may be transmitted to humans through aerosolization or direct handling of urine from an infected patient?

Leptospirosis Leptospirosis is caused by a spirochete bacterium and is most often transmitted through direct contact with infected urine or infected fetus/vaginal discharge. It most commonly causes kidney and liver disease and is one of the most important zoonotic diseases of concern nationwide. It tends to be diagnosed most frequently in areas of high rainfall or where there are lakes or water sources, because the organism may survive in water. Rodents and wildlife are the most common reservoir hosts.

The term pediculosis indicates a problem with which of the following?

Lice Pediculosis refers to infestation with lice. It does not differentiate between biting and sucking lice. Myiasis is the term for infestation with fly larvae.

You are assisting with exams on 1-week old piglets. Which of the following may be used as a method of restraint?

Lift the piglet by its rear leg A piglet up to 10 kg may be lifted by a rear leg. Be careful not to swing the piglet by the leg; just lift it so that you may then support the chest with your other hand when moving it. When finished with the exam or procedure, lower the piglet back down so that both front legs have contacted the ground and gently lower the back legs to the ground and release your hold.

After parturition, it is normal for the bitch to have a reddish brown vaginal discharge for a few weeks. This discharge is referred to as which of the following?

Lochia Lochia is the term for the reddish brown discharge following birth. Melena is digested blood in the stool. Meconium is the dark intestinal excretion (first stool essentially) from a newborn. Pyuria is the presence of pus in the urine.

What is fluoroscein stain used for?

Look for a corneal ulcer Fluoroscein stain is a green/yellow dye placed in the eye to highlight a corneal ulcer or abrasion. It is best seen under a blue light (see image). A Schirmer Tear Test (STT) tests tear production. A distichia is an eyelash that arises from an abnormal spot on the eyelid and can cause a corneal ulcer if it irritates the surface of the eye.

Heart murmurs are often referred to in terms of "grade". What does this refer to?

Loudness Murmurs are graded according to the loudness/severity. They are typically graded on a scale of 1 through 6 (with 6 being the loudest). A description of the grades is: Grade 1: very soft murmur, only heard when carefully ausculted Grade 2: soft murmur but is evident Grade 3: moderate murmur but not associated with a palpable vibration/thrill Grade 4: loud murmur but a palpable vibration or thrill is not present or intermittent Grade 5: loud murmur that has a palpable thrill but is not heard when stethoscope is lifted away from the chest Grade 6: loud murmur with palpable thrill and can be heard even when the stethoscope is lifted off the chest

What type of diet should a cat or dog in renal failure should be fed?

Low in phosphorus and protein A low-protein diet helps to decrease workload on the kidneys. This is because protein (when metabolized by the liver) leads to production of the waste product urea. The kidneys excrete urea. If the kidneys are not functioning properly, then urea is not excreted efficiently and begins to build up in the peripheral blood and causes the pet to feel sick clinically. Phosphorus is removed from the body via the kidneys. If filtration is impaired, then the phosphorus begins to climb higher in the peripheral blood. Therefore, a diet low in phosphorus is ideal. A low-carbohydrate and high-protein diet would be more ideal for a diabetic patient and is not good for a patient in renal failure.

Which diet is the most appropriate for a cat in chronic renal failure?

Low protein To filter and eliminate the by-products of protein digestion (like creatinine), the kidneys must work harder. A diet lower in protein will decrease the workload on the kidneys as they process and eliminate waste products produced during protein metabolism. Many higher protein diets are also higher in phosphorus, which should be restricted in pets with kidney failure. Protein restriction is more important in pets that have proteinuria as a result of renal failure.

A dog presents with scabies. You are instructed to give the dog a lime-sulfur medicated dip. Which of the following must you do?

Lubricate the dog's eyes with protective ointment Dips can cause serious irritation and even damage eye tissue; protective ointment is mandatory.

Where would you collect cerebrospinal fluid (CSF) from a standing sedated horse?

Lumbosacral space There are two sites that are routinely used to collect CSF from horses: the lumbosacral space and the atlantooccipital space. In the standing horse, the lumbosacral space is the only space that can be used. CSF is collected from the atlantooccipital space only in the anesthetized horse.

Which hormone is the trigger for ovulation and development of the corpus luteum?

Luteinizing hormone (LH) Luteinizing hormone is produced by the anterior pituitary gland. The LH surge is the trigger for ovulation and development of the corpus luteum. Progesterone levels increase after ovulation has already occurred.

Borrelia burgdorferi is the cause of which disease?

Lyme disease Borrelia is a spirochete bacterium that causes Lyme disease. It is transmitted by the Ixodes tick (typically a deer tick). Ehrlichia is a rickettsial disease transmitted by the brown dog tick. Leptospirosis is also a spirochete bacteria transmitted by Leptospira. In veterinary medicine it most commonly transmitted through contact with urine from an infected animal. The causative agent for cat scratch disease is the bacteria Bartonella henselae.

What is the disease Bovine Spongiform Encephalopathy also known as?

Mad Cow Disease BSE is most commonly known as Mad Cow Disease and is a neurodegenerative prion disease in cattle. It causes a spongy degeneration in the brain and spinal cord and is fatal. Polioencephalomalacia is a central nervous system condition that is ultimately a result of thiamine deficiency. One cause may be by an enzyme that destroys thiamine. Black Leg in cattle is caused by Clostridium chauvoei and is an acute febrile disease causing emphysematous swelling in the heavy muscles. Hardware disease is a common term for bovine traumatic reticulopericarditis. It is caused when the cow swallows a sharp, heavy metallic object that falls to the floor of the rumen and is pushed forward into the reticulum, where it can penetrate into the pericardium.

The parathyroid gland is responsible for which of the following?

Maintaining blood calcium level The parathyroid gland is responsible for regulating and maintaining blood calcium levels. The four parathyroid glands are located on or near the thyroid gland. When parathyroid hormone is released into the blood, it circulates to act in a number of places to increase the amount of calcium in the blood (ie. removing calcium from bones). When the calcium level in the blood is too high, the cells of the parathyroids make less parathyroid hormone (or stop making it altogether), thereby allowing calcium levels to decrease. Catecholamines (epinephrine and norepinephrine) are released by the adrenal medulla. Insulin is made by the pancreas. Milk letdown and uterine contractions are stimulated by oxytocin, which is released by the pituitary gland.

What does a lysosome do?

Makes enzymes to break up cellular debris A lysosome is a spherical organelle that produces enzymes which break up cellular debris that has been taken up by the cell. Lysosomes digest excess or old organelles, food particles, and viruses or bacteria. Mitochondria are responsible for making energy. The Golgi apparatus packages proteins and lipids after they are synthesized.

What is the most common method of pregnancy diagnosis in cattle?

Manual rectal palpation Manually palpating for a pregnancy is the most common method in cows and can be done as early as 30 days. Ultrasound is becoming more popular for cows but is not the most common. Progesterone tests can detect presence of a corpus luteum, but sometimes a CL can be present while the cow is not pregnant. This is not a reliable method. The skin fold test is the test in cattle for tuberculosis.

Which animal is seasonally polyestrous and is not an induced ovulator?

Mare Mares are seasonally polyestrous, so they repeatedly cycle during the breeding season. This season occurs around the period when the days are longest (which is late March through September). The estrous cycle of the mare is about 21 days in length. During estrus, the mare is receptive to the stallion and lifts her tail and everts the clitoris, which is called "winking".

You are asked to place a nasal oxygen tube in a dog. What is the measurement for placement of nasal oxygen?

Measure to the medial canthus of the eye A topical anesthetic (such as proparacaine) is instilled into a nostril prior to catheter placement. The catheter is lubed and slid into the nostril and introduced into the ventral nasal meatus. It is advanced to the level of the carnassial tooth or the medial canthus of the eye and is sutured to the skin.

The cat in this picture is having an IV catheter placed into which vein?

Medial saphenous The medial saphenous vein is a great site for venipuncture, especially in fractious cats. The cat is placed in lateral recumbency for this procedure and is easiest to restrain in this manner.

The fetlock joint of a horse refers to what structure?

Metacarpophalangeal joint The fetlock in a horse is the common or lay term for the metacarpophalangeal joint. In the horse, the proximal phalanx (pastern bone) lies immediately distal to the third metacarpal bone (also known as the cannon bone), with which it articulates to form the condylar metacarpophalangeal joint (fetlock joint). The middle phalanx or second phalanx lies distal to the proximal phalanx, forming the proximal interphalangeal joint known as the pastern joint. The distal phalanx (coffin bone), is the most distal bone of the forelimb, and lies completely within the hoof capsule. The distal phalanx articulates with both the middle phalanx and the distal sesamoid, forming the distal interphalangeal joint known as the coffin joint.

Which type of gas is formed due to rumenal fermentation?

Methane Microbial fermentation of feed leads to production of methane gas. Carbohydrate digestion in the rumen (anaerobic methanogenesis) causes the highest amount of methane gas to be released.

What does MRSA stand for?

Methicillin-Resistant Staphylococcus aureus MRSA is a resistant bacterium that is becoming increasingly difficult to treat. Most infections are nosocomial and occur in open wounds, surgical or otherwise. This is why meticulous care and attention to cleanliness of the hospital, hand washing, and sterilization of medical equipment are so important.

Which of the following are treatments for hyperthyroidism?

Methimazole and radioactive iodine Hyperthyroidism (elevated thyroid level) in cats is treated with radioactive iodine or methimazole. I-131 and radioactive iodine are the same thing. Levothyroxine is used to treat hypothyroidism (thyroid level is too low).

An animal with inflammation of the uterine lining is said to have which of the following conditions?

Metritis Metritis is inflammation of the lining of the uterus (the endometrium). Pyometra is pus in the uterus.

Which of the following is a diagnostic stage of Dirofilaria immitis, the parasite that causes heartworm, in dogs?

Microfilarial stage The diagnostic stage is the stage of the parasite that is used to detect it with a diagnostic test. For heartworm, microfilaria can be identified on a concentrated blood sample.

Horner's syndrome can cause which of the following clinical symptoms?

Miosis Horner's syndrome can occur as a result of damage along the sympathetic trunk, especially as the trunk goes through the middle ear. Symptoms include miosis (constricted pupil), ptosis (drooping of the upper eyelid), enophthalmos, and third eyelid protrusion. At least three of these signs must be present to confirm the diagnosis. Weakness of the thoracic limb can also be present on the affected side.

"Scabies" in dogs generally refers to skin disease caused by which type of organism?

Mite Scabies refers to infestation with Sarcoptes scabiei var. canis, which is a skin mite that can cause severe pruritus and skin lesions. It is usually seen in dogs and rarely in cats. It may also cause less severe lesions in humans.

Which cell structure is responsible for producing ATP?

Mitochondria The mitochondria are responsible for producing the ATP necessary for all cellular functions. The endoplasmic reticulum acts as a transportation system for proteins to be moved as well as for protein synthesis. The Golgi complex modifies and packages compounds that came from the endoplasmic reticulum. They are then released into the cytoplasm for use within the cell. Lysosomes contain enzymes which digest intracellular bacteria and non-functional organelles.

Most distemper-parvo vaccinations are of which type?

Modified live virus Most distemper and parvo vaccinations are modified live virus vaccinations. Rabies is a killed virus vaccination in general. There is a feline-only rabies vaccination that is a canarypox vaccine. The injectable form of Bordetella vaccine is a cellular antigen extract vaccine.

You are asked to collect a urine sample for urinalysis on a patient while it is boarding for the week. What would be the best time to collect this sample?

Morning A first morning urine sample is always the best sample. This is because the first morning sample is likely the most concentrated. When the sample is better concentrated, it is easier to evaluate the urine sediment and to see how capable the kidneys are of concentrating urine.

What is the usual vector of western equine encephalitis?

Mosquito The correct answer is mosquitoes. Culex mosquitoes as well as others are the important vectors of western, eastern, and Venezuelan equine encephalitis.

What is the vector for transmission of heartworm disease?

Mosquito The heartworm, Dirofilaria immitis, is transmitted by infected mosquitoes (the mosquito is the intermediate host). The mosquito ingests microfilariae from the blood of an infected dog (an L1). Then the microfilariae mature in the mosquito to an infective state (L3). The mosquito punctures the skin, and the L3 enter into the new host where they begin migrating and maturing. The adults arrive in the heart about 110 days after infection. The females begin to produce the microfilariae about 6 months after infection, at which time the cycle can begin again via a mosquito bite.

How is West Nile Virus transmitted?

Mosquito bite The West Nile Virus is transmitted by mosquitoes which bite and infect birds. The virus amplifies in the birds as intermediate hosts. The birds are then bitten again by other mosquitoes that can now take the increased viral load; then they go on to infect other birds and many other animal species.

A client calls and plans to breed her prized Yorkie Emmy-Lou. She just finished her heat cycle, and the client would like to breed her the next time she comes into heat. She wants to know when this will likely occur. Which of the following statements is most accurate?

Most dogs have their heat cycle every 6 months Most dogs have their heat cycle every 6 months. Cats are induced ovulators.

You perform a vaginal cytology of a bitch in heat. Which of the following should you normally see?

Mostly cornified cells During heat, cytology will show 90-100% cornified epithelial cells.

Horses with laminitis should be fed which type of diet?

Mostly grass hay The cause of laminitis is multifactorial; however, a diet high in grains or grain overload has been identified as a potential cause. Also, alfalfa is higher in energy and protein, and consumption of this hay alone may attribute to laminitis in some horses. Therefore, horses with laminitis should be fed an average or mixed hay only and no grains.

Which of the following is a reason why a tick should not be removed without an effective removal device?

Mouthparts may be left embedded and create a focus for infection Incomplete tick removal can leave the mouthparts of the tick attached. This will maintain an exposed, dirty and open wound that is prone to infection.

A patient in ventricular fibrillation needs to be defibrillated, and you are assisting. Someone yells "clear" before the defibrillation. What do you do?

Move away from the patient and table Defibrillation delivers an electrical shock to the heart to try to re-establish a normal sinus rhythm. When this occurs, you should not touch either the patient or the table, or you could be shocked as well.

Johne's disease is an intestinal infection that can lead to clinical signs of diarrhea and weight loss. It affects cattle, sheep, goats, and other species. What is the Genus of the causative agent of Johne's disease?

Mycobacterium The correct answer is Mycobacterium. Mycobacterium avium subsp. paratuberculosis is the cause of Johne's disease and is known to affect cattle, goats, and sheep, as well as many exotic hoof-stock. Infection usually occurs when the animals are young, but clinical signs do not show up until the animal becomes stressed, often around 2 to 3 years of age. Clinical signs are essentially chronic intermittent diarrhea and weight loss. Listeria monocytogenes causes a disease of the central nervous system. Fusobacterium necrophorum is a bacteria often isolated from footrot in cattle. Brucella abortus is a bacterium which causes abortion in cattle (known as "Bang's disease") and also causes a disease in humans known as undulant fever.

Where might you find the parasite Oestrus ovis?

Nasal cavity of the sheep Oestrus ovis is the sheep nasal bot. Flies deposit their larvae around the nares. They enter the nasal cavity and migrate to the sinuses where they mature. They then leave the nose and pupate in the soil. The word "ovis" should have been a hint that the parasite is likely found in the sheep (ovine). Cuterebra is a bot that can be found under the skin, notably under a hole found on the skin in several different species. Hypoderma bovis is the warble fly whose larvae cause "warbles" or bumps under the skin in cattle.

You are asked to administer an intramuscular injection to a beef cow. Where should you administer it?

Neck muscles Because this is a beef cow, the injection should be given in the neck muscles to avoid creating a scar or damaging tissues in the muscles that may be used as meat for human consumption. When injections are given in inappropriate locations, the meat could potentially be condemned.

Dirofilaria immitis falls under which category of parasite?

Nematode Dirofilaria immitis is the heartworm and is a nematode (a type of roundworm). The mosquito is the intermediate host. Trematodes are flukes and require a snail as an intermediate host. Protozoans are single celled organisms (microscopic). They may take on a cyst form or may be in trophozoite form (such as Giardia). Coccidians (such as Cystoisospora) are a type of protozoan. Cestodes are tapeworms. Dipylidium, a type of tapeworm, requires a flea as an intermediate host.

What term would describe inflammation of the kidneys?

Nephritis Inflammatory conditions typically end in "-itis". Nephritis is inflammation of the kidneys. Hepatitis is inflammation of the liver. Cystitis is inflammation of the bladder. Blepharitis is inflammation of the eyelids/follicles. Gastritis is inflammation of the stomach.

What is the basic structural and functional unit of the kidney?

Nephron The nephron's job is to regulate the concentration of water and salts by filtering the blood, absorbing what is needed, and excreting the rest as urine. It eliminates waste from the body, regulates blood volume and blood pressure, controls electrolyte and metabolite levels, and regulates blood pH. The glomerulus is a cluster of vessels located within Bowman's capsule and is the main filter of the nephron. Hepatocytes are liver cells.

Feline toxoplasmosis most often presents with which type of symptoms?

Neurologic Most cats infected with Toxoplasma gondii will not show any symptoms. When disease does occur, it typically occurs when the cat's immune response is suppressed. Fever, loss of appetite, and lethargy can be symptoms. Most commonly, neurologic symptoms occur including seizures or circling. Uveitis can be seen. Toxoplasmosis is treated with Clindamycin.

Canine parvovirus causes which of the following laboratory changes?

Neutropenia Parvoviral enteritis causes severe vomiting and diarrhea, which leads to dehydration and often sepsis and fever. The sepsis (overwhelming infection) leads to a low neutrophil count.

What is the proper name for the "third eyelid"?

Nictitating membrane The nictitating membrane is the "third eyelid". The tympanic membrane is the "ear drum". A frenulum is a fold of tissue that prevents movement (such as with a persistent frenulum in the bull regarding the penis). Palpebra is the proper term in general for eyelid. A ranula is a mucocele usually found under the tongue region.

Viruses with this characteristic are significantly more resistant to environmental degradation; in other words, they survive longer in the environment.

Non-enveloped viruses The correct answer is non-enveloped viruses. Non-enveloped viruses are typically very resistant to environmental degradation and therefore survive longer and are harder to disinfect than enveloped viruses. For example, parvoviruses, which are non-enveloped viruses, are difficult to disinfect and survive longer compared to influenza viruses which are enveloped. Whether a virus is a DNA or RNA virus has no effect on its environmental durability.

What type of estrous cycle does a cow have?

Non-seasonal polyestrous Cows have non-seasonal polyestrous cycles. This means that they have estrus cycles year round. Average estrus is 21 days and lasts for 18 to 24 hours, but heat stress can shorten this window. Estrus is the time of "standing heat" when the cow will stand to be mounted by the bull. Sheep are polyestrous in the fall. Cats are induced ovulators.

An individual believes that no kill shelters are the correct moral standard for professional behavior. This is an example of which branch of ethics?

Normal ethics Normal ethics are defined as: An individual's attempt to discover what he or she believes to be the correct moral standard and norms for professional behavior.

A practitioner's view of convenience euthanasia is an example of which branch of ethics?

Normative ethics Normative ethics is defined as: An individual's attempt to discover what he or she believes to be the correct moral standard and norms for professional behavior.

An owner has fed her diabetic cat and has given the insulin injection. She calls the clinic because there is a wet spot on the fur where she gave the injection and she doesn't think it went in. What will you advise her to do?

Not re-dose the insulin, just give the next scheduled dose as directed The cat should not be re-dosed. It would be more detrimental for the cat to receive additional insulin than for him to miss an injection. Advise the owner to just give the next regularly scheduled dose. Karo syrup is given when hypoglycemia is suspected (if too much insulin is given or if the animal is having symptoms of hypoglycemia)

A dog presents for stumbling, has a head tilt, and you notice his eyes are moving back and forth rapidly in a horizontal motion. What is the term for this type of eye movement?

Nystagmus This type of eye movement is termed nystagmus. It is most commonly seen with brain lesions or with vestibular disease. Mydriasis is the term for dilated pupils; miosis is the term for constricted pupils. Ataxia is the term for an unsteady gait. Hypermetria is an exaggerated gait caused most often by cerebellar disease.

You are getting the weight of a patient in the exam room and are going to write it in the record. What part of a SOAP record would you put the weight in?

O O is the objective part of the record. It requires facts. All other sections, S, A, and P are subjective parts. They require interpretation and opinion. S- subjective O- objective A- assessment P- plan

Choose the proper corresponding acronym for the following in order: Right eye, left ear, both ears, both eyes, left eye.

OD, AS, AU, OU, OS Think O for ocular and A for auricular. OD-Right eye OS-Left eye OU-Both eyes AD-Right ear AS-Left ear AU-Both ears

A horse with flared nostrils and a heave line from the tuber coxae towards the elbow with forced expiration likely has which of the following conditions?

Obstructive airway disease These symptoms are typical of a horse with chronic increased respiratory effort. Coughing, weight loss, lack of energy, and exercise intolerance are also signs of this condition, termed "chronic obstructive pulmonary disease" (COPD). It is more common in older horses that are stabled during winter months. It is helpful to turn these horses out to pasture to help control the disease. COPD is similar to asthma in humans.

What does OSHA stand for?

Occupational Safety and Health Administration OSHA (Occupational Safety and Health Administration) is the main federal agency charged with the enforcement of safety and health legislation. They set standards for safety and practice, and veterinary services falls under their regulation.

Which of the following is NOT a common method of restraint in the horse?

Ocular pressure Ocular pressure will not do anything except aggravate the horse more. A nose chain involves placing a chain over the bridge of the nose and applying pressure; a lip chain places a chain to the mucous membranes of the rostral aspect of the mouth (after lifting the lip); a nose twitch involves placing a rope around the soft part of the horse's muzzle and twisting the rope (it is attached to a long wooden stick); and a skin twitch involves grasping a fold of skin (such as the loose skin of the neck) and twisting the skin.

The National Association of Veterinary Technicians in America adopting a code of ethics is an example of which branch of ethics?

Official ethics Official ethics is defined as: Values formally adopted by organizations composed of members of that profession.

In domestic animals, which of the following is true regarding inheritance of a trait that is coded by a recessive gene according to classical rules of inheritance?

Offspring may display a recessive trait even if neither parent displays the trait Domestic animals have paired chromosomes and therefore inherit 2 alleles of each gene, one from the mother and one from the father. For classical rules of inheritance, a single gene can be dominant or recessive. If an individual has 2 copies of a recessive gene, it will show the recessive trait. If an individual has 2 copies of a dominant gene, it will show the dominant trait. In the event that an individual has one copy of a dominant gene and one copy of a recessive gene, it will show the dominant trait (the recessive gene is "masked" by the dominant gene). Typically, the dominant gene is indicated by a capital letter and the recessive gene is indicated by a lower-case letter. For this question, let's consider the trait of congenital deafness (h) which is recessive versus normal hearing (H) which is dominant. In order for an individual to show the phenotype of congenital deafness, it must have an hh genotype. An individual that has normal hearing could have either an Hh genotype or an HH genotype. In the correct answer to this question, it states that offspring may display a recessive trait even if neither parent displays the trait. It is possible that both parents could have normal hearing and have a Hh genotype. If both parents contribute the h gene to the offspring, it will display the recessive trait of congenital deafness. In the other choices: Offspring may display a recessive trait even if only one parent carries a gene for the trait - If only one parent carries the h gene, the offspring must receive the H gene from the other parent and will have normal hearing. If both parents display the recessive trait, some offspring will show the trait but some will not - If both parents display the recessive trait, they both must have the hh genotype and will each pass on the h gene to offspring; therefore, all offspring should show the recessive trait. If one parent displays the recessive trait and one does not, up to 75% of the offspring should display the trait - If one parent has the trait (has hh genotype) and one parent does not (has either Hh or HH genotype), the offspring will have a 50% chance of inheriting the trait (if the other parent is Hh) or a 0% chance of inheriting the trait (if the other parent is HH). There is not a scenario where more than 50% of the offspring should inherit the trait.

In domestic animals, which of the following is true regarding inheritance of a trait that is coded by a dominant gene according to classical rules of inheritance?

Offspring may inherit the trait if one of the parents does not have the gene for the trait as long as the other one does have the trait Domestic animals have paired chromosomes and therefore inherit 2 alleles of each gene, one from the mother and one from the father. For classical rules of inheritance, a single gene can be dominant or recessive. If an individual has 2 copies of a recessive gene, it will show the recessive trait. If an individual has 2 copies of a dominant gene, it will show the dominant trait. In the event that an individual has one copy of a dominant gene and one copy of a recessive gene, it will show the dominant trait (the recessive gene is "masked" by the dominant gene). Typically a dominant gene is indicated by a capital letter and a recessive gene is indicated by a lower-case letter. For this question, let's consider an example of dwarfed limbs (D) which is dominant versus normal limbs (d), which is recessive. In order for an individual to show the phenotype of normal limbs, it must have a dd genotype. An individual that has dwarfed limbs could have either a Dd genotype or a DD genotype. In the correct answer to this question, it states that one of the parents does not have the gene for the dominant trait (has a dd genotype). It is still true that the offspring may inherit the trait because if the other parent has a gene for the trait, it may pass that gene to the offspring and because it is dominant, the trait will be expressed. In the other choices: Offspring may inherit the trait even if neither of the parents has the trait - This choice is not correct because if neither one of the parents has the trait, they both must carry two copies of the recessive allele (dd in the example) and do not have a copy of the dominant gene to pass on. All offspring will inherit the trait if both parents have the trait - If both parents have the trait, it is possible that they both have one dominant and one recessive gene (Dd). Therefore, it is possible that each parent could pass on the recessive allele to the offspring and the trait would not be seen. No offspring will inherit the trait if only one parent has the trait - If one parent has the trait, they can pass on the dominant allele for the trait, which will then be expressed and will be seen in the offspring.

You are restraining a horse with a halter while the veterinarian is examining a sore on the lateral side of the left pelvic limb. Where should you stand while holding this horse?

On the left side, hold lead rope with little slack When handling a horse, the holder and the person working on the horse should be standing on the same side. If the exam starts on the left, stand on the left; as the examiner moves to the right side, you should also move to the right side. There should be little slack in the lead rope to prevent the horse from moving.

What is the best way to restrain a dog for examination?

One arm under and around the neck and the other under the abdomen and in front of the flank The correct answer describes the best way to restrain a dog for physical examination. Securing the head in this manner helps prevent biting and restricts movement. The tail grip restraint is used for foals.

What is the medical term for declawing?

Onychectomy The procedure consist of surgically removing or amputating the third phalanx of each toe. The majority of practitioners who perform this controversial operation will only do the procedure on the front paws. Caudectomy is tail amputation. Orchiectomy is removal of the testicles. Ovariohysterectomy is surgical removal of the uterus and ovaries. This sterilization procedure prevents pregnancy and is called a spay. Enucleation is surgical removal of the eye.

What is the definitive host for the causative agent of equine protozoal myelitis/myeloencephalitis?

Opossum The correct answer is opossum. The opossum is the definitive host for Sarcocystis, the causative agent of equine protozoal myelitis.

Contagious ecthyma is a disease that causes painful oral ulcerations, pustules, and scabs. It is most often seen in sheep and goats, and is more commonly known as:

Orf Contagious ecthyma, most commonly called Orf is a highly contagious and zoonotic parapox virus. Blue tongue is a virus of ruminants and camelids causing fever, lung disease, and congestion of the lips and oral mucosa. Oral thrush is Candidiasis (yeast). Thrush is caused by an overgrowth of normal yeast flora seen mostly in immunocompromised animals. Foot-and-mouth disease is a contagious vesicular virus that affects cloven-hooved species. It causes vesicles and ulcers in the mouth, lameness of the feet, and is a reportable disease.

The movement of fluid from an area of lower solute concentration to an area of higher solute concentration across a semi-permeable membrane is called:

Osmosis In chemistry, osmosis is the movement of solute molecules through a semi-permeable membrane to an area of higher solute concentration. Net movement of solvent occurs from the less concentrated (hypotonic) to the more concentrated (hypertonic) solution to help equalize the concentration on either side of the membrane.

Which parasite is the most important cause of parasitic gastritis in cattle and is commonly known as the brown stomach worm?

Ostertagia ostertagi Ostertagia is the brown stomach/abomasal worm and is a major cause of parasitic gastritis in cattle. It is the most common roundworm in cattle. Strongylus vulgaris is a large intestinal worm in horses that can migrate to the anterior mesenteric artery. Ancylostoma braziliense is known as the Southern dog hookworm and is a primary cause of cutaneous larval migrans. Haemonchus is known as the barberpole worm or red stomach worm and is especially problematic in sheep.

The doctor is performing a physical exam on a horse and notices the horse has some ticks in the ears. The ticks have large spines on their bodies. He refers to them as "spinous ear ticks". What is the proper name for this parasite?

Otobius megnini Otobius megnini is the spinous ear tick. These ticks are most commonly seen in the Southwestern states. They may infest many different warm blooded animals (including dogs), but are most commonly seen in horses and livestock. These ticks may crawl deep into the ears and feed on blood. They can spread rickettsial disease and other infectious disease such as Q-fever and tularemia. Otodectes cynotis is the ear mite of dogs and cats. Oxyuris is the equine pinworm. Dermacentor albipictus is the winter tick typically found on horses, cattle, or deer, especially in the mid-Western regions (Nebraska, Colorado, and Wyoming)

Regarding sterilization in reptiles, which is true?

Ovaries should always be removed during sterilization When sterilizing a reptile, the ovaries should always be removed. Removal of only the oviducts puts the patient at risk for egg binding. With just the oviducts removed, the ovaries are still active and yolks can be released into the coelomic cavity. In ferrets, spaying is recommended due to the risk of bone marrow toxicity from the high levels of circulating estrogen during prolonged heat cycles.

What is of the biggest concern when shipping pigs?

Overheating Heat prostration is of greatest concern when moving and shipping porcine. The same is true when performing medical procedures on them. It is always best to handle swine in the early morning when the temperature is cooler. This is due to their large amount of body fat and lack of sweat glands. The same is true for sheep; they easily over-heat due to their thick coat.

The egg is fertilized in which of the following structures?

Oviduct Fertilization of the ovum occurs in the oviduct.

Orf is the cause of contagious ecthyma primarily in which species?

Ovine Ovine (sheep) and goats are the species which may contract Orf, also known as "soremouth". It is zoonotic and caused by a parapox virus. It is transmitted via direct contact with infected animals or by contact with fomites, so handlers should wear gloves when dealing with infected animals. It is a benign and self-limiting disease in most people but may be serious in immunocompromised people. Symptoms in sheep and goats include papules or pustules on the lips and muzzle that may then crust and bleed.

Which hormone is the main cause of milk letdown?

Oxytocin Oxytocin stimulates the milk letdown reflex. Progesterone and estrogen help with the growth of the mammary tissues that produce milk. Progesterone maintains a pregnancy. Testosterone is the principal male sex hormone.

What hormone stimulates uterine contraction?

Oxytocin is the hormone responsible for uterine contraction and is sometimes administered to animals with dystocia to help expel the fetus. Prolactin stimulates lactation, and oxytocin causes the milk letdown. Luteinizing hormone (LH) stimulates ovulation. Antidiuretic hormone results in water re-absorption when released.

Anal pruritus in the horse can be associated with an infestation of this parasite. Note the alopecia around the anal region and around the tail caused by rubbing.

Oxyuris equi The correct answer is Oxyuris equi. This is the equine pin worm, which will infest the perineal region and result in perineal irritation after eggs are laid. A frequent diagnostic test performed when visualizing damaged perineal regions is a scotch tape preparation.

What represents depolarization from the SA node through the atria on an electrocardiogram (ECG)?

P wave The P wave represents depolarization spreading from the SA node through the atria. The PR segment on the ECG represents the delay at the AV node and partial atrial repolarization. The QRS complex represents ventricular depolarization, and the ventricular rate can be calculated by the time interval between the QRS complexes. The ST segment comes after the QRS and is when the ventricle is depolarized. The ST segment can help to diagnose ischemia or hypoxia because it will become either depressed or elevated. The T wave represents ventricular repolarization.

What is another common name for herpes infection in psittacine birds?

Pacheco's disease Pacheco's disease is a highly fatal and contagious virus of psittacine birds. It is mostly transmitted through contaminated food, water, or feces. Less commonly, airborne transmission can occur. The most common sign is sudden death, as many birds do not show symptoms of disease. Pacheco's disease is generally considered untreatable because of its acute onset and rapid mortality. Antiviral drugs such as acyclovir may be of benefit to suppress the virus.

All of the following are examples of Regulated Medical Waste (RMW) that require special treatment or handling EXCEPT which of the choices listed?

Paint-stripping solvents that may release toxic fumes RMW, also known as biohazardous waste, applies to waste that is potentially infectious. It must meet both of the following conditions: 1) The waste must be generated as a result of the diagnosis, treatment, or immunization of animals 2) It must be either sharps or biohazardous waste The scalpel and syringe are examples of sharps, and the blood-soaked sponges and carcass are examples of biohazards. While reactive, solvents do require additional consideration when being disposed of, they are not Regulated Medical Waste as they do not meet the 2 criteria above.

Which organ is responsible for producing most of the digestive enzymes used to breakdown food?

Pancreas The pancreas produces enzymes such as trypsin, lipase, and amylase to breakdown food. These are packaged into zymogens, which are released and activated by trypsin.

Which organ(s) in the body is (are) responsible for producing insulin?

Pancreas The two main functions of the pancreas are endocrine or production of hormones (insulin and glucagon) and exocrine or production of digestive enzymes. The pancreatic beta cells are responsible for secreting insulin. The adrenal glands produce several hormones including aldosterone, cortisone, testosterone, and catecholamines. The pituitary gland produces ACTH, growth hormone, thyroid stimulating hormone, prolactin, and others.

What is feline "parvo" virus is known as?

Panleukopenia Feline panleukopenia is sometimes referred to as "feline parvo". It causes feline infectious enteritis. It can be associated with cerebellar disease in young kittens.

A dog presents dragging the hindlimbs. While the doctor is pinching along the dogs back on either side of the spine, the skin twitches, causing contraction of the cutaneous trunci muscle. Which reflex specifically is being tested?

Panniculus reflex The panniculus (cutaneous) reflex tests for superficial pain. A fold of skin is grasped with a small hemostat and pinched just lateral to midline on both sides. A twich of the skin (the panniculus reflex) is observed (contraction of the cutaneous trunci muscle). This helps to determine the caudal margins of normal superficial pain bilaterally. Proprioception is a test to determine if the pet knows where the limbs are in space, or position awareness. The paw is first flipped under so that the dorsal part of the foot is touching the floor, and then the paw is released. The pet should return the paw to the normal positioning. If the remaining paw flipped over, this indicates a proprioceptive deficit. Patellar reflex is performed by a tap on the patellar ligament in the knee. This should extend the stifle. Spinal reflexes are a general term and encompass multiple tests for sensory and motor components.

Which of the following is a causative agent of warts?

Papilloma virus Papilloma virus is responsible for causing warts. Dermatophilus is a bacterium that invades the skin typically in wet weather and causes crusting and infection of the skin (rain rot). Dermatophytosis is a fungus, most commonly ringworm (Trichophyton) in cattle. Orbivirus causes bluetongue disease in sheep, cattle, and goats.

Which internal parasite is a threat to young (but typically not adult) horses?

Parascaris equorum Ascarids (Parascaris equorum) can cause significant problems in young (<1 year) horses and can cause poor growth and a pot-bellied appearance. A more severe problem occurs when the small intestine becomes obstructed from a heavy worm burden. Adult horses develop an immunity to ascarids, so they are typically not a problem in adults.

A foal presents for colic caused by the organism shown in the image below (microscopic image from 40X, organism is approximately 100 um). What is this organism?

Parascaris equorum In foals, a significant ascarid burden with Parascaris equorum can lead to intestinal impaction and associated colic. This is a roundworm of horses.

Which of the following parasites can be associated with colic and respiratory disease in the foal?

Parascaris equorum Parascaris equorum, or the equine ascarid (roundworm), typically only infests foals and can cause ill thrift, coughing, pneumonia, and colic. If a foal has a heavy burden of roundworms and is subsequently given an anthelmintic, impaction colic can result from a mass of dead worms obstructing the intestinal tract. Foals develop immunity to this organism as they age and are typically free of infestation as adults.

What is the common term for Psittacosis (Ornithosis)?

Parrot fever Psittacosis is also known as Parrot fever or Chlamydiosis. The treatment of choice is doxycycline. It is a reportable disease, and any bird suspected of having Psittacosis should be immediately isolated. In birds, C. psittaci may manifest itself as an upper respiratory infection with nasal, and/or ocular discharge, diarrhea, or a combination of all three. In some cases, birds may be infected but show no signs. It is caused by the obligate intracellular bacterium Chlamydia psittaci. It is especially found in cockatiels and parakeets. After exposure, the incubation period can vary from 3 days to several weeks and can survive in the environment for several months. It is zoonotic, especially in immunocompromised people, and protection should be worn when handling a suspected case.

Where should an injection ideally be given to a pet bird?

Pectoral muscles For birds, the best place to give an injection is into the large pectoral muscle mass region. If drugs are injected in the lower half of the body, such as the legs, the medication can be absorbed towards the kidneys and could cause a problem. However, in some large birds, such as ostriches and emus, injections are sometimes given in the legs.

Which of the following lymph nodes are not palpated peripherally?

Perihilar Above the heart is a triangular depression named the hilum, near which the perihilar lymph nodes are located. They are inside the chest and therefore not palpated peripherally.

Which cat would be most prone to having a hairball problem?

Persian Persians have long hair and are thus most likely to have hairballs. However, all cats can get hairballs. This occurs if the cat swallows hair while it is grooming itself. Laxatone can help to keep the hairballs to a minimum and daily brushing also helps to reduce the amount of shedding.

The pinpoint red marks on this patient's gums can be described as:

Petechia Petechia are small (1-2mm)"pinpoint" red bruises under the skin, often associated with thrombocytopenia. Ecchymosis (also often called purpura) are very similar but are much larger areas of bruising, usually over 3mm. Papillomas are warts caused by a virus. Erythema is the general term for redness.

Which of the following methods is the best way to handle a tractable cat?

Petting the cat and using minimal restraint A "tractable" cat is easily handled and well behaved. Cats that are calm and easy to examine should have minimal restraint. An "intractable" cat or a "fractious" cat may need to be scruffed for exam to prevent scratching or biting. Cats that are high risk for biting should be muzzled and wrapped in a towel if necessary.

The lower respiratory tract includes all of the following except for which of these?

Pharynx The lower respiratory tract includes portion of the larynx that is past the vocal cords, the trachea, the bronchi, bronchioles, and alveoli. Alveoli are the sites of gas exchange between air and blood and are in the lower-most respiratory tract (lung). The nasal passages, sinuses, pharynx, and the portion of the larynx above the vocal cords are all part of the upper respiratory system.

What is the biggest risk when giving an animal fluids or medications via an orogastric or a nasogastric tube?

Placing tube into the lungs Sometimes animals require medications through a tube. Horses that are having colic symptoms often receive fluids and mineral oil through a nasogastric tube. Sometimes small animals need to receive activated charcoal or barium through an orogastric tube. The biggest risk when tubing a patient is accidentally placing the tube into the lungs, so the tube goes down the trachea instead of the esophagus. This leads to compromise of the lungs and can lead to subsequent pneumonia and can have fatal consequences.

Yersinia pestis is the causative agent for which zoonotic disease?

Plague Yersinia pestis is the causative agent of plague. Plague is usually transmitted by the bites of infected fleas. Yersinia pestis is a gram-negative bacterium and has a bipolar safety-pin appearance. Definitive diagnosis is based on culture. However, before collecting any samples, the state vet or CDC should be contacted. Plague is highly zoonotic, and bubonic, septicemic, pneumonic, and meningeal forms can occur. Early treatment is critical for survival. Antibiotic treatment early in the course of disease can greatly improve prognosis. Cat scratch fever is caused by the bacterium Bartonella henselae. Parrot fever is caused by the bacterium Chlamydophila psittaci. Q fever is caused by the bacterium Coxiella burnetii.

The bottom of a tortoise shell is called what?

Plastron The carapace is the upper shell and the plastron is the bottom shell. Scutes are a layer of keratin covering the shell. A bony bridge is an area that joins the carapace and the plastron together. A terrapin is a type of turtle which lives in fresh or brackish water.

What is the term used for increased urine output?

Polyuria Polyuria is the term that describes increased amount of urination. Pollakiuria refers to an increase in frequency in urination. Oliguria is a decrease in urination. Anuria is the complete absence of urine formation.

Which breed does not typically shed its haircoat?

Poodle Poodles do not typically shed hair. They may lose hair during chemotherapy treatments or with certain endocrine diseases.

What is cholestasis?

Poor flow of bile from the liver to the GI tract Cholestasis is a condition in which the bile has difficulty flowing from the liver into the duodenum. The prefix chole- means bile and the term stasis means to stand still. The enzymes associated with cholestasis are alkaline phosphatase (ALP) and gamma glutamyltransferase (GGT).

You are on a farm call with a veterinarian to investigate the cause of agalactia in the farmer's cows. These cows have:

Poor milk production Agalactia (agalactorrhea) is either an absence of milk production or abnormally low milk production following parturition. There are many common causes including infection (especially in sheep, goats, and pigs), poor nutrition or water deprivation, hormonal imbalances, anatomical causes(inverted teats or absence of mammary tissue, scarring of tissue), or failure of the neonate to suckle properly so that the milk let-down reflex is not stimulated.

Which of the following should never be given in a bolus to a patient?

Potassium chloride Rapid infusion of potassium can cause cardiac arrest. There is never a good reason to bolus potassium chloride; however sometimes it is inadvertently bolused when administering fluids which had potassium chloride added to them. Administration of greater than 0.5 meq/kg of potassium chloride can be deadly. All other choices as long as appropriately dosed may be bolused.

A 3-year old male neutered Maltese presents to your clinic because the owner saw "some white rice-looking things" around his anus. They were small and flat and seemed to be moving. Some of them were dried up. Which of the following drugs is effective against this organism?

Praziquantel (Droncit) This dog likely has a tapeworm infection. Tapeworm segments are typically flat and white and small, resembling a grain of rice. The most common tapeworms in cats are Taenia taeniaeformis and Dipylidium caninum. The only medication that will treat both types is praziquantel. Praziquantel is in the products Drontal Plus and Profender, approved for the use in cats. Drontal Plus also contains pyrantel. Profender also contains emodepside. Both are also effective against roundworm and hookworm. Pyrantel is not effective against tapeworms and treats hookworm and roundworm infection. Fenbendazole (or Panacur), treats Taenia but not Dipylidium, and also treats hookworm, roundworm, and whipworm infection. Revolution treats and prevents hookworm, roundworm, heartworm, fleas, and ear mites in cats. Frontline treats and prevents fleas and ticks. A flea control should be recommended, since Dipylidium is transmitted by ingestion of an infected flea. Taenia is transmitted through eating an infected prey.

Which of the following species require Vitamin C in their diets because they cannot synthesize it?

Primate Primates, including humans, are unable to synthesize adequate amounts of Vitamin C and must have it included in the daily diet. Guinea Pigs too have the same requirement for additional daily Vitamin C.

A bitch with a swollen vulva and bloody vulvar discharge that attracts but will not accept a male is likely in which stage of the estrous cycle?

Proestrus In proestrus, vulvar edema is present with a bloody discharge. The bitch is attractive to males but will not stand for mating. A bitch that is in estrus will typically accept a male for mating. During estrus, the vulva is not quite as swollen and the discharge becomes more clear.

Which of the following lists the correct order of the phases in the estrous cycle?

Proestrus, estrus, metestrus, diestrus, anestrus Proestrus is the time when ovarian follicles are starting to grow. It immediately precedes estrus. Estrus is the time of actual heat when follicles are mature and ovulation occurs or can be induced depending on species. During Metestrus, estrogen stimulation subsides and the corpus luteum starts to form. The uterine lining begins to secrete small amounts of progesterone. Diestrus is regression of the corpus luteum. Anestrus is resting of the sexual cycle.

Which hormone maintains pregnancy?

Progesterone Estrogen is made when the follicle stimulates GnRH from the hypothalamus and causes LH release from the anterior pituitary. LH causes the follicle to ovulate. The follicle then becomes the corpus luteum, and progesterone maintains the pregnancy. Prostaglandin actually causes luteal death.

What are the segments of a tapeworm that are sometimes seen in canine feces called?

Proglottids The progottids are the individual segments in a long chain that make up most of the tapeworm. When mature, each proglottid segment contains male and female reproductive organs; reproduction takes place, and eggs develop in each segment.

A dog is very weak in the hind-end but is able to stand with support. The dog is being supported under the abdomen, and the paws are individually turned over so that the dorsal paw is touching the ground. The dog is not able to correct this posture. This is an evaluation of:

Proprioception Proprioception is a test to determine if the pet knows where the limbs are in space, or position awareness. The paw is first flipped under so that the dorsal part of the foot is touching the floor, and then the paw is released. The pet should return the paw to the normal positioning. If the paw remains flipped over, this indicates a proprioceptive deficit. It is most often a result of spinal cord disease or spinal cord compression from disk disease, neoplasia, etc.

What is the proper term for when the globe (eye) comes out of the orbit due to trauma?

Proptosis A proptosed globe is an eye that has actually popped out of the orbit. Buphthalmos describes an enlarged and bulging globe which is usually caused by increased intraocular pressure (such as with glaucoma). Miosis is the term for constricted pupils and mydriasis is the term for dilated pupils.

The veterinarian asks you to retrieve a psittacine bird out of its cage for examination. Which of the following restraint tools would be least recommended for this type of bird?

Protective falconry gloves Thick protective gloves should never be used to restrain psittacine birds in the clinical setting because it is too difficult to feel the patient through the gloves and can lead to trauma. Towels can be helpful, and a bird should always be approached from the front with the towel. By letting the bird view the towel, it helps to alleviate stress and more easily allows handling than grabbing from behind. The most important thing with restraint in birds is to prevent restriction of the chest so that the bird can breathe properly.

Giardia is what type of parasite?

Protozoan Protozoa are unicellular organisms. Giardia exists in two forms: the motile trophozoite and the cyst form. Trematodes are flukes. Cestodes are tapeworms. Coccidia are a different type of protozoan parasite consisting of several species of Cystoisospora, Cryptosporidium, Toxoplasma, and others.

Which of the following is considered a reportable swine disease?

Pseudorabies Pseudorabies is a virus caused by porcine herpesvirus 1. Wild mammals, cattle, sheep, dogs, and raccoons are also susceptible, and the disease can be fatal to these species. It is shed in oral and nasal secretions of swine and spread through oral/nasal contact or by fomites. Symptoms in pigs include abortion, nasal discharge/sneezing, seizures, depression, circling, and increased salivation. In cattle, it causes intense itching, also known as "mad itch", then seizures and death. The other diseases listed are more common in pigs and are not considered as reportable.

Which of the following is a zoonotic disease?

Psittacosis Psittacosis, also known as Parrot fever, is caused by the bacterium Chlamydia psittaci. It is spread via bird droppings and nasal discharge. In humans, it causes cough, fatigue, fever, headache, and muscle aches. Some birds may be carriers of the bacteria and appear healthy. These carriers can intermittently shed the bacteria in their stool. Symptomatic birds may have droppings which look like pea soup, have decreased appetite, weight loss, ruffled feathers, conjunctivitis, or symptoms of upper respiratory infection. Pacheco's disease (known as Parrot Herpesvirus) is a fatal bird disease but is not contagious to humans. White muscle disease in livestock is caused by a selenium deficiency and is not contagious. Coccidioides immitis is a fungus which is not considered zoonotic from direct contact between animals and people, although humans may contract Coccidiodes indirectly. Any draining lesions thought to be caused by the organism should be handled as such.

A parrot presents because it is bleeding profusely from one of its feathers. What would likely be the best course of action to take after examination by the veterinarian?

Pull the feather out Blood feathers are immature feathers that still have a blood supply to the shaft. The shaft usually appears dark as opposed to white or clear as would be seen in a mature feather. If one is cut or breaks from trauma, it can bleed profusely and not clot. The bird can actually die of blood loss in some cases. Typically, the best course of action is to pull the blood feather out. This is typically done using a hemostat and giving a firm pull.

Where do adult heartworms live?

Pulmonary arteries Most adult heartworms reside in the pulmonary arteries.

A new client presents her puppy and is inquiring about crate training. You tell her that crate training can be used for all of the following except:

Punishment The puppy crate should be a place of comfort and a place the pet seeks to go. Punishing a puppy with the crate may result in a negative association with the crate.

What is the ideal diet to feed a healthy lactating (nursing) dog?

Puppy formula food Puppy food is higher in protein and is usually enriched with higher amounts of fats, vitamins, and minerals. A nursing mother needs about twice as much food than when she is not pregnant or nursing.

Newborn mice are often called which of the following?

Pups Newborn mice are also referred to as pups or sometimes pinkies. They are pink and hairless and blind and deaf when born. Newborn rabbits are called kits and are also blind, deaf, and hairless when born. Ferrets are also called kits. Guinea pigs are also referred to as pups and are "precocious" when born. This means their eyes are open and they are aware and can see and hear and walk from birth.

What is the term used for an elevated number of white blood cells in a urine sample?

Pyuria The correct answer is pyuria. Hematuria is the excessive presence of red blood cells in urine. Stranguria is difficulty or straining to urinate. Anuria is the lack of urine production.

Coxiella burnetti is the cause of what zoonotic disease in humans?

Q fever Coxiella is a Gram-negative bacterium. Cattle, sheep, and goats are the primary reservoirs. Inhalation from infected birth tissues or urine is the main method of transmission. Undulant fever, or Bang's disease, is caused by Brucella. Cat scratch disease is caused by Bartonella. Plague is Yersinia pestis, a Gram-negative rod. Be wary of cats presenting with enlarged or abscessed mandibular lymph nodes and fever. Contact the local and state health departments if you suspect a case of plague.

When performing a major cross-match when a blood transfusion is necessary, which of the following samples are mixed?

Recipient serum with donor erythrocytes In a major cross-match, the recipient's serum is combined with the donor erythrocytes, and the sample is examined for agglutination or hemolysis, indicating incompatibility. A minor cross-match combines recipient erythrocytes with donor serum.

Tularemia would most likely be carried by which animal?

Rabbits Tularemia is caused by the bacteria Francisella tularensis and is known as rabbit fever. It is zoonotic, and the bacteria can gain entrance through a cut or scratch when handling an infected animal. It leads to a skin ulcer and then swollen glands, fever, headache, and rash. It can also be transmitted to humans via tick bites or deerflies.

A dog presents deceased to your clinic with foam coming from the mouth and his neck appears swollen. The owner says the dog had been sick for a few days and then had a seizure and died. They live on the outskirts of town on some acreage, and the dogs roam free on the property. What disease would you be concerned about? (Hint: You need to wear protective clothing before handling this pet)

Rabies While you are not making a diagnosis, you should know that swollen neck area or enlargement of lymph nodes, foam from the mouth, and history of seizure and death may be symptoms of Rabies virus. While it is not very common, cases of Rabies are diagnosed every year in domestic pets. It is best to take precautions, get a history from the owner, and discuss with the veterinarian proper handling of the animal and if any testing will be required. Exposure of the owner or other people to the dog should be documented if Rabies is suspected; specimens will need to be sent to lab for testing.

Which vaccine can a pregnant dog receive?

Rabies Pregnant dogs may receive killed antigens such as the Rabies vaccine. The DHPP vaccine is modified live. The Bordetella intranasal is modified live, and the Bordetella injectable is cellular antigen extract and should not be given to pregnant animals.

Which of the following is NOT considered a leukocyte?

Red blood cells (RBCs) Leukocytes, or white blood cells, are responsible for many of the immune functions of the body and are classified as granulocytes (those that stain positive for granules) and agranulocytes. Red blood cells are not considered leukocytes.

Erythematous skin might also be described as:

Red skin Erythema is redness of the skin. It is usually caused by hyperemia (increased blood flow) to the affected area. Itchy skin is described as being pruritic. Lichenification is a descriptor for thickened skin.

Which of the following is a technician not allowed to do?

Refill medications without veterinarian approval A technician must have approval by a veterinarian before refilling medications. Technicians are allowed to dispense controlled substances that are prescribed by a veterinarian, taking care to count and log the medications appropriately. Technicians commonly give injections and perform dental cleanings.

What is a common presenting complaint in a patient with megaesophagus?

Regurgitation Megaesophagus is a condition in which there is severe dilation and often distension of the esophagus. Food may pool in the esophagus and be regurgitated (often appearing as an elongated sausage or bolus of food). Many clients do not realize the difference in regurgitation (passive process) and vomiting (active process with abdominal effort). Therefore, it is essential to thoroughly question the owner about the nature in which the animal is bringing up the food or vomitus.

What is the most common reason for placing a urinary catheter in a male cat?

Relieve urethral blockage Feline lower urinary tract disease (FLUTD) is a common problem in young male cats. The etiology is unknown but is thought to be related to stress and potentially diet. In this condition, the urethra becomes obstructed due to inflammation, or sometimes the formation of a plug of crystals and mucus. This is a life-threatening condition and requires immediate placement of a urinary catheter to relieve the obstruction.

A patient with which condition would be most likely to receive weekly subcutaneous fluid administration?

Renal failure Animals in renal failure are often given subcutaneous fluids on a routine basis. With chronic renal failure, cats commonly lose more fluids than usual through their kidneys as they are trying to excrete waste products from the body. The filtration system is impaired, and the toxins (such as urea, creatinine, and phosphorus) continue to build up. As the disease progresses they become chronically dehydrated and benefit from routine fluid administration.

Of the following options, oral ulceration is most common in patients with which condition?

Renal failure Severe azotemia causes generalized acidity in the body. This acidity makes animals in renal failure susceptible to development of ulcers in the gastrointestinal tract, including the mouth. This is an important reason why animals with azotemia are often on acid-blockers (such as famotidine).

Where is the sinoatrial node located?

Right atrium The sinoatrial node provides automaticity to the heart and is located on the right atrium. The impulse then travels to the atrioventricular node which then conducts the impulse down the Bundle of His to the Purkinje fibers. This pattern of depolarization results in a very coordinated contraction of the heart, allowing for smooth blood flow.

You are reading a cardiologist report discussing tricuspid valve insufficiency and grade 3 murmur. The mitral valve is said to be normal. If you listened to this patient, where would you expect to hear the murmur the loudest?

Right side The tricuspid valve is located between the right atrium and right ventricle. Mitral murmurs are usually left-sided holosystolic murmurs.

Which of the following conditions in rats results from living in an area with the humidity too low?

Ringtail Ringtail occurs primarily when rats are kept in an environment too low in humidity. It shows up as rings around the tail where the blood supply has been compromised. Correcting the humidity to 50% will prevent the problem, but will not reverse any damage that has already been caused. Red tears or chromodacryorrhea in rats is thought to be caused by stress. Chronic Respiratory Disease and Mycoplasma are infectious respiratory diseases.

What is the drug of choice for treating Tritrichomonas foetus in cats?

Ronidazole The only proven effective treatment for treating T. foetus is ronidazole. It is a nitroimidazole antimicrobial that has mutagenic properties and should be handled as would a chemotherapeutic agent. Ronidazole is not licensed for use in cats and is thus used off-label for this parasite; it should only be used with caution and with informed, signed, owner consent.

A wound in front of the eye in a dog (toward the front of the nose) may be described as being which of the following?

Rostral to the eye Rostral is a term that indicates a structure is towards the front of an animal; the term comes from the latin term rostrum indicating the beak or snout. In veterinary medicine, anterior and posterior are often used only to describe distal extremities but may sometimes be used to describe positions on the head. Cranial is toward the head; caudal is toward the tail. Ventral is toward the abdomen; dorsal is toward the backbone. Lateral is away from midline; medial is toward midline. Proximal is close to the spine or body while distal is away; these are typically used when describing limbs (e.g. fracture of the proximal femur vs. fracture of the distal femur). Palmar is the bottom of the paw of the forelimb; plantar is the bottom of the paw of the hind limb.

What is the causative agent of visceral larval migrans in humans?

Roundworm The correct answer is roundworm. The roundworm, Toxocara canis, is the most common cause of visceral larval migrans in humans. Transmission is usually fecal-oral. Puppies may acquire the disease transplacentally.

The correct order of the ruminant digestive system in regards to the chambers of their stomach is:

Rumen, reticulum, omasum, abomasum. This is the order of the "stomachs" in ruminants such as cattle. The abomasum is the true glandular stomach. The rumen is the largest chamber. The reticulum, which has a honeycomb type lining, is positioned up against the diaphragm and is the chamber where a magnet can be placed to try and prevent Hardware Disease. If the cow swallows a metal object, it falls into the reticulum where the magnet can attach to it and prevent the object (such as a nail or wire) from piercing through the diaphragm into the heart.

What is a frightened sheep most likely to do?

Run Sheep are a fight or flight species and will run when frightened.

When a horse is feeling threatened or is resisting restraint, what is its first instinct to do?

Run away Horses typically will try to run away first if possible, but if they are cornered and feeling threatened with no escape, they will also kick, bite, or rear up. When working with a horse that is not used to handling, it is important to have a plan of attack if the horse is not cooperating to avoid injury.

You volunteered at the zoo and spent the day handling the tortoises. A day later you have gastrointestinal symptoms. Reptiles are a major source for which of the following?

Salmonella Salmonella is a Gram-negative rod of the family Enterobacteriaceae. It is transmitted via the fecal-oral route. Reptiles and sometimes birds can be chronic carriers of Salmonella. Turtles, iguanas, and snakes are especially known to harbor the bacteria.

Which organism causes equine protozoal myeloencephalitis (EPM) in the horse?

Sarcocystis neurona EPM is caused by the aberrant migration of Sarcocystis neurona through the central nervous system of the horse. This causes variable clinical signs such as ataxia and muscle atrophy in the horse.

The pulse oximeter measures what variable?

Saturation of hemoglobin with oxygen Pulse oximetry detects the amount of oxygen present on hemoglobin and is expressed as a percentage (i.e. 97% saturation). This monitor measures saturation by using the different wave lengths of saturated and desaturated hemoglobin. This is different than the amount of pressure that oxygen exerts on a vessel (i.e. the partial pressure of oxygen).

Which test checks for keratoconjunctivitis sicca?

Schirmer Tear Test The Schirmer Tear Test is the test for keratoconjunctivitis sicca (KCS), or most commonly known as "dry eye". Tonometry is the measurement of intraocular pressures and tests for glaucoma, which is increased pressure in the eye. Fluoroscein stain checks for corneal ulcers or abrasions; it is a yellow dye that these defects will uptake on the cornea and can be seen under a blue light. Nasolacrimal flush is used to flush/clear out the nasolacrimal duct (tear duct) if it becomes clogged.

Which structure are you most concerned about when giving an injection in a hindlimb?

Sciatic nerve The sciatic nerve starts where the lumbar spine ends and the sacral spine begins. It is located on the underside of the L6-S1 vertebrae. It travels through the ileum and continues along the back and upper thigh area. Care must be taken when giving injections (especially intramuscular injections) in this area as damage to the sciatic nerve can occur.

Which structure is the white part of the eye?

Sclera The sclera is the white fibrous tissue that extends from the cornea (the clear surface of the eye) to the optic nerve in the back of the eye.

What is the term commonly used to describe diarrhea in calves?

Scours Scours is the common term for diarrhea in calves. It can be a serious health problem in calves and is best prevented by good husbandry.

If a veterinarian says an animal has pruritus, which of the following clinical signs is the animal likely displaying?

Scratching Pruritus is the term indicating that an animal is itchy. Scratching is the most common sign of pruritus. Others might include rubbing or head shaking, depending on the location of the pruritus.

This canine patient presents following his neuter surgery. He was neutered at the local animal shelter and the owner did not follow instructions to use an E-collar. He now has an open wound and a swollen scrotum. The scrotum is filled with blood due to self trauma. This could be called which of the following?

Scrotal hematoma "Hematoma" is the term for blood-filled or localized collection of blood outside the blood vessels, usually in liquid form within the tissue (in this case the scrotum). Torsion means to twist or turn (usually when an organ twists on itself such as the mesentery, lung lobe, or testicle), ablation is removal of material from the surface of an object (total ear canal ablation), and atrophy is partial or complete wasting away of a part of the body.

The sac containing the testicles is known as which of the following?

Scrotum The scrotal sac contains the male gonads (testes).

Which of the following arrhythmias is common in horses and can be alleviated by exercise?

Second degree AV block Second degree AV block is a very common arrhythmia in adult horses as a result of high vagal tone. This arrhythmia is "regularly irregular" and often alleviates with exercise. It is more common in athletic horses. Atrial fibrillation is an irregular arrhythmia and a more serious condition that occurs when the SA node isn't firing properly. Electrocardiogram is the best way to determine atrial fibrillation by identifying a lack of p waves. It is treated with Quinidine, an antiarrhythmic. Ventricular fibrillation is a severely abnormal heart rhythm that is most often identified in patients that have undergone cardiac arrest. Third degree AV block, or complete heart block, is rare in horses and usually associated with degenerative or inflammatory changes in the heart. Horses with this condition usually have exercise intolerance or syncope.

Which breed should not receive ivermectin for treating Demodex?

Sheltie Ivermectin should not be given to collie-type breeds due to a possible MDR1 gene mutation that could cause the drug to be toxic. Breeds included for possible ivermectin toxicity include Collies, Shelties, and Australian Shepherds. The saying is "white feet, don't treat".

A 4-month-old Angus calf in a beef herd in Northern California has died. Symptoms before death included lethargy and coughing. You assist with a necropsy and find pale areas in the cardiac muscles (see image). The veterinarian diagnoses white muscle disease. He asks you to assist with collecting blood samples from the herd to test for what deficiency?

Selenium Nutritional myodegeneration (white muscle disease) is associated with low dietary selenium levels and compounded by low dietary vitamin E. Unsupplemented cattle grazing soils of volcanic origin are at high risk, and calves born in these herds may show skeletal or cardiac muscle syndromes. Measurement of whole blood selenium in some of the cows from the herd will indicate if the herd needs supplementation with selenium.

What species, if overworked, is most at risk for hyperthermia?

Sheep Sheep are at risk for hyperthermia due to their thick coat. A sheep's normal body temperature is around 102-103F. Pigs are also at risk for hyperthermia due their increased amount of body fat.

What is the most likely period of time for a scrotal hernia to occur in a stallion?

Shortly after breeding Newborn foals and breeding stallions are most commonly affected with scrotal hernias. In foals, the hernias may spontaneously resolve as long as they are addressed every time they occur. However, there is a risk that the herniated contents can become incarcerated, resulting in a surgical emergency. In breeding stallions, the contents usually become incarcerated and emergency surgery will be needed.

You are asked to restrain a dog for a cephalic draw. How are you going to hold the dog?

Sitting position, holding off and stabilizing a forelimb The cephalic vein courses over the dorsal aspect of the radius and is a common place to draw blood. The position holding the head and pointing the muzzle upward is for jugular draws. The lateral aspect of a pelvic limb is used to draw from the lateral saphenous vein. The dorsal recumbency position is used sometimes for cystocentesis (inserting a needle in the bladder to collect urine)

The vomeronasal organ is closely associated with which sense?

Smell The vomeronasal organ is associated with olfaction, or smell. The Flehmen response sometimes seen in horses occurs when the horse extends its upper lip, facilitating exposure of this organ to better detect smells or pheromones.

Fasciola hepatica requires which intermediate host?

Snail Fasciola hepatica is the liver fluke. It is typically found in wet environments where snails proliferate. Snails are the intermediate host for this parasite, and the most common definitive hosts are sheep and cattle. It is a flat worm, resembling a leaf.

A rabbit presents with a history of sneezing, nasal discharge, and conjunctivitis. The veterinarian suspects the rabbit has a Pasteurella infection, most commonly known as which of the following?

Snuffles Snuffles is caused most often by Pasteurella multocida and results in upper respiratory symptoms and sometimes conjunctivitis in rabbits. It is highly contagious and generally treated with antibiotics. Tularemia is sometimes known as Rabbit fever. Q fever is caused by Coxiella burnetti.

An animal that has been diagnosed with underlying cardiac insufficiency may need to be restricted on intake of:

Sodium Water follows salt, so an animal consuming more sodium is likely to retain more water. This may be detrimental to an animal with underlying heart disease.

Which species is particularly aggressive when with her young, and extreme caution must be used?

Sows In general, pigs are a more aggressive species. The lactating sow is particularly dangerous; and when handling piglets, it is best to remove the sow to a separate area where she cannot hear her piglets.

What is the biggest risk when restraining a rabbit?

Spinal cord trauma Rabbits have very strong hindlimbs that must be restrained properly. This will prevent them from kicking. If not, they will kick and risk severe spinal cord trauma that could result in a broken back or permanent paralysis. A rabbit should never be picked up or restrained by its ears.

What does the term "metastasis" used in veterinary medicine most often describe?

Spread of cancer from the primary site to another site A malignant tumor is synonymous with cancer. Malignant tumors may metastasize, or spread to other sites. Very common areas to which tumors metastasize are the lymph nodes and lungs. This is why lymph node aspirations and chest radiographs are often done as part of cancer staging.

What is the best type of feeding/drinking bowls to give to patients staying in your hospital?

Stainless Steel Bowls It is best to use stainless steel bowls since they are durable and can be sterilized if needed. It also beneficial to patients that may have allergies to plastic.

Ketones in the urine may be caused by unregulated diabetes (diabetic ketoacidosis). What is another potential cause of ketonuria?

Starvation Ketones are intermediate products of fat metabolism. Diabetes is the most common condition in which ketones are produced. Starvation can also cause ketonuria, except without the hyperglycemia.

The main purpose of stimulating the perineal area of puppies and kittens is to:

Stimulate urination and defecation Orphaned puppies and kittens need manual stimulation of the perineal area to help them to urinate and defecate. This mimics the mother dog licking the region to stimulate evacuation.

What is the causative agent of Equine strangles?

Streptococcus equi The most common symptoms of strangles are mucopurulent nasal discharge, fever, decreased appetite, cough, and abscessed lymph nodes that often rupture and drain. Horses with strangles should be isolated for 6 to 8 weeks because the disease is highly contagious. It is caused by the bacteria Strep equi. Moraxella bovis is the cause of pinkeye in cattle. EHV-1 causes abortion in horses.

A guinea pig presents with swollen glands in the neck (see image). What organism most often causes this problem?

Streptococcus zooepidemicus Cervical lymphadenitis is a swelling or abscess of the cervical lymph nodes in a guinea pig's neck. It is most often caused by Streptococcus zooepidemicus. It is a bacteria most commonly transmitted through broken oral mucosa. It is treated by surgical draining and flushing of the abscess. Enrofloxacin (Baytril) is the antibiotic of choice in adult guinea pigs. It is thought that this bacterium could be zoonotic in immune-compromised humans, so you the technician should wear gloves and take extra precautions when treating a guinea pig with a swelling such as this.

What type of muscle is the heart?

Striated muscle, involuntary The three muscle types are cardiac, smooth, and skeletal. Cardiac muscle cells are located in the walls of the heart. This is striated muscle that is under involuntary control. Smooth muscle fibers are located in walls of hollow visceral organs, except the heart and are spindle-shaped. This type is also under involuntary control. Skeletal muscle fibers are striated and are in muscles attached to the skeleton. They are under voluntary control.

Infection of foals with the parasite whose egg is shown in the image below can be associated with diarrhea in young horses. What is the parasite? The microscopic image was taken at 40X; the egg is approximately 50 um.

Strongyloides westeri This image shows a Strongyloides westeri egg. Strongyloides pass from the host in the larvated form and are recognizable as an oval-shaped, thin-shelled embryonated egg. Larvae of Strongyloides westeri are transmitted to foals in the mare's milk. Adult horses rarely have patent infections except when larvae harbored in their tissues migrate into a mare's milk after parturition. The worms are found in the small intestine and may cause diarrhea in young horses. Ivermectin or oxibendazole are effective in treatment of S. westeri.

Which of the following is NOT considered a clinically important large strongyle in the horse?

Strongylus equorum Strongylus vulgaris, Strongylus edentatus, and Strongylus equinus are the large strongyles of horses. Strongylus equorum does not exist. Although Parascaris equorum is a relevant parasite (round worm) in foals,

A 4-year old male Thoroughbred horse presents for colic due to verminous arteritis caused by an equine parasite. An egg of this parasite is shown below. What is the parasite that causes verminous arteritis of the cranial mesenteric artery in horses?

Strongylus vulgaris Colic with an associated painful mass at the root of the mesenery is seen with verminous arteritis, caused by damage to the cranial mesenteric artery and its branches by Strongylus vulgaris. The strongyle egg shown in the picture is an extra hint. A number of anthelmintics are effective including benzimidazoles, pyrantel and ivermectin.

What organism is verminous arteritis associated with in the horse?

Strongylus vulgaris S. vulgaris is a large strongyle that causes arteritis and thrombosis of the vessels of the GI tract in the horse and eventually can result in segmental ischemia or infarction of the bowel wall. This, in turn, results in signs of colic in the horse. Fortunately, with the advent of modern anthelmintics, this form of colic is not common.

The veterinarian asks you to please administer an FVRCP vaccination to a cat that is boarding. Where should you administer the vaccination?

Subcutaneous on the lateral aspect of a forelimb as distal as possible Because of the risk associated with feline vaccine-associated sarcomas, it is recommended to give feline vaccines on the lateral aspect of a limb as distal as possible. The intrascapular region and nape of the neck area should be avoided.

The interns at the hospital where you work are required to "SOAP" their patients daily. What does this stand for?

Subjective, Objective, Assessment, Plan SOAP stands for Subjective, Objective, Assessment, Plan. Subjective includes the history; Objective includes the physical exam and other information gathered; Assessment includes the problem list and rule outs; and Plan includes the next steps in the evaluation/plan.

What is a ranula?

Sublingual mucocele A ranula is a fluctuant swelling of connective tissue consisting of collected mucin from a ruptured salivary gland duct. They are found under the tongue.

What is the best way to administer long-term medications to the eye of a horse with a severe corneal ulcer?

Subpalpebral lavage system The horse has very strong palpebrae (eyelids) that tend to spasm and close when they are manually manipulated, especially if the eye is painful from a corneal ulcer. The subpalpebral lavage system is placed under the eyelid (not in contact with the cornea) and a long tube extends to the withers. This allows the frequent administration of medications without touching the eye. This is the best method if long term eye medications are necessary.

In the horse, the condition known as Sweeney is caused by damage to which structure?

Suprascapular nerve Sweeney is the term for atrophy of the shoulder muscle in the horse. It is caused by damage to the suprascapular nerve. This nerve is responsible for innervation to the infraspinatus and supraspinatus muscles, which are found in the scapula (shoulder blade).

What is collected when performing an arthrocentesis?

Synovial fluid Arthrocentesis is inserting a needle into the joint space to collect synovial fluid (joint fluid). This procedure is performed by a veterinarian.

Parrots have the unique ability to mimic sound. What anatomical structure in birds is responsible for producing sound?

Syrinx The syrinx is considered to be the voice box of parrots. They don't have a larynx, like mammels. The syrinx is located between the base of the tongue and the trachea. The choana is the slit-like opening in the roof of the bird's mouth that connects to the sinus cavity in the skull. The cloaca is the opening where the feces, urine, and reproductive activity exit the body.

When handling a raptor, what is the anatomic structure that can cause the most damage to a handler?

Talons In the birds of prey, the talons are very sharp and dangerous and need to be secured. While the beak and wings are also "weapons", the mouth is soft except the point of the beak, and the jaw isn't very strong. In the raptorial species, gloves are often used and needed as part of the restraint method.

What is the name of the reflective layer behind the retina that allows animals to have improved night vision?

Tapetum Domestic animals have a tapetum to help them see in dim light. Pigs do not have a tapetum. The tapetum reflects light back through the retina, increasing the light available to photoreceptors. This is a photo of a normal canine fundus. Note the reflective green tapetum.

A healthy 1-year old Labradoodle presents for a wellness exam. You are asked to evaluate the dog's fecal float; you find the structures seen in the image below (shown from 40X magnification, each is about 40 um in diameter). What are they?

Tapeworm eggs These are Taenia eggs, although it is frequently not possible to distinguish them visually from Spirometra. Adult cestodes in the intestine of dogs and cats rarely cause serious disease but potential signs include malaise, increased appetite, colic, or mild diarrhea

What does bog spavin describe?

Tarsocrural effusion Bog spavin is a term used in the equine field. It describes the accumulation of synovial fluid in the tarsocrural joint. Rotation of the coffin joint occurs with laminitis. "Bowed tendon" and tendonitis are the descriptive terms for inflammation of a tendon. If in the correct location the abscessed hoof wall may be termed a subsolar abscess. These result in extreme pain and lameness and must be distinguished from laminitis.

The "hock joint" of a horse is more appropriately identified as what structure?

Tarsocrural joint The hock joint, more specifically called the tarsocrurual joint in the horse, is a high-motion joint composed of numerous bones.

Which essential amino acid is critical for cats?

Taurine Taurine deficiency in cats is associated with dilated cardiomyopathy (DCM) and retinal degeneration (think taurine Deficiency = DCM).

What are hoof testers used for in horses?

Test for pain The hoof tester is use to apply pressure over parts of the hoof, to see if a pain response results.

If a horse gets a cut on loose barbed wire or steps on a rusty nail, which disease would be the biggest risk?

Tetanus Places where horses are kept (pastures, barns, etc.) should be free of loose nails, barbed wire, or other things that may penetrate the skin and cause infection with Clostridium tetani. Horses should be vaccinated against tetanus. Rhodococcus is a gram-positive bacterium and is often found in dry and dusty soil and is most commonly transmitted via inhalation. Clostridium chauvoei is the cause of blackleg in cattle and is a soil-borne bacterium spreading through food, water, and wounds. Horses are not usually susceptible to blackleg bacteria and are protected by natural resistance. Leptospirosis may be transmitted to horses through ingestion of contaminated water or soil. The environment becomes contaminated through infected urine and is found most often in damp ground where the bacteria can most easily survive.

Which of the following conditions causes muscle rigidity and a "saw horse"-like stance?

Tetanus Tetanus, also known as "lock-jaw", causes muscle rigidity. Animals with tetanus are light-sensitive and have a "saw horse" stance. Tetanus is caused by the bacteria Clostridium tetani and can be present in the soil, gaining access to the body through an open or penetrating wound. Horses should be vaccinated against tetanus.

If a horse receives a cut on the leg from barbed wire and is not vaccinated, what is the protocol for preventing tetanus?

Tetanus antitoxin within 24 hours A horse that is potentially exposed and not vaccinated should receive the tetanus antitoxin within 24 hours of injury. This will provide some temporary immunity for about 2 weeks. If the wound is still present at that time, the vaccination is given again. A horse that actually has tetanus is given both tetanus toxoid and antitoxin.

NAVTA stands for which of the following?

The National Association of Veterinary Technicians in America This is the correct name of the organization that represents veterinary technicians and assistants in the United States.

Capnography is a means of measuring what?

The amount of carbon dioxide in respiratory gases Capnography is a means by which carbon dioxide is measured in gas such as inhaled or exhaled respiratory gases. Capnography measures CO2 tension by passing an infrared light beam across a measuring chamber in which a stream of exhaled gas passes. A photodetector on the other side of the chamber measures the intensity of light that is transmitted. The intensity of light transmission is inversely related to the concentration of CO2 in the exhaled gas sample.

Meningoencephalomyelitis is an inflammatory neurologic condition that affects the central nervous system. What comprises the central nervous system?

The brain and spinal cord The central nervous system is comprised of the brain and spinal cord. In most cases, when a neurologic disease is said to be a "central" problem, it is suspected to involve the brain. The peripheral nerves are part of the peripheral nervous system (PNS). Sensory and motor neurons make up the PNS. Lower motor neurons are what connect the spinal cord to the muscles and relay the signals. The vagus nerve (cranial nerve 10) is an important peripheral nerve in the PNS. The cerebellum is part of the brain and is located just above the brainstem. It is responsible for coordination of voluntary movement.

A woman adopts a pet from the humane society and you are reading through the record from the shelter. The record says the cat presented with a history of pyrexia. What does this mean?

The cat had a fever Pyrexia is an elevated temperature above the normal range.

Which of the following is true of a dog with polycythemia?

The dog has an increased hematocrit Polycythemia is an increase in the absolute number of circulating red blood cells (which results in an increased hematocrit or PCV). This is not the same as an increased hematocrit due to hemoconcentration from dehydration. A low red blood cell count is anemia. A low platelet count is thrombocytopenia. An increased white blood cell count is leukocytosis.

What does the term "euthanasia" literally mean?

The good death The word euthanasia is Greek and means "good death". Euthanasia is defined as an easy and painless death, or an act or method of causing death painlessly, to end suffering. The term for ceasing of the heartbeat is asystole.

Horses have unique structures that are paired diverticuli of the Eustachian tube located near the pharyngeal region of the horse. What is the common name of these structures?

The guttural pouches The guttural pouches are blind pouches located in the pharynx of the horse. The function of these pouches is unknown but numerous vital cranial nerves and blood vessels course through these structures.

A woman presents with her 10-week-old kitten that has been experiencing diarrhea. She brought a fresh fecal sample and you are checking it while the doctor is performing his exam. You see the following under the microscope (see image). What do you tell the doctor when he asks about the fecal sample?

The kitten has roundworms. This is a photo of a Toxocara egg, which is a roundworm. Roundworms are very common in kittens and puppies.

The menace test will elicit which response? Which two nerves is this testing?

The menace test will elicit which response? Which two nerves is this testing? The menace is when you wave your hand to the eye to try and elicit a blinking response. This is testing vision via the Optic nerve (Cranial nerve II). I-Olfactory nerve (smell) II-Optic nerve III-Oculomotor nerve IV-Trochlear nerve V-Trigeminal nerve (sensory to the face) VI-Abducens nerve VII-Facial nerve VIII-Vestibulocochlear nerve IX-Glossopharyngeal nerve (sensory for gag reflex) X-Vagus nerve (motor for gag reflex) XI-Accessory nerve XII-Hypoglossal nerve

Which of the following is true regarding Dirofilaria immitis, the parasite that causes heartworm?

The mosquito is the intermediate host The dog is the definitive host of D. immitis, and the mosquito is the intermediate host. The definitive host harbors the adult, sexual stage of a parasite. The intermediate host harbors larval, asexual, or immature forms of a parasite. A paratenic host (also known as a transport host) indicates a type of intermediate host where a parasite does not undergo development into the next stage. A reservoir host is another vertebrate host for a parasite that serves as a source of infection for people or domestic animals. For this parasite, the mosquito is the intermediate host but not a paratenic host because the first, second, and third larval stages are found within the mosquito.

Regarding blood draws, which of the following is true?

The needle should be held parallel to the vein at a slight angle with the bevel facing up There should be no air in the syringe before drawing blood. The bevel is held facing upward and the syringe held parallel to the vein at a slight angle, depending on how superficial the vein is.

What does it mean if an animal is a paratenic host of a parasite?

The parasite does not undergo development on the animal A paratenic host is also known as a transport host. This indicates a type of intermediate host on which a parasite does not undergo development into the next stage. The definitive host harbors the adult, sexual stage of a parasite. The intermediate host harbors larval, asexual, or immature forms of a parasite. A reservoir host is another vertebrate host for a parasite that serves as a source of infection for people or domestic animals.

How can you differentiate a male tortoise from a female?

The plastron is concave in males In order for the males to be able to mount the females, their plastron is concave. This allow for a good fit over the convex shape of the female's shell.

Ectopic pregnancy means which of the following?

The pregnancy is occurring outside of the uterus Ectopic pregnancy occurs when a fertilized egg has implanted outside the uterus, most often in the fallopian tubes. Mostly these pregnancies are not viable and pose a bleeding risk which can lead to death.

Where would you give a sub Q injection in a snake?

The proximal 1/3 of the snake along the lateral edge of the ventral scales. Due to the portal system of reptiles, it is important to give injections towards the head. The drug will distribute much more slowly and evenly. The dorsal and ventral scales meet along the lateral sides. This is where the drug can be injected.

What is the "chief complaint" of a client?

The reason why they brought the pet The chief complaint is the reason why the pet was brought in for exam. The chief complaint is not always the most severe problem. If an animal is currently being treated for congestive heart failure and is doing well but comes in for eye discharge, then the chief complaint is eye discharge (even though the heart problem is a more severe problem). Signalment is the description of the animal which includes age, breed, sex, and if they are spayed/neutered.

What maintains the membrane potential in excitable cells?

The sodium-potassium pump The principal ions involved in an action potential are sodium and potassium cations. During depolarization ion channels allow sodium to enter the cell, and potassium to leave the cell. The sodium-potassium pump then helps to restore the normal resting membrane potential. The sodium-potassium pump maintains the normal ratio of ion concentrations across the membrane. Calcium cations and chloride anions are involved in a few types of action potentials, such as the cardiac action potential.

A pet-sitter is holding a 4-year old male intact Bichon while the veterinarian examines a rash on the dog's abdomen; you are assisting with a skin scraping. The dog reaches back and bites the pet-sitter on the arm. She had insisted on holding the dog for examination despite the posted sign (see image). This dog has been known to be aggressive in the past. If the pet-sitter seeks compensation for being bit, who is most likely to be found liable for her injury?

The veterinarian Even though the owner insisted on holding the dog for the exam, the veterinarian could still be liable for the owner's injury. While most clients would not pursue damages for an incident such as this, courts have ruled that veterinarians are liable if a pet hurts its owner while the two are located on the premises of a veterinary hospital. The best solution to prevent this is to have a technician or assistant trained to restrain animals hold the patients for examination. It is a good idea for a veterinary practice to post a sign in each exam room stating that owners please not ask to restrain pets during exam and this rule should be enforced. In animals that are known to be aggressive, a muzzle should be placed prior to the exam for the safety of all involved.

The veterinarian is performing an orthopedic exam and says a dog has positive Ortolani sign. What corresponds with this?

The veterinarian is performing an orthopedic exam and says a dog has positive Ortolani sign. What corresponds with this? Ortolani is the palpable sensation of gliding the femoral head in and out of the acetabulum and suggests joint laxity, most often seen in hip dysplasia. A "drawer sign" would correspond with a cranial cruciate rupture.

Which of the following statements is NOT included in NAVTA's Code of Ethics for the Profession of Veterinary Technology?

The veterinarian must assume accountability for individual professional actions and judgements of the veterinary technician The statement that the veterinarian must assume accountability for individual professional actions and judgements of the veterinary technician is not represented in the NAVTA Code of Ethics. The correct statement is as follows: Assume accountability for individual professional actions and judgments. This means that you are reliable for your own actions.

You are working in a laboratory where the rats are getting "ringtail." Which of the following may be contributing to this condition?

The wire-bottom cages The correct answer is the wire-bottom cages in which the rats are housed. Ringtail is an annular constriction of the tail found in weanling rats and rats kept in wire-bottom cages. The condition causes the tail to constrict, undergo dry gangrene, and fall off. Factors contributing to ringtail include increased environmental temperatures, low humidity, impaired blood supply to the tail, and possibly drafts. The tail stumps usually heal with no complications. Increased humidity, low temperature and too much nesting material would not typically contribute to the development of ringtail.

You see that the veterinarian you are working with has a patient scheduled to come in today for PU/PD. Which of the following complaints do you expect the owner to have?

Their animal is drinking and urinating excessively PU/PD is a commonly used medical abbreviation that stands for polyuria (increased urination) and polydipsia (increased thirst or drinking).

Why are guinea pig sows prone to C-section if they are bred for the first time after 6 months of age?

Their pelvic bones fuse between 4 and 6 months. Guinea pigs must be bred for the first time prior to 6 months of age because their pelvis must be flexible enough to allow the the fetus to pass through the birth canal. Their pelvic bones fuse at 4 - 6 months. If this occurs and the sow is bred, dystocia will occur.

Why are polyurethane catheters more appropriate for long-term (> 7 days) use in horses?

They are the least thrombogenic Polyurethane catheters (such as those made by Mila) are the least reactive and thrombogenic and, if placed correctly, can be left in place for more than a week. Polyurethane is a flexible material. All IV catheters require maintenance and monitoring.

What is true regarding chelonians?

They don't have a diaphragm Chelonians are turtles and tortoises. They do not have a diaphragm. They have eyelids, and also have a nictitating membrane. They have a spleen. They have a bladder, and bladder stones are not uncommon in pet tortoises.

The fetus does most of its growing during which time period?

Third trimester Most of the development of the fetus occurs in the first trimester; most of the growth occurs in the third trimester.

What is the best way to restrain a unwilling rat for an exam?

Thumb and forefinger under front legs, other hand at base of tail Rats require restraint when they have not been handled much. The best restraint method is to place your thumb and forefinger under their front legs while applying gentle tension on the base of the tail. Rats cannot be handled by the tip of the tail because they can slough the skin, leaving the vertebrae exposed. Scruffing a rat produces an exaggerated fear response and is best not done.

Why do you use a heparin flush to flush an IV catheter?

To prevent clot formation in the catheter Heparin is an anticoagulant and helps to prevent clot formation in the IV catheter. A catheter should be flushed prior to and after giving IV medications to make sure it is functioning properly, and then flushed every 4 hours when not in use to help prevent clot formation. Phlebitis is inflammation of the vein; heparin does not prevent this.

Why should pigs be fed from a bin or trough and not on the ground?

To prevent development of trichobezoars A trichobezoar is a ball of swallowed hair that collects in the stomach and can fail to pass through the intestines. Pigs are constantly shedding hair, so it accumulates on the ground; when eating food off that ground, pigs will ingest enough over time to put them at risk for formation of a trichobezoar.

What device or instrument is typically used to help diagnose glaucoma?

Tonometer The tonometer (Tonopen) is used to measure intraocular pressures. Glaucoma is the condition of raised intraocular pressures. Normal eye pressures for dogs is typically between 10-20 and for cats 10-25 but these must be interpreted in light of clinical symptoms. Eye pressures lower than 10 can be caused by uveitis (inflammation in the eye).

Which of the following is a chelonian?

Tortoise Turtles and tortoises are from the order Chelonia, and are often referred to as chelonians.

Which parasite is commonly transmitted transplacentally to puppies?

Toxocara Toxocara canis (roundworms) commonly infect puppies via the transplacental route.

Ocular larval migrans is a zoonotic disease caused by which parasite?

Toxocara canis

It is recommended that all puppies be treated for which of the following parasites?

Toxocara canis The correct answer is Toxocara canis. The most common method of transmission of Toxocara canis is transplacentally. Larvae in the somatic tissue of a pregnant bitch are mobilized during pregnancy and infect the fetuses transplacentally.

A direct life cycle is seen in which of the following parasites?

Toxocara canis Toxocara canis (roundworm) has a direct life cycle. In a direct life cycle, the immature parasite can infect the same host from which it came (i.e., this worm lives in the intestine, eggs are passed in feces, and the eggs are passed to the next host directly). The other parasites all require an intermediate host. Intermediate hosts are: Dirofilaria (mosquito), Dipylidium (flea), and Fasciola (snail)

Which parasite has the feline as its only definitive host?

Toxoplasma gondii The definitive host of T. gondii is the cat, but the parasite can be carried by many warm-blooded animals. D. immitis, A. caninum, and Giardia lamblia can have multiple hosts. T. vulpis is canine specific.

Which small animal disease has been implicated as a specific danger to pregnant women?

Toxoplasmosis The Toxoplasmosis tachyzoites can migrate transplacentally and harm the fetus. Transmission can occur by eating undercooked meat or by inadvertently ingesting oocytes from cat feces. Therefore, it is advised that pregnant women not clean litter boxes, or if they must, wear personal protective equipment while doing so.

Which of the following is another term for the type of parasite known as a fluke?

Trematode Trematodes, or flukes, are leaf-shaped flatworms with unsegmented bodies. Adults are hermaphrodites. They primarily are found in the intestinal tract, liver, and lungs. Examples include the liver fluke of cattle (Fasciola hepatica) and the lungworm of cats (Paragonimus kellicotti). Nematodes are roundworms, ascarids are a type of nematode (roundworm), cestodes are tapeworms, and protozoans are single-celled organisms that may be parasitic.

Ivermectin would not be used when treating which parasite?

Trematodes The correct answer is trematodes (flukes). Ivermectin is ineffective against flukes. Ivermectin has activity against ticks, mites, nematodes, and even lice. Ivermectin works by stimulating GABA which is an inhibitory neurotransmitter.

The elbow is extended by which muscle?

Triceps brachium The triceps extends the elbow. The biceps supinates the forelimb and flexes the elbow.

Rabbits are prone to hairballs. What is the medical term for a hairball?

Trichobezoar Tricho refers to hair, bezoar is the compaction, "ball" that can result from different substances. Phytobezoar is a "plant - ball". Piloerection is used to describe when hair stands up. Like when a cat or dog is fearfull. Dermatophyte is the general term for skin fungus.

Regarding lice parasites, the biting louse found in canines is known as which of the following?

Trichodectes Trichodectes canis is the chewing or biting louse in dogs. Hematopinus asini is the sucking louse in cattle. Damalinia bovis is the chewing louse in cattle (also called Bovicola). Felicola subrostratus is the chewing or biting louse in cats. Lice are ectoparasites; infestation with lice is known as pediculosis. Trichodectes canis can also serve as an intermediate host for Dipylidium caninum (although more commonly, the intermediate host is a flea).

Which disease does an intradermal caudal skin fold test check for in cattle?

Tuberculosis The skin fold test checks for tuberculosis, an infectious granulomatous disease caused by bacteria from the genus Mycobacterium. A small amount of tuberculin is injected into a caudal fold at the base of the tail in cattle. A diffuse swelling at the site within 48-96 hours constitutes a positive reaction.

You are in the clinic parking lot when you are approached by 2 large dogs who are about to attack you. They are about 25 yards away. You should immediately do which of the following?

Tuck yourself into a ball on the ground and protect your face and neck If you do not have a way of defending yourself and cannot get away, the best thing to do is to crouch into a ball and protect your face and neck. By taking this stance, you are not a threat to them and they are most likely to back away or not be as aggressive. If you run, the dogs are likely to chase you and will likely catch up to you causing more bodily harm. They will likely be more aggressive if you are standing there or hitting at them because they may view you as a threat.

Torsion of an organ means that what has happened to it?

Twisted on itself Examples of torsions are testicular torsion (twisting of the spermatic cords), gastric torsion (gastric-dilatation volvulus), mesenteric torsion (twisting of the mesentery and cutting off of blood supply to the intestines), lung-lobe torsion, etc.

The eardrum is also known as which of the following?

Tympanum The proper term for the eardrum is the tympanic membrane, or tympanum.

What is the most common blood type in cats in the United States?

Type A Cats most commonly have type A blood. Some cats have type B blood, many of which are exotic-type species. Type B cats have strong anti-A alloantibodies, so type A blood given to a type B cat results in life-threatening acute hemolytic transfusion reactions. A cat should always have a blood type test before a transfusion.

When a tortoise is kept in an indoor enclosure, what is the most important light in that enclosure?

UVB A UVB light is needed to properly process and absorb calcium from the diet. Sunlight is the natural way to process calcium. Other lighting is also needed for proper digestion. A basking bulb and heat lamp help regulate temperature, and UVA light helps regulate activity.

Where would you find a Cuterebra parasite?

Under the skin Cuterebra are flies whose larvae infest the skin of rodents, squirrels, rabbits, dogs, and cats. Typically a lump is seen with a small hole under the skin from with the larva breathes.

The word "idiopathic" is commonly used in veterinary medicine. What does this word imply?

Unknown cause Idiopathic means that the cause or mechanism for a disease is unknown. Nosocomial means acquired in the hospital

Gout is an accumulation of which of the following?

Uric acid Gout occurs when uric acid accumulates, usually due to poor nitrogen metabolism and excretion.

Which of the following must be sterile before use?

Urinary catheter Urinary catheters must be sterile so that bacteria are not introduced into the bladder. The other things listed should be clean but don't have to be sterile. If you were planning on doing a culture of the ears, a sterile swab would be warranted but not needed for cytology.

Micturition is a medical term for which of the following?

Urinating Micturition is the act of voiding urine, or urinating. Emesis is another term for vomiting. Stranguria is the term for straining to urinate.

What does isosthenuria indicate?

Urine that has the same specific gravity as the plasma Isosthenuria describes urine that has the same concentration (specific gravity) as the plasma. Urine-specific gravity describes the concentration of a fluid as compared to water (specific gravity of water = 1.000). Isosthenuria may be observed during renal failure, as the kidney cannot concentrate the urine.

Which condition occurs least commonly in the horse?

Urolithiasis As compared to small animals, urolithiasis is uncommon in horses. If they do develop stones in their bladder, it is usually secondary to formation of calcium oxalate crystals. Enterolithiasis and intestinal incarcerations are common reasons for colic in horses and usually require surgical intervention. Laminitis is a very serious and often life-threatening condition in horses. The pathophysiology of laminitis is poorly understood but involves inflammation of the lamina of the feet. It can involve all feet or even just one foot. There are multiple conditions that can lead to laminitis, such as endotoxemia, grain overload, and pneumonia.

What is the most accurate method to estimate a horse's weight out on a farm call?

Use a weight measuring tape Specialized measuring tapes are commonly used to approximate the weight of a horse. There are markings that show weights and the tape is wrapped around the barrel of the horse just caudal to the withers.

What is the proper way to restrain a rat for routine procedures?

Using two hands, one around the thorax and under the front legs, and one hand grasping the base of the tail. The proper restraint for a rat involves both hands. One hand grasps around the thorax under the front legs, and the other hand keeps downward pressure on the base of the tail. This handhold prevents the rat from being able to turn in it's skin and bite you. It is very important that the grasp of the tail is at the base, as the tail will deglove if pulled on any more distal than the base.

A dog presenting with a head tilt and loss of balance most likely has disease associated with which nerve?

Vestibulocochlear Head tilt and loss of balance are symptoms of vestibular syndrome, which is most often assoiated with damage to the vestibulocochlear nerve (CN VIII). Other symptoms may include nausea and nystagmus.

The designation of VTS in someone's credentials stands for which of the following?

Veterinary Technician Specialist VTS is a designation given to a credentialed technician in their field of expertise after spending 3 years full time in that specific field and passing a rigorous board examination. There are several different specialities that can be achieved by credentialed veterinary technicians looking to go beyond their state credentials.

The Veterinary Technician Code of Ethics contains which of the following statements?

Veterinary technicians shall protect confidential information provided by clients. The veterinary technician oath includes these following points: Veterinary technicians shall aid society and animals through providing excellent care and services for animals. Veterinary technicians shall prevent and relieve the suffering of animals. Veterinary technicians shall promote public health by assisting with the control of zoonotic diseases and informing the public about these diseases. Veterinary technicians shall assume accountability for individual professional actions and judgments. Veterinary technicians shall protect confidential information provided by clients. Veterinary technicians shall safeguard the public and the professional against individuals deficient in professional competence or ethics. Veterinary technicians shall assist with efforts to ensure conditions of employment consistent with the excellent care for animals. Veterinary technicians shall remain competent in veterinary technology through commitment to life-long learning. Veterinary technicians shall collaborate with members of the veterinary medical profession in efforts to ensure quality health care services for all animals.

You are on a house call with the veterinarian to evaluate some goats that are having "night blindness" per the owner. Upon arrival you can see that the goats have a dull haircoat, are very thin, and have nasal discharge. What deficiency can cause these symptoms?

Vitamin A Vitamin A deficiency causes a rough dry haircoat, thick nasal discharge, diarrhea, and can also lead to night blindness. Vitamin E can cause white muscle disease (a nutritional muscular dystrophy). Zinc deficiency can cause hypersalivation, chronic skin problems, testicular hypoplasia, and deformed hooves. Iodine deficiency most often causes a goiter.

Mulberry heart disease in swine is caused by which of the following?

Vitamin E deficiency Mulberry heart disease is a common form of Vitamin E and selenium deficiency in pigs. It typically occurs due to a lack of these vitamins in the feed. It can cause sudden death and difficulty breathing due to fluid accumulation around the muscles, including the heart, in which the pericardial sac becomes grossly distended. It also causes cardiac muscle necrosis.

Which of the following is not a nutrient that provides energy?

Vitamins There are six categories of nutrients. Proteins, fats, and carbohydrates provide energy. Water, vitamins, and minerals do not.

Which of the following vitamins are fat-soluble?

Vitamins A, D, E, and K The fat-soluble vitamins are A, D, E, and K. Vitamin C (ascorbic acid) and the B-complex vitamins are water-soluble.

What is a symptom of myasthenia gravis?

Weakness and muscle fatigue In Myasthenia gravis, there are few acetylcholine receptor sites (AChR) on the muscles, and acetylcholine is broken down before it can fully cause muscle stimulus. This results in muscle weakness, and with this condition, animals experience severe muscle fatigue.

Which of these is NOT part of the regular signalment for a pet?

Weight The typical signalment includes age, spay/neuter status, and breed. (i.e. a 9-yr female spayed Rottweiler)

Hamsters are susceptible to proliferative ileitis. What is the common name for this debilitating disease?

Wet tail Wet tail is the common name given to the disease that plagues hamsters in stressful situations. It is called "wet tail" because of the diarrhea that soils the rear of the hamster. Caused by a mulitude of issues (stress being one of the most common), this condition is contagious and debilitating. An overgrowth of bacteria causes profound diarrhea.

A castrated male goat is known as a:

Wether Wether is the term for a neutered male goat. Buck is an intact male goat. Barrow is a castrated male pig. Gelding is a castrated male horse.

What is the shape of a bacillus such as the bacteria that causes anthrax, Bacillus anthracis?

What is the shape of a bacillus such as the bacteria that causes anthrax, Bacillus anthracis? A bacillus is a rod- or cylindrical- shaped bacterium. Circular or round bacteria are cocci.

What is a pulse deficit?

When some heart beats do not result in a palpable pulse A pulse deficit is the condition in which a peripheral pulse rate is less than the ventricular contraction rate because some heart beats do not create a palpable pulse. This can be indicative of a lack of peripheral perfusion.

Under which circumstances are pigs most likely to bite each other's tails?

When the facility is over-crowded Tail-biting is most likely to occur when a hog farm is over-crowded.

Which area of the horse is generally referred to as the withers?

Where the dorsal neck joins the thorax The withers is the ridge between the shoulder blades. It is the standard place to measure the height of a horse. It is formed by the dorsal spinal processes around the third to eleventh thoracic vertebrae.

An owner presents her puppy for severe itching. The veterinarian diagnoses Sarcoptes mites with a superficial skin scraping. The owner wants to know if this mite is contagious to her and her children. What should you tell her?

Yes, there can be a temporary transmission from her dog to those with direct contact, but the mites are host-specific and the infection is temporary Sarcoptic mites (scabies) from the dog can transfer to humans and cause skin irritation. However, since humans cannot host the canine sarcoptic mite, it dies off without treatment being required for the human, just the dog. Humans have their own sarcoptic mite that is contagious between humans. Notoedres cati causes feline scabies and is host-specific to the cat. It can also temporarily infect humans.

Which breed is most likely to have a portosystemic shunt?

Yorkshire terrier A portosystemic shunt is an abnormal vessel that allows blood to bypass the liver. The Yorkshire terrier is overrepresented for this condition, but the condition can be seen in any breed. It is most common in toy breeds and dogs that are the "runt" of the litter.

What is the most commonly used method for fecal parasite testing for dogs and cats?

Zinc sulfate centrifugation Zinc sulfate centrifugation is the most common and best of these choices. Technique: Zinc sulfate (ZNSO4) solution is added to 2-3 grams of feces. It is mixed and then centrifuged for a few minutes on high speed. A loop can then skim the surface and place the sample on a slide for examination, or the tube can be filled to the top with more zinc solution and a coverslip set on top for a few minutes. Then the coverslip can be placed onto a slide for examination. Sometimes a drop of iodine is added to the sample to help identify the eggs more easily. Baermann is useful for detecting lungworm and some other nematodes. Direct smears can help detect trophozoites, but they are not useful for finding tapeworm or nematode eggs, or coccidia. Cytology of feces would mostly just demonstrate the bacterial flora.

A male rabbit is also referred to as a ______.

buck A female rabbit is a doe, while a male rabbit is a buck.

Which of the following clinical signs is LEAST COMMONLY observed in horses with colic?

fever Colic is a non-specific term describing abdominal pain. There are many causes of colic in the horse resulting in various clinical signs, including the ones listed here. Of these listed, fever is the least commonly seen. Occasionally, diseases such as infectious peritonitis or metritis can result in fever and signs of colic but are less common.

The vet thinks there is a fox tail in a dog's ear canal. What instrument will the doctor need?

otoscope An otoscope is used for examining ears. An ophthalmoscope is used to perform eye and fundic exams. An endoscope is used to perform endoscopy: the scope is placed down the esophagus, into the stomach and upper small intestine for visual aid and for collecting biopsies. It can also be used to assist in placement of peg tubes or in retrieving foreign bodies in the esophagus and stomach.


Kaugnay na mga set ng pag-aaral

Systems of Linear Equations and Inequalities - Part 1

View Set

American Republic Chapter 1 Activity 5

View Set

U.S. Citizenship and Immigration Services (AMERICAN HISTORY)

View Set

Chapter 6: Networks, Groups, and Organizations

View Set